FAR Becker Final Review

Lakukan tugas rumah & ujian kamu dengan baik sekarang menggunakan Quizwiz!

Shares Authorized, Issued, and Outstanding

- Authorized: Legal number of shares available for sale (maximum). - Issued: Number of shares sold. - *Outstanding: Shares issued less treasury shares (repurchased). Only outstanding shares are entitled to dividends.*

Form and Content of CAFR (CAN CPAS RIDE or SIT)

Reconciliation of Fund Statements to Government-wide Statements Because of the different measurement focus and related basis of accounting used, the governmental fund balance as reported in the balance sheet must be reconciled to net position of government-wide statements as reported in the statement of net position. The difference in measurement focus provides the following reconciling items: *- Add non-current assets.* *- Subtract non-current liabilities.* *- Add internal service fund net assets.* The difference in basis of accounting produces the following reconciling items: - Adjust for *accrual of revenue* accounted for on the full accrual basis of accounting rather than the modified accrual basis of accounting. - Adjust for *accrual of expenses* accounted for on the full accrual basis of accounting rather than the expenditures accrued on the modified accrual basis of accounting. - The *changes in fund balance* displayed in the governmental fund statement of revenues, expenditures, and changes in fund balances must be *reconciled to changes in net position* of the governmental activities column as reported in the statement of activities of the government-wide statement. The difference in measurement focus produces the following reconciling items: - Subtract debt proceeds (not accounted for as other financing sources in the government-wide financial statements). - *Add capital outlay* (not accounted for as expenses in the government- wide financial statements). - *Add internal service fund changes in net position* accounted for in the proprietary funds. The difference in basis of accounting produces the following reconciling items: - Adjust for *accrual of revenue* accounted for on the accrual basis of accounting rather than the modified accrual basis of accounting. - Adjust for *accrual of expenses* accounted for on the accrual basis rather than the expenditures accrued on the modified accrual basis Summary Balance Sheet: *GRaSPP-Fund Balance* Assets (non-current) Liabilties (non-current) Service (internal) fund net position Basis of accounting: Accrued Revenues and Expenses Statements of Revenues, Expenditures, and Changes in Fund Balance: *GRaSPP-Net Change in fund balance* Other financing sources Expenditure-capital outlay (net of depreciation) Service (internal) fund net income Basis of accounting:

Straight-Line Depreciation (Productive Output)

(Cost - Salvage value) / Estimated useful life = Depreciation expense

Form 10-K

60 large accelerated-75 accelerated-90 all others Must be filed annually by US registered companies (issuers). -The filing deadline for Form 10 K is 60 days for large accelerated (larger than 700 million), 75 days for accelerated (75 - 699 million) and 90 days for all others (smaller than 700 million). -The forms contain a summary of financial data, MD&A, and audited financial statements prepared using GAAP.

Operating Loss

A net operating loss (NOL) is treated differently depending on the year it arises: - NOLs arising in 2018, 2019, or 2020 tax years can be carried back five years and carried forward indefinitely. *100% deductible and carryback 5 years.* - NOLs utilized in the five-year carryback period or in 2018, 2019, or 2020 tax years are not subject to a taxable income limitation. - NOLs carried forward to taxable years beginning in 2021 or later are limited to 80 percent of taxable income before the NOL deduction. *In 2021 going forward no carryback is limited to 80% of future income* - Taxpayers can elect not to carry back and just carry forward. - *NOLs arising in 2021 and beyond cannot be carried back but can be carried forward indefinitely.* Taxable income and financial accounting income will differ for the years in which the loss is incurred and carried back or forward. Carryforward benefit: DR Deferred tax asset CR Tax benefit*

State and Local Government Concepts

Fund Categories and Fund Types Eleven fund types *(GRaSPP SE-CIPPOE)* are classified in the following three generic categories: 1.1.1 *Governmental Funds: are current financial resources measurement focus (No fixed assets & long term debt). They use modified accrual accounting.* - *"G"eneral:* The general fund accounts for the ordinary operations of a governmental unit that are financed from taxes and other general revenues. All transactions not accounted for in some other funds are accounted for in this fund. - *Special "R"evenue:* Special revenue funds account for revenues from specific taxes or other earmarked sources that are restricted or committed to finance particular activities of government other than debt service or capital projects. And - *Debt "S"ervice:* Debt service funds account for the accumulation of resources and the payment of interest and principal on all "general obligation debt." - *Capital "P"rojects:* Capital projects funds account for resources used for the acquisition or construction of major capital assets by a governmental unit. - *"P"ermanent:* Permanent funds are used to report resources that are legally restricted to the extent that income, and not principal, may be used for purposes that support the reporting government's programs. *Proprietary Funds: are economic resources measurement focus (Have fixed assets & long term debt). They use full accrual accounting.* - *Internal "S"ervice:* Internal service funds account for goods and services provided by departments on a cost-reimbursement fee basis toother departments. - *"E"nterprise:* Enterprise funds account for the acquisition and operation of governmental facilities and services that are intended to be primarily (over 50 percent) self-supported by user charges. Fiduciary Funds: are economic resources measurement focus (Have fixed assets & long term debt). They use full accrual accounting.* - *"C"ustodial:* Custodial funds usually account for resources in temporary custody of the governmental unit and any fiduciary activities that are not required to be reported in other fiduciary fund classifications. - *"I"nvestment Trust:* Investment trust funds account for external investment pools. - *"P"rivate Purpose:* Private purpose trust funds are used for activities not properly accounted for either as pension or investment trust funds. - *"P"ension (and Other Employee Benefit):* Pension trust funds account for resources of defined benefit plans, defined contribution plans, post-employment benefit plans, and other long-term employee benefit plans. Current Financial Resources Measurement Focus = GRaSSP Financial statement readers are focused on the sources, uses and balances of current financial resources. The focus often includes a budgetary element. The governmental fund types use the current financial resources measurement focus. - *Non-current assets and liabilities are not reported* on the governmental fund types balance sheets. - *Capital outlay expenditures* are reported on the face of the governmental fund types operating statements. - Proceeds from *long-term debt,* or leases that are either contracts that transfer ownership or other than short-term leases and contracts that transfer ownership, are recorded in the governmental funds as *"other financing sources."* - Payment of *principal and interest* are recorded as *"expenditures."* Economic Resources Measurement Focus = SE CIPPOE Financial statement readers are focused on the determination of operating income, changes in net assets, financial position and cash flow. The proprietary fund and fiduciary fund types use this focus. Accounting is nearly *identical to commercial accounting* used in "for profit" entities. *- Non-current assets and non-current liabilities are recorded on the balance sheet.* *- Depreciation expense is recorded.* *Modified Accrual Basis of Accounting = "BAE BAE" (GRaSSP)* The current financial resources measurement focus is accomplished using the modified accrual basis of accounting. The difference between modified accrual and accrual primarily relates to the timing of revenue recognition. *Revenues* are generally accrued when they are both measurable and available *(due and collected within 60 days of year-end).* There are four classifications of non-exchange revenues that serve as the basis for most governmental fund resources. There is no underlying exchange transaction that produces these revenues; the government does not provide a specific service in exchange for the revenue earned: Modified accrual also creates important expenditure recognition differences, including no interest accrual.

Deferred Outflows and Inflows of Resources (GRaSPP SE PAPI)

GASB 63 requires that certain transactions that do not qualify for treatment as either assets or liabilities be accounted for and reported as deferred outflows of resources or deferred inflows of resources. *A deferred outflow of resources is a consumption (using up of a resource) of net assets that is applicable to a future reporting period.* Deferred outflows of resources have a *positive* effect on net position and are reported following assets but before liabilities. *A deferred inflow of resources is an acquisition (purchase) of net assets that is applicable to a future reporting period.* Deferred inflows of resources have a *negative* effect on net position and are reported following liabilities but before equity. Deferred outflow = *+* future consumption (using up of a resource) of net assets Deferred inflow = *-* future acquisition (purchase) of net assets Government-wide Statement of Net Position Governments are encouraged to report net position as the difference between assets plus deferred outflows of resources and liabilities plus deferred inflows of resources. A - L = Net position *Assets* *+ Deferred Outflows of Resources* - *Liabilities* *Deferred Inflows of Resources* *= Net Position* Governmental Fund (GRaSPP) Balance Sheet Governmental funds should present the statement of financial position in a balance sheet format that displays assets plus deferred outflows of resources equal to liabilities plus deferred inflows of resources plus fund balance. A = L + Fund balance Assets + Deferred Outflows of Resources = Liabilities Deferred Inflows of Resources + Fund Balance Total Liabilities, Deferred Inflows of Resources, and Fund Balance Proprietary and Fiduciary Statement of Net Position (SE PAPI) *Proprietary funds are encouraged and fiduciary funds are required* to report net position as the difference between assets plus deferred outflows of resources and liabilities plus deferred inflows of resources (similar to the government-wide statement of net position). Proprietary funds can also use a balance sheet format [(Assets + Deferred outflows of resources) = (Liabilities + Deferred inflows of resources) + Net position]. A - L = NP (SE PAPI) or A = L + NP (SE) *Key take away: Deferred outflows are always added to Assets in determining NP. Deferred inflow are always added to Liabilities in determining NP.* Types of Transactions Accounted for as Deferred Outflows and Inflows of Resources Service Concession Arrangements *Advance payments by vendors* that operate infrastructure in exchange for volume-based fees are displayed as *deferred inflows* of resources until earned. *Similar to deferred revenue (liability).* Hedge Accounting Derivatives used as hedges to mitigate the risk of fluctuations in fair value or cash flows qualify for accounting as deferred outflows of resources and deferred inflows of resources if they are effective. Effectiveness means that changes in the value of a derivative used as a hedge offsets the fluctuations in the fair value or cash flows of the hedged item. A sample entry for the decline (unrealized loss) in value of a qualifying derivative instrument would be: Dr Deferred outflows of resources (loss) $XXX Cr Derivative instrument$XXX This is similar to the use of *other comprehensive* (PUFIER) income when recording *unrealized gains and losses* for a nongovernmental entity. Other Deferred Outflows/Inflows of Resources GASB 65, "Items Previously Reported as Assets and Liabilities," revises the treatment of a variety of transactions previously reported as either assets or liabilities on the statement of financial position to classification as either deferred outflows/inflows of resources or recognition as revenue or expense. Transactions addressed in the statement include: - Refunding of debt - Imposed non-exchange revenue transactions - Government-mandated non-exchange transactions and voluntary non-exchange transactions - Sales of future revenues and intra-entity transfers of future revenues - Debt issuance costs - Leases - Regulated operations - Assets associated with unavailable revenues - Certain changes in net pension liability (GASB 68) - Lease receivable for leases other than short-term leases and contracts that transfer ownership in SE-CIPPOE funds (GASB 87)

*Exchanges Lacking Commercial Substance (U.S. GAAP)*

If projected cash flows after the exchange are not expected to change significantly, then the *exchange lacks commercial substance and a book value approach is used.* - All losses are recognized on exchanges lacking commercial substance. Gains are recognized based on the nature of the transaction: *Can only recognize a gain if it received boot/cash.* - No boot received = No gain - Boot paid = No gain - *Boot received < 25% = Recognize gain in proportion to boot received* - *Boot ≥ 25% of total consideration = Monetary exchange (Both parties to exchange recognize all gains and losses)*

Interfund Activity

Interfund activity is subject to specific requirements related to financial statement display and disclosure and can be classified as follows: - *Reciprocal interfund activity* - *Nonreciprocal interfund activity* Reciprocal Interfund Activity - Includes *exchange-type transactions* between funds. - Interfund loans are expected to be repaid and are accounted for as interfund *receivables and payables (due from/due to).* - Unrealizable balances are reclassified as transfers. - *Interfund services provided and used* represent *sales and purchases* between funds at external pricing. Transactions of this type are accounted for as *revenues and expenses.* Nonreciprocal Interfund Activity - Represents *non-exchange transactions* between funds. - *Interfund transfers of assets* between funds without the exchange of equivalent value represent interfund transfers. These are normally reported as *other financing sources* and uses after nonoperating revenues and expenses. - *Interfund payments of expenses/expenditures* made by one fund on behalf of another fund are *accounted for as reimbursements.* Interfund reimbursements *are not displayed as interfund transactions.* Government-wide Financial Statement Displays of Interfund Activity - *Interfund activity within a particular column* displayed on the governmental activities or business-type activities financial statements (intra-activity transaction between governmental funds and internal service funds) *is eliminated prior to the preparation of governmental- wide financial statements.* - *Interfund activity between columns* displayed on the governmental activities or business-type activities financial statements *(inter-activity transaction between governmental funds and enterprise funds)* is reported as "internal balances" on the statement of net assets and "transfers" (revenues or expenses) in the statement of activities. They are *not eliminated.* - Interfund activity between the primary government and its fiduciary funds should be reported as if between *external parties.*

Operating Capital Lease Versus Finance Capital Leases

Lesse : - Operating capital lease - Financing capital lease Lessor: - Operating capital lease - *some risks* and rewards are transferred to lessee - Financing capital lease: 1. Sales-type capital lease - *all risks* and rewards are transferred to lessee 2. Direct financing capital lease - *most risks* and rewards are transferred to lessee The "OWNES PC" criteria are used to determine the lease classification for the lessee and lessor. If *at least one* of the five OWNES criteria below are met it's a Finance Capital Lease: - The lessee will classify the lease as a finance lease; and - The lessor will classify the lease as a sales-type lease.(O) *Ownership* of the underlying asset transfers from the lessor to the lessee by the end of the lease term. (W) The lessee has the *written option* to purchase the underlying asset; the option is one that the lessee is "reasonably certain" to exercise. (N) The *net present value of all lease payments* and any guaranteed residual value is equal to or substantially exceeds the underlying asset's fair value. (*90 percent* of FV threshold is reasonable.) (E) The term of the lease represents the major part of the *economic life remaining* for the underlying asset. (*75 percent* of economic life threshold is reasonable.) (S) The asset is *specialized* such that it will not have an expected, alternative use to the lessor when the lease term ends. *If any of the OWNES criteria are meet the lessor threats the Finance Capital Lease as a Sales-Type Lease.* If none of the OWNES criteria are met it's an Operating Capital Lease: - The lessee will classify the lease as an operating lease; and - The lessor will classify the lease as a direct finance lease or an operating lease depending on the PC criteria below. - If *both PC criteria are met*, the lessor will classify the lease as a direct financing lease. - If *neither or just one* PC criteria is met, the lessor will classify the lease as an operating lease. *(P) Present* value of the sum of the lease payments, lessee guaranteed residual value not included in the lease payments, and any third-party guaranteed residual value is equal to or substantially exceeds the underlying asset's fair value. *(C) Collection* of the lease payments and any amounts necessary to satisfy residual value guarantees is probable.

CPAS RIDES- Gov't wide FS reconciliation

Net change in fund bal. - Total gov't funds + Capital Outlay + Principal payments on non-current debt - Asset disposals (NBV) - Sources (other financing sources-debt proceeds) + Revenue (measurable but unavailable) - Interest expense (accrued) - Depreciation expense + internal Service fund net revenue = Change in net position of governmental activities

*Notes to the financial statements*

Notes clarify information presented in the financial statements and provide additional detail. *The first note is the summary of significant accounting policies, which includes methods, policies, and criteria (e.g., methods: LIFO, FIFO, straight-line, concentration risk).* The other footnotes proterm-26vide details of the financial statements being anything relevant to decision-makers.

Imputed Interest Notes Payable

Notes receivable and notes payable contain an interest element. Money is not loaned for free or for a below-market interest rate. Notes are recorded at present value when the interest rate is not stated or when the stated interest rate is unreasonably low. The difference between the face amount of the note and the present value of the note is recorded as a discount and amortized over the life of the note.

Units-of-Production (Productive Output)

Step 1: Cost - Salvage value / Total estimated units or hours = Depreciation rate per unit Step 2: Depreciation rate per unit x Units produced or Hrs used in a period = Depreciation expense

Government-wide Reporting

The GASB 34 reporting model (as amended by GASB 63) focuses the reader on both government-wide and fund financial statements using an integrated approach to highlight both the operational and fiscal accountability requirements of the government. The basic structure includes the presentation of: *1. Management discussion and analysis* a letter that presents a brief, objective, and easily readable analysis of the government's activities. *2. Basic financial statements (comprised of government-wide financial statements, fund financial statements, and notes to the financial statements)* *3. Required supplementary information* that includes multi-year pension data, infrastructure data for governments using the modified approach for infrastructure, and budgetary disclosures. Fund financial statements emphasize *fiscal accountability* while government-wide financial statements emphasize *operational accountability.* Financial presentations are integrated by the *reconciliation of fund financial statements to government-wide presentations.* Government-wide financial presentations are prepared using the *economic resources measurement focus* utilizing the *accrual basis of accounting.* The government-wide financials include presentation of governmental activities and business-type activities such as the enterprise funds. Fiduciary funds are excluded from the government-wide financial statements but are included as part of the fund financial statements. A matrix to keep in mind for GASB 34 reporting categorizes our fund structure mnemonic as follows for government-wide reporting: Governmental: GRaSPP S Business Type: E Excluded: CIPPOE Governement-wide financial statements are the: √ Statement of Net Position (Balance sheet) √ Statement of Activities (Income statement) - Other supplementary information (optional) may be included. Optional information includes budget variances and individual financial statements for nonmajor funds. - A *statement of cash flows is not prepared* for government-wide presentations.

Forms 3, 4, and 5

These forms are required to be filed by directors, officers, or beneficial owners of more than 10 percent of a class of equity securities of a registered company.

*Dilutive Securities*

securities that would DECREASE EPS if exercised If X< Avg. stock price then could be exercised If X> Avg. stock price then will not be exercised Some examples of dilutive securities include convertible preferred stock, convertible debt instruments, warrants, and stock options.

Acquiring Company's Adjustments

"CAR IN BIG" 1. (C) CS, (A) APIC and (R) RE are eliminated (subs old books) 2. (I) Investment in Sub is eliminated (parents books) 3. (N) NCI is Created (if not 100% owned) 4. (B) BS of Sub. Adjusted to FV (100% Assets & Liabilities as of acquisition date 5. (I) Identifiable Intangible Assets Adj. to FV 6. (G) Goodwill or Gain is required ---Pay Premium = Goodwill ---Pay less than worth = Gain

*Impairment of Intangible Assets (Patent, copyright, trademark) & PPE*

*Impairment Test Finite Life Intangibles US GAAP:* *-No reversal* Step 1: Undiscounted CF < CV if yes, impaired Step 2: Impairment Loss Calculation: FV - CV = Impairment loss *Impairment Test Indefinite Life Intangibles (e.g., brand) US GAAP:* If FV < CV an impairment loss needs to be recognized. Same as step 2 above, FV - CV + [cost to dispose (only if held for sale)] = Impairment loss *Impairment Test for Finite life, long-lived assets/Fixed assets (PPE) other than Goowill IFRS* *-Reversal allowed except Goodwill* Step 1: Pick higher of NRV (FV - cost to sell) or Value in use (PV of future CF), this is Recoverable amount (RA) Step 2: RA - CV = Impairment loss *PPE (Fixed Assets) Impairment US GAAP* *-Reversal not allowed except FA held for sale* Step 1: Undiscounted CF < CV if yes, impaired Step 2: Impairment Loss Calculation: FV - (CV + Cost of disposal) = Impairment loss

MCQ-11006 The following data pertains to Tyne Co.'s investment in marketable debt securities: Values as of 12/31/Year 2 Amortized cost - PV of expected pmts. (interest + principal) - FV Trading $150,000 $154,000 $154,000 Available for sale $150,000 $142,000 $144,000 What amount should Tyne report as net unrealized loss on available for sale marketable securities at December 31, Year 2, in accumulated other comprehensive income on the balance sheet? A. $0 B. $2,000 C. $6,000 D. $8,000

A. $0 The trading security will not have any credit loss, as there will be a $4,000 unrealized gain (the difference between cost and fair value) recognized on the income statement in Year 1. For the available-for-sale security, the credit loss is equal to the difference between amortized cost and present value of the expected payments, with the amount not to exceed the difference between amortized cost and fair value. Amortized Cost $150,000 - Present Value $142,000 = Difference $8,000 Amortized Cost $150,000 - Fair Value $144,000 = Difference $6,000 (Max loss recognizable in IS) Although the difference between amortized cost and present value is $8,000, the amount that will be reflected as a credit loss on the income statement is capped at $6,000. As far as other comprehensive income, the balance in this account will be zero. There will be no additional loss of $2,000 booked in other comprehensive income because for available-for-sale securities, the company always has the option of selling at fair value.

MCQ-00511 *Lee Corp. reported the following marketable equity security on its December 31, Year 1, balance sheet:* *Neu Corp. common stock, at cost $100,000* *Less: Allowance for decline in market value (20,000)* *Balance $80,000* *At December 31, Year 2, the market value of Lee's investment in the Neu Corp. stock was $85,000. As a result of the Year 2 increase in this stock's market value, Lee's Year 2 income statement should report:* A. An unrealized gain of $5,000. B. A realized gain of $5,000. C. An unrealized loss of $15,000. D. No gain or loss.

A. An unrealized gain of $5,000. Investments in Equity Securities: Equity investments are typically carried at fair value through net income (FVTNI), with *unrealized gains/losses included in earnings as they occur (income statement).* For equity investments that do not have a readily determinable fair value, the practicability exception allows an entity to measure an investment at cost plus/minus observable price changes of identical or similar investments, less impairment.

MCQ-01161 *Which of the following amounts are included in a general fund's encumbrance account?* *I. Outstanding vouchers payable amounts.* *II. Outstanding purchase order amounts.* *III. Excess of the amount of a purchase order over the actual expenditure for that order.* A. II only. B. I only. C. I and III. D. II and III.

A. II only. Choice "1" is correct. The amounts in a general fund's encumbrance account include only outstanding purchase order amounts. Not I. Outstanding vouchers payable amounts represent actual incurred obligations which are included in "vouchers payable" or "accounts payable." Not III. Once an "actual expenditure for that order" has taken place (which means the goods or services have been received), the encumbrance is reversed and the actual expenditure is recorded as vouchers or accounts payable.

MCQ-04833 Under U.S. GAAP, what method of accounting must be used in the preparation of financial statements for an entity that is not considered a going concern? A. Liquidation basis. B. Historical cost basis. C. Realization basis. C. Market basis.

A. Liquidation basis. Under U.S. GAAP, preparation of financial statements presumes that the reporting entity will continue as a going concern (to stay in business). Under this presumption, financial statements are prepared under the going concern basis of accounting. If an entity's liquidation is imminent (and the entity is therefore no longer considered to be a going concern), financial statements are prepared under the liquidation basis of accounting.

MCQ-12688 Mobe Co. reported the following operating income (loss) for its first three years of operations: 2019 $ 300,000 2020 (700,000) 2021 1,200,000 For each year, there were no deferred income taxes (before 2019), and Mobe's effective income tax rate was 21 percent. Mobe elected to forgo the carryback of the 2020 net operating loss. In its 2021 income statement, what amount should Mobe report as total income tax expense? A. $168,000 B. $252,000 C. $105,000 D. $84,000

B. $252,000 Choice "2" is correct, $252,000 total income tax expense for 2021. 2020 Dr Deferred tax asset ($700,000 × 21%) 147,000 Cr Income tax benefit 147,000 2021 Dr Income tax expense ($1,200,000 × 21%) 252,000 Cr Income tax payable 105,000 [($1,200,000 - $700,000) × 21%] Cr Deferred tax asset 147,000 Note: NOLs utilized in the five-year carryback period or in 2018, 2019, or 2020 tax years are not subject to a taxable income limitation. NOLs carried forward to taxable years beginning in 2021 or later are limited to 80 percent of taxable income before the NOL deduction. In this example, 80 percent of $1,200,000 equals $960,000. The $700,000 NOL does not exceed the 80 percent limitation for 2021.

MCQ-09416 Clothes Horse Corp. ("CHC") issued $500,000 bonds due in 10 years in January 1, Year 1 at a premium for $567,105. On January 1, Year 6, when the carrying value of the bond was $539,940, CHC redeemed the bonds at 102. What amount of gain should CHC record related to the redemption? A. $10,000 B. $29,940 C. $39,940 D. $0

B. $29,940 The gain or loss is equal to the difference between the net carrying value of the bond and the redemption price, calculated as follows: Carrying value 539,940 Less: Redemption price ( 510,000) [$500,000 × 1.02] = 29,940 Gain CHC's Issuance of BP with premium JE: Dr Cash $567,105 Cr Bonds payable $500,000 Cr Premium; on B/P less bond issuance costs $67,105 CHC's redemption would be recorded as follows: Dr Bond payable 500,000 Dr Premium 39,940 Cr Cash 510,000 Cr Gain on extinguishment 29,940 Choice "1" is incorrect. This is the difference between the cash paid to redeem the bonds and the face amount of the bonds, which is not equal to the gain on the redemption. See the explanation above. Choice "3" is incorrect. This is the unamortized premium on the bond, not the gain from the redemption of the bond. Choice "4" is incorrect. $0 is incorrect; there is a gain on the redemption because the cash paid of $510,000 is less than the carrying amount of the bond of $539,940.

Segment Reporting

Helps the users better understand entity's performance and risk, if company is public must report, 1. Operating 2. Products/services 3. Geographic areas of business 4. Major customers (concentration risk)

MCQ-09670 Which of the following cost must be expensed when incurred? A. The cost of purchasing a trademark. B. The fee to obtain a franchise. C. The cost of goodwill in an acquisition method business combination. D. Research and developing a patented product.

D. Research and developing a patented product. Research costs related to internally developed intangible assets must be expensed in the period incurred. Choice "1" is incorrect. The cost of purchasing a trademark is capitalized as an intangible asset. Choice "2" is incorrect. The fee paid to obtain a franchise (the initial franchise fee) is capitalized as an intangible asset. The continuing franchise fee is expensed when incurred. Choice "3" is incorrect. Goodwill acquired in an acquisition method business combination is capitalized as an intangible asset. However, costs to develop, maintain, or restore goodwill are expensed.

*Summary of Research and Development (R&D)*

U.S. & IFRS Similarities 1. Research expensed, 2. production (inventory) capitalized bc part of cost of goods available for sale, 3. successful legal defense capitalized Development (after feasibility): U.S. GAAP-> capitalized for software only IFRS-> capitalized

*Lower-of-cost-or-market method (GAAP) vs lower-of-cost-and-net-realizable-value (IFRS)*

*GAAP uses the lower-of-cost-or-market method* Step one: Replacement cost = 20 NRV (SP - CC) = 28 NRV - GP = 21 Use 21 bc it's middle number Step two: Chose the lower of cost 26 or Mkt 21 21 is lower *IFRS uses the lower-of-cost-and-net-realizable-value* One step only: Chose the lower of cost 26 or NRV (SP - CC) 28 26 is lower

Equity Method Accounting

20%-50% Ownership = Equity method Purchase 20% of Company (Sub) JE: Dr Equity investment Dr Cash Income 20% reported by Sub JE: Dr Equity investment Cr Equity income 20% Depreciation amortization of excess FV over CV JE: Dr Equity income Cr Equity investment Cash dividend 20% paid by Sub JE: Dr Cash Cr Equity investment Parent's initial investment + parent % of investee NI - parent % of investee dividends - parent % of depreciation = Investment in investee (Parent balance sheet) Investee's NI x parent's % ownership - parent % of depreciation = parent's net equity earnings in investee (Parent income statements)

MCQ-09438 Backdoor Inc. had 200,000 shares of $5 par common stock outstanding. The company declared a stock dividend of 100,000 shares when the market price was $25. By how much did additional paid-in capital increase when the dividend was distributed to the shareholders? A. $0 B. $500,000 C. $1,000,000 D. $2,500,000

A. $0 This stock dividend is an example of a large stock dividend, as 100,000 shares is 50% of the 200,000 shares outstanding. A large stock dividend (>20-25% of outstanding stock) is recorded at the par value of the stock because it is assumed that the size of the dividend will affect the market price of the stock. The journal entry to record the dividend is: Debit (Dr)Credit (Cr)Retained earnings 500,000 Common stock (100,000 x $5 par) 500,000 Because the dividend is recorded at par value, no additional paid-in capital is recorded. Choice "2" is incorrect. This is the total value of the stock dividend, which is recorded at par value, as explained above. Choice "3" is incorrect. This is the total par value of the common stock before the stock dividend.

MCQ-09293 The following data pertain to Tyne Co.'s investments in marketable debt securities: Market Value Cost -- 12/31/Y2 -- 12/31/Y1 Trading $150,000 $155,000 $100,000 Available-for-sale 150,000 130,000 120,000 There are no expected credit losses. What amount should Tyne report as *unrealized gain (loss)* in its Year 2 *income statement?* A. $55,000 B. $50,000 C. $60,000 C. $65,000

A. $55,000 Trading securities Unrealized Gain/Loss go to Income statement. Available-for-sale securities unrealized Gain/Loss go to other comprehensive income PUFIER. 155,000 Current Yr FV - 100,000 Prior Yr FV = 55,000 on IS 150,000 Cost - 130,000 Current Yr FV = 20,000 on OCI Rule: Unrealized gains and losses are reported as follows: trading debt securities are reported at fair value with unrealized gains and losses included in earnings (along with "realized" gains and losses, if any). Where there are no expected credit losses, available-for-sale debt securities are reported at fair value with unrealized gains and losses reported as a separate component of other comprehensive income until realized.

MCQ-09672 On December 31, an entity analyzed a finite life trademark with a net carrying value of $750,000 for impairment. The entity determined the following: Fair value $700,000 Undiscounted future cash flows $740,000 What is the impairment loss that will be reported on the December 31 income statement under U.S. GAAP? A. $0 B. $10,000 C. $40,000 C. $50,000

C. $50,000 *-No reversal* Step 1: Undiscounted CF < CV if yes, impaired $740,000 Undiscounted future CF < $750,000 CV Step 2: Impairment Loss Calculation: FV - CV = Impairment loss $700,000 FV - $750,000 CV = 50,000 Impairment loss

MCQ-01153 Which event(s) should be included in a statement of cash flows for a governmental entity? I. II. Cash inflow from issuing bonds to finance city hall construction.Cash outflow from a city utility representing payments in lieu of property taxes. A. Neither I nor II. B. Both I and II. C. II only. D. I only.

C. II only. Choice "3" is correct. The cash inflow from issuing bonds to finance city hall construction is a cash inflow for a governmental fund. Governmental funds do not prepare a statement of cash flows since the measurement focus is financial resources. The cash outflow from a city utility representing payments in lieu of property taxes is a cash outflow from operating activities for an enterprise fund. Enterprise funds prepare a statement of cash flows.

MCQ-09325 *On September 1, Year 1, Royal Corp., a newly formed company, had the following stock issued and outstanding:* *- Common stock, no par, $1 stated value, 5,000 shares originally issued for $15 per share.* *- Preferred stock, $10 par value, 1,500 shares originally issued for $25 per share.* *Royal's September 1, Year 1, statement of stockholders' equity should report:* Common Stock -- Preferred Stock -- Additional Paid-In Capital A. $5,000 $15,000 $92,500 B. 5,000 37,500 70,000 C. 75,000 37,500 0 D. 75,000 15,000 22,500

A. $5,000 $15,000 $92,500 CS (5,000 shares × $1 stated value) 5,000 PS (1,500 shares × $10 par value) 15,000 Additional paid-in capital from: CS ($15 − $1) × 5,000 shares = 70,000 APIC-CS PS ($25 − $10) × 1,500 shares = 22,500 APIC-TS 70,000 + 22,500 = 92,500 Total APIC $5,000, common stock; $15,000 preferred stock; $92,500 APIC.

MCQ-00309 *When the equity method is used to account for investments in common stock, which of the following affect(s) the investor's reported investment income?* *I. A change in market value of investee's common stock:* *II. Cash dividends from investee:* I. -- II. A. No No B. Yes Yes C. Yes No D. No Yes

A. No No Rule: Investor records as revenue its "share of the investee's earnings" (not "dividends received") under the equity method. Dividends from an investee company are recorded by the investor as a reduction in the carrying amount of the investment on the balance sheet of the investor. Dr Cash Cr Equity investment/Investment in investee Changes in the market value of investee's common stock are not considered income to the parent under the equity method. Under the fair value method, receipt of a dividend is recorded as income and does not affect the investment account. *Fair value method (less than 20% ownership) JE for receipt of dividends* Dr Cash Cr Equity income

MCQ-09364 Two independent situations are described below. Each situation has future deductible amounts and/or future taxable amounts produced by temporary differences: Situation 1 -- 2 Taxable income $40,000 $80,000 Amounts at year-end: Future deductible amounts 5,000 10,000 Future taxable amounts 0 5,000 Balances at beginning of year: Deferred tax asset 1,000 4,000 Deferred tax liability 0 1,000 The enacted tax rate is 21% for both situations. Determine the deferred tax asset balance at year-end. Situation 1 Situation 2 A. $5,000 $10,000 B. $2,150 $6,100 C. $1,000 $3,000 D. $1,050 $2,100

B. $8,350 $18,750 This questions is asking for the end of year deferred tax asset balance only, disregard all other information. Situation 1 EOY Future deductible amounts: 5,000 = 5,000 x 0.21 = 1,050 EOY DTA Situation 2 EOY Future deductible amounts: 10,000 = 10,000 x 0.21 = 2,100 EOY DTA

MCQ-09320 Which of the following statements regarding lease accounting is/are correct? I. A finance lease is a form of off-balance sheet financing which transfers substantially all of the benefits and risks of ownership of property to the lessee. II. It is possible for a lessee to account for a lease as a finance lease while the lessor accounts for the lease as an operating lease. III. When calculating the present value of the minimum lease payments, the implicit rate (if known) should be used. A. II only. B. III only. C. I and III. D. I, II, and III.

B. III only. Statement III is the only correct statement. Statement I is incorrect because a finance lease is not a form of off-balance sheet financing as the lessee is required to report both an asset and a liability on its books when finance lease accounting is used. Statement II is incorrect because if a lessee accounted for a lease as a finance lease, that would mean that at least one of the OWNES criteria was met and therefore the lessor could not account for the lease as an operating lease. Choices "1", "3", and "4" are incorrect, based on the above discussion

MCQ-09673 On December 31, an entity analyzed a finite life trademark with a net carrying value of $750,000 for impairment. The entity determined the following: Fair value (less cost to sell) $700,000 Present value of future cash flows $710,000 What is the impairment loss that will be reported on the December 31 income statement under IFRS? A. $0 B. $10,000 C. $40,000 C. $50,000

C. $40,000 *Impairment Test for long-lived assets/Fixed assets (PPE) other than Goowill IFRS* Step 1: Pick higher of NRV (FV - cost to sell) or Value in use (PV of future CF), this is Recoverable amount (RA) Pick higher of $700,000 NRV or $710,000 PV of future CF, $710,000 is Recoverable amount (RA) Step 2: RA - CV = Impairment loss $710,000 RA - $750,000 CV = 40,000 Impairment loss

MCQ-00281 When the market value of an investment in debt securities in which the company has a positive intent and ability to hold to maturity exceeds its carrying amount, how should each of the following assets be reported at the end of the year? I. Long-term marketable debt securities: II. Short-term marketable debt securities: I. -- II. A. Carrying amount -- Market value B. Market value -- Carrying amount C. Market value -- Market value D. Carrying amount -- Carrying amount

Choice "4" is correct, carrying amount, carrying amount. Marketable debt securities that the company has the intent and ability to hold to maturity, both "long" and "short" term, are reported at carrying amount (amortized cost) unless there is a permanent decline in market value.

MCQ-01163 *Tott City's serial bonds are serviced through a debt service fund with cash provided by the general fund. In a debt service fund's statements, how are cash receipts and cash payments reported?* Cash receipts -- Cash payments A. Revenues Expenditures B. Revenues Transfers C. Transfers Transfers D. Transfers Expenditures

Choice "4" is correct. When serial bonds are serviced through a debt service fund with cash provided by the general fund, the debt service fund should report cash receipts as "Transfers" (other financing sources) and cash payments as "operating expenditures." Journal entry to record interfund transfer from the general fund: Dr Cash $XXX Cr Interfund transfers (from the general fund) $XXX Journal entry to record payment of interest: Dr Expenditures $XXX Cr Cash $XXX

MCQ-05621 On October 1, Year 1, Pepper Inc. acquired 100% of Salt Inc. for $275,000. On that date, the carrying values of Salt Inc.'s assets and liabilities were $450,000 and $200,000, respectively. The fair values of Salt's assets and liabilities were $550,000 and $200,000, respectively. Additionally, Salt had identifiable intangible assets at the time of acquisition with a fair value of $60,000. What is the gain to be reported on Pepper's December 31, Year 1 consolidated income statement? A. $25,000 B. $135,000 C. $0 D. $75,000

B. $135,000

MCQ-09318 On January 2, year 1, Kean Co. purchased a 30% interest in Pod Co. for $250,000. On this date, Pod's stockholders' equity was $500,000. The carrying amounts of Pod's identifiable net assets approximated their fair values, except for land whose fair value exceeded its carrying amount by $200,000. Pod reported net income of $100,000 for year 1, and paid no dividends. Kean accounts for this investment using the equity method. In its December 31, year 1 balance sheet, what amount should Kean report as investment in subsidiary? A. $210,000 B. $220,000 C. $270,000 D. $280,000

D. $280,000 JE to record 30% purchase of Kean Co.: Dr Equity investment 250,000 Cr Equity income 250,000 JE to record net income from Kean Co.: Dr Equity investment 30,000 (100,000 x .30) Cr Equity income 30,000

MCQ-00870 Abel and Carr formed a partnership and agreed to divide initial capital equally, even though Abel contributed $100,000 and Carr contributed $84,000 in identifiable assets. Under the bonus approach to adjust the capital accounts, Carr's unidentifiable asset should be debited for: A. $0 B. $46,000 C. $16,000 D. $8,000

D. $8,000 Under the bonus method, unidentifiable assets (i.e., goodwill) are not recognized. The total resulting capital is the FV of the tangible investments of the partners. Thus, there would be no unidentifiable assets recognized by the creation of this new partnership. Total Assets 50% Abel 50% Carr Initial contributions: Abel - cash $100,000 $100,000 0 Carr - identifiable assets 84,000 0 84,000 Subtotal 184,000 100,000 84,000 Bonus from Abel to Carr 0 (8,000) 8,000 Total $ 184,000 92,000 $92,000 Note: The examiners were trying to be tricky - it is important to reread the question to be sure you answered it.

*Non-for-profit Expense Classification and Display*

Expenses are defined as 1. program (operating or mission expense) or 2. support services (other non-operating expenses). Program expenses relate to the *mission* of the organization while support services relate to the *organization's administrative, membership development and fundraising expenses.* The total amount of each functional expense (the expenses by individual program or support service) is displayed on the face of the statement of activities or disclosed in the notes to the financial statement. Support services typically involve items such as, - fundraising (i,e., salaries for fundraisers) - administration, - management, - and membership development. *All expenses of NFP's are reported on the statement of activities are reported on the without donor restriction column.*

Property, plant, and equipment (PP&E)

The cost of a fixed asset (or any other asset) is the cost to acquire the asset and place it in condition for its intended use. *As an example, purchased equipment would include purchase price, freight in, installation, saletaxes, etc.* - Ordinary repairs are expensed, not capitalized. - *Extraordinary repairs should be capitalized if they increase the usefulness of the asset* and should be recorded by decreasing accumulated depreciation if they increase the life of the asset. - *Under IFRS and GAAP, the freight cost should be capitalized as part of the machinery's historical cost. Just like GAAP, the interest incurred to finance the purchase is expensed and not capitalized as part of the historical cost.* - Land is not a depreciable asset; land improvements are depreciated (fences, sprinkler, etc). - Sometimes, fixed assets are acquired in a "basket" purchase. The amount paid must be allocated to the various assets acquired, generally on a relative fair value or appraisal value basis. *Land = Includes all cost before excavation* *Building = Includes the cost of excavation forward*

Form 8-K

This form is filed to report major corporate events such as 1) corporate asset acquisitions or disposals, 2) changes in securities and trading markets, 3) changes to accountants or financial statements, and 4) changes in corporate governance or management.

MCQ-09326 Lore Co. changed from the cash basis of accounting to the accrual basis of accounting during the current year. The cumulative effect of this change should be reported in Lore's current year financial statements as a: A. Prior period adjustment resulting from the correction of an error. B. Prior period adjustment resulting from the change in accounting principle. C. Component of income from continuing operations. D. Component of nonoperataing gains.

A. Prior period adjustment resulting from the correction of an error.

MCQ-09311 The proper accounting treatment to account for a change in inventory valuation from FIFO to LIFO under U.S. GAAP is: A. Prospective Application. B. Retrospective Application. C. Retroactive Application. D. Ignored.

A. Prospective Application. A change in accounting principle, if considered "impractical to accurately calculate the cumulative effect adjustment," is handled prospectively. A change from FIFO to LIFO (U.S. GAAP) would require the establishment and recalculation of old inventory layers, which are considered impractical to rebuild. Hence, the beginning inventory of the year of change is the first LIFO layer.

*Operating Activities*

All current assets except cash and cash equivalents (mature in 90 days or less). Does include trading securities, exclude AFS, and HTM (both are investing). *Includes Current assets and current liabilities.* Operating activities examples: -Charitable contributions made by the (NFP). -*Receipts of unrestricted resources designated by the governing body to be used for long-lived assets. (NFP)* -Receipts or payments for the settlement of lawsuits -*Cash payments for interest, Interest received, dividends received.* -Purchase of trading securities -Increase in income taxes payable -Equity earnings of unconsolidated affiliate (are subtracted) NI (accrual) -> Cash NI Rev > Exp = Inflow of cash Rev < Exp = Outflow of cash Indirect Method (Always starts with NI) Step 1: NI Step 2: Analyze CA ↑CA = ↓CF ↓CA = ↑CF Analyze CL ↑CA = ↑CF ↓CA = ↓CF Step 3: Add non-cash expenses such as depreciation, amortization and bad debt Step 4: Add losses Deduct Gains

MCQ-09744 Which of the following choices is least likely to represent an actual debt covenant? A. Collateral cannot fall below a specific amount. B. The debt/equity ratio must stay below a specific level. C. Working capital levels cannot fall below specific amount. D. The interest coverage ration must stay below a specific level.

D. The interest coverage ration must stay below a specific level. Choice "4" is correct. The higher the interest coverage ratio (EBIT/interest expense), the better it is for the company paying interest on the debt. A more appropriate debt covenant would be for the interest coverage ratio to stay above (rather than below) a specific level. Choice "1" is incorrect. A minimum level for collateral is an appropriate debt covenant. Choice "2" is incorrect. The lower the debt/equity ratio is for a company, the better (from a risk management and debt holder perspective). So a maximum threshold for the debt/equity ratio is an appropriate debt covenant. Choice "3" is incorrect. A minimum level for working capital is an appropriate debt covenant.

*Overstatement and understatement of inventory errors*

End Inv. overstated: Beg Inv. + Purchases = CGAS - End Inv. ↑ = COGS ↓ Sales -COGS ↓ = Profit ↑ Beg Inv. overstated: Beg Inv. ↑ + Purchases = CGAS ↑ - End Inv. = COGS ↑ Sales -COGS ↑ = Profit ↓

Goods In Transit

FOB shipping point: Belong to the seller *until goods are in the shipping truck.* FOB destination: Belongs to the seller until goods are received by the buyer.

Financial Instruments

Financial Instruments Are: - Cash, foreign currency, and demand deposits. Ownership interest in an entity (stock, partnership, LLC). - Contracts that both (i.e., bonds): Impose on one entity a contractual obligation or duty. Convey to the second entity a contractual right to do the opposite. - Derivatives. (i.e., Forward, futures, options, swaps) Disclosure of Financial Instruments *Fair value must be disclosed for all financial instruments* for which it is practicable to estimate that value together with the *related carrying amounts.* Disclosure of concentrations of credit risk is required. Credit risk is the possibility of loss from the failure of another party to perform according to the terms of a contract. Disclosure of market risk is encouraged but not required under U.S. GAAP. Under *IFRS, disclosure of market risk (risk of losing money in the stock market) is required.*

Comprehensive Annual Financial Report

Governments often present a Comprehensive Annual Financial Report (CAFR). A CAFR is not required by GASB 34. It is a GFOA designated presentation that adds introductory and statistical sections to the GASB 34 presentation, as follows: 1. Introductory Section (unaudited) Letter of transmittal Organizational chart List of principal officers 2. Basic Financial Statements and Required Supplementary Information (audited) MD&A Government-wide financial statements *(Basic financial statements)* Fund financial statements *(Basic financial statements)* Notes to financial statements *(Basic financial statements)* Required supplementary information 3. Statistical Section (unaudited) *Ten years of selected data* *Ten years of economic data* Other data

Exceptions to the General Rule *(Treat Like Change In Estimate - Prospective Application)*

Impractical to Estimate If it is considered impractical to accurately calculate this cumulative-effect adjustment, then the change is handled prospectively (like a change in estimate). *An example of a change handled in this manner is a change in inventory cost flow assumption to LIFO (U.S. GAAP only).* Since a cumulative-effect adjustment to LIFO would require the reestablishment and recalculation of old inventory layers, it is considered impractical to try and rebuild those old cost layers. Change in Depreciation Method *A change in the method of depreciation, amortization, or depletion is considered to be both a change in accounting principle and a change in estimate.* These changes should be accounted for as changes in estimate and are handled prospectively. The new depreciation method should be used as of the beginning of the year of change in estimate and should start with the current book value of the underlying asset. No adjustment should be made to retained earnings.

Income from continuing operations (Net Income net of tax if no Disc. OP)

Income from continuing operations includes operating activities (i.e., revenues, COGS, selling expenses, and administrative expenses), nonoperating activities (e.g., other revenues and gains and other expenses and losses), and income taxes. *Normal operating + Non-operating + Unusual and infrequent = Income from continuing operations*

Multi-Step Income Statement

Income statement format that contains subtotals to highlight significant relationships. In addition to net income, it reports gross profit and operating income.

Non-For-Profit Accounting for Marketable Securities

Investments in securities are displayed at *their fair values* and increases and decreases in the fair value of securities are classified as *without donor restrictions in the statement of activities unless there are donor-stipulated restrictions.* *Investment income (dividends and interest)* are reported in the period earned in the net asset category as either with or *without donor restrictions.* Investment returns are reported net of any related investment expense. Endowments Not-for-profit organizations *account for assets with donor restrictions that are perpetual in nature* in endowment funds (NFP cannot use the principal/corpus but may be able to use the income earned depending on donor restrictions). Generally, a benefactor will donate an amount and direct that earnings and not principal be used for specific or general purposes of the not-for-profit. An *underwater endowment is a donor-restricted endowment fund* for which the fair value at the reporting date is less than either the original gift amount or the amount required to be maintained by the donor or by law. Example: *Donation of stock with a valued at 500,000 and the fair value falls to 400,000 resulting in a 100,000 loss. The endowment must be reported as with donor restrictions.* Underwater endowment funds will report accumulated losses together with the endowment fund in net assets with donor restrictions.

*Operating segments*

It engages in business activities, generates revenue. For which discrete financial information is available (Traceable CF). *Pension plan or Headquarters are not reportable*. *Materiality test - Must satisfy 1 of 3* An identified operating segment is reportable if it has at least 10 percent of all operating segments: -Total Internal and external sales combined -Total profit or loss -Total assets. Additional segments must be disclosed disclosed despite their failure to satisfy one of the thresholds until 75% of external segmented.

*Lease Accounting, Governmental, Proprietary, & Fiduciary Funds*

Lessor (Seller) Short-term leases: GRSPP Funds Dr Receivable Cr Revenue SE-CIPPOE Funds Dr Receivable Cr Revenue Contracts that transfer ownership: GRSPP Funds Dr Receivable Cr Current revenue & deferred inflow SE-CIPPOE Funds Dr Receivable Cr Revenue Leases other than short-term leases and contracts that transfer ownership: GRSPP Funds Dr Receivable CrCurrent revenue & deferred inflow SE-CIPPOE Funds Dr Receivable *Cr Deferred inflow* Deferred inflows are used in connection with lessor activity recorded in governmental funds because revenues are measurable but not available. Deferred inflows are used by lessors in proprietary and fiduciary funds in response to the requirements of the standard. Lessee (Buyer) Short-term leases: GRSPP Funds Dr Lease expense Cr Cash/payable SE-CIPPOE Funds Dr Lease expense Cr Cash/payable Contracts that transfer ownership: GRSPP Funds Dr Capital outlay expenditure Cr Other financing sources SE-CIPPOE Funds Dr Capital asset expenditure Cr Lease liability Leases other than short-term leases and contracts that transfer ownership: GRSPP Funds Dr Capital outlay expenditure Cr Other financing sources SE-CIPPOE Funds *Dr Right-of-use asset* Cr Lease liability Right-of-use assets are only recorded for proprietary and fiduciary funds lessees for leases classified as other than short term and contracts that transfer ownership.

*Intangible Assets*

Patents, copyrights, franchises, trademarks, and goodwill are common intangible assets. - *Purchased intangibles are recorded at cost.* - *Under U.S. GAAP, internally developed intangibles are expensed* when incurred because research and development costs cannot be capitalized. - Under IFRS, research costs related to internally developed intangibles must be expensed, but *development costs can be capitalized if certain criteria are met.* - Costs of developing, maintaining, or restoring intangible assets that are not specifically identifiable, or have indeterminate lives, such as goodwill, are expensed when incurred.

Permanent and Temporary Differences (DTA, DTL)

Permanent Differences = No deferred taxes *Permanent differences do not affect the deferred tax computation.* They only affect the current tax computation. *These differences affect only the period in which they occur.* They do not affect future financial or taxable income. - Enter into pretax GAAP financial income, but never enter into taxable income (e.g., interest income on state or municipal obligations, life insurances, proceeds/expense). - Enter into taxable income, but never enter into pretax GAAP financial income (e.g., dividends received deduction). Deferred Tax Liability = *Pay taxes later* A deferred tax liability is a future payable. Current financial income is greater than current taxable income *(BI > TI)*. Expense First - Depreciation expense greater for tax than for book -Amortization of goodwill for tax Revenue Later - Prepaid expenses (cash basis for tax) - Installment sales (used for tax purposes) - Contractor accounting Deferred Tax Assets = *Gift certificate* A deferred tax asset is a future receivable. Current financial income is less than current taxable income *(TI > BI)*. Is considered the more likely than not test. You need a valuation allowance for any amount you're not expected to use. Revenue First = *Prepaid expense, which are taxable now and deductible later.* - Unearned rent (taxable income before book income) - Unearned interest (taxable income before book income) Expense Later = Estimated tax deductible later. - Bad debt expense (allowance for GAAP and direct write-off for tax) - Estimated liability/warranty expense (allowance for GAAP and direct write-off for tax)

Perpetual and Periodic Concepts

Perpetual Inventory System: a running total of the inventory is maintained as goods are purchased and sold and the cost of goods sold is updated as sales occur. Periodic Inventory System: a running total is not maintained, and the cost of goods sold cannot be determined until the end of the period when the ending inventory is counted. Beg Inv. + Purchases = CGAS - End Inv. = COGS *FOB Shipping Point: Buyer's inventory once goods "in truck" - freight in - added to COGS* *FOB Shipping Destination: Seller's inventory until goods are received by buyer - freight out - selling expense*

Postretirement Benefits Other Than Pensions

Post retirement benefits include: - Health care insurance - Life insurance - Welfare benefits - Tuition assistance 2.2 Financial Statement Reporting Postretirement benefits must be reported on the balance sheet (funded status and OCI components), income statement *(SIRAGE)*, and footnotes in the same manner as pensions if: - the obligation is attributable to employees' services already rendered; - the employees' rights accumulate or vest; - payment is probable; and - the amount of benefits can be reasonably estimated. *EPBO = APBO + Any nonvested benefits* Postretirement benefits must be accrued during the period the employee works, called the "attribution period" (date hired to date fully vested). The calculation of the funded status of a postretirement benefit plan is done using the APBO (accumulated postretirement benefit obligation), which is the present value of future benefits that have vested as of the measurement date: Fair Value of Plan Assets < APBO > = Funded Status

Pension Obligations

Projected Benefit Obligation - US GAAP The projected benefit obligation (PBO) is the actuarial present value of all benefits attributed by the plan's benefit formula. The PBO is used in the calculation of funded status, service cost, and interest cost and is computed using future salary levels. Accumulated Benefit Obligation The accumulated benefit obligation (ABO) is the actuarial present value of benefits attributed by a formula using current and past salary levels. Defined Benefit Obligation - IFRS Under IFRS, the pension liability is called the defined benefit obligation (DBO). The DBO is very similar to the U.S. GAAP PBO. Pension Plan Funded Status Pension plans are accounted for on the accrual basis. *Defined benefit pension plans are reported on the balance sheet based on funded status:* *Fair Value of Plan Assets* *< PBO or DBO >* *= Funded Status* U.S. GAAP Under U.S. GAAP, companies are required to aggregate all overfunded *(FV of plan assets > PBO)* pension plans and report them as a *non-current asset* on the balance sheet. All underfunded *(FV of plan assets < PBO)* pension plans should also be aggregated and reported as a *current liability, a non-current liability, or both*. A pension plan is reported as a current liability to the extent that the benefits payable in the next 12 months exceed the fair value of the plan's assets. IFRS Under IFRS, the funded status (DBO - fair value of plan assets) of the pension plan is reported on the balance sheet as the net defined benefit liability (asset). *A liability is reported if the plan is underfunded (DBO > fair value of plan assets)* and *an asset is reported if the plan is overfunded (DBO < fair value of plan assets).* IFRS do not specify whether an entity should classify the net defined benefit liability (asset) as current or non-current.

Subsequent Events

Recognized Subsequent Events Subsequent events occur after the balance sheet date but before the financial statements are issued. *Journal entry if condition existed at balance sheet date and new information presented.* Subsequent events that provide additional information about conditions that existed at the balance sheet date must be recognized on the financial statements. An example of this type of event is litigation settled after the balance sheet date where the litigation arose before the balance sheet date. The settlement amount is considered in determination of the liability. *Nonrecognized Subsequent Events = No JE but disclosed if material to financial statements* Subsequent events that provide information about conditions that occurred after the balance sheet date and did not exist at the balance sheet dateare not recognized in the financial statements, but may be disclosed if necessary to keep the statements from being misleading.

Compensation and Benefits

Reporting Changes in Funded Status in OCI: *U.S. GAAP = SIRAGE -> Start in OCI net of tax -> Amortized as come out of OCI (income smoothing)* U.S. GAAP Under U.S. GAAP, a change in the funded status of a pension plan due to pension net losses or gains or prior service cost is reported in the period incurred as a component of accumulated other comprehensive income, net of tax. Any unrecognized net transition obligation (or asset) is also reported in accumulated other comprehensive income. IFRS *Under IFRS, prior (past) service cost is reported as a component of service cost on the income statement in the period incurred.* Pension gains and losses are reported in other comprehensive income in the period incurred and are not reclassified (amortized) to the income statement. To report net loss or prior service cost: Dr Other comprehensive income Cr Pension benefit asset/liability Dr Deferred tax asset Cr Deferred tax benefit—OCI Pension Reclassification Adjustments: Dr Pension benefit asset/liability Cr Other comprehensive income Dr Deferred tax expense—OCI Cr Deferred tax liability *Pension net gains or losses, prior service cost, and net transition assets or obligations remain in accumulated other comprehensive income until recognized in net periodic pension cost through amortization.* *= SIR"AGE" -> OCI -> % age of "AGE" each year is expensed* Reclassification adjustment to record *amortization of net loss*, prior service cost, or net transition obligation to net periodic pension cost: Dr Net periodic pension cost Cr Other comprehensive income (% AGE loss taken out of OCI with Cr above) Dr Deferred tax benefit—OCI Cr Deferred tax benefit—income statement Reclassification adjustment to record amortization of net gain or net transition asset to net periodic pension cost: Dr Other comprehensive income (% AGE gain taken out of OCI with Dr above) Cr Net periodic pension cost Dr Deferred tax expense—income statement Cr Deferred tax expense—OCI

Reporting Units

Reporting units (the primary government and its component units) are the governmental version of "consolidations." When to consolidate and how to consolidate is important under the GASB 34 model. Primary Governments and Component Units A government is viewed as a stand-alone or primary government if it has a separately elected governing board, it is a legal entity and it is financially independent (SELF). *SE: separate elected governing board* *L: legal entity* *F: financially independent* *If government meets all four criteria it is a Primary Government.* A government that cannot stand by itself is a component unit of another government and should present its financial statements with the primary government. *Discrete vs. Blended Presentation* Component units are presented either discretely or in a blended format. Generally, component units are presented as discrete *separate columns* on the primary government's financial statements. Blended presentations are made when the component unit either 1. *exclusively serves the primary government* or 2. *when the component unit's governing body (board) is substantially the same as the primary government's governing body.* Blending involves consolidation of activities.

*Accounting for Stock Options*

Stock Options Compensatory stock options should be valued at the fair value of the options issued. The compensation expense is allocated over the service period. Example Accounting for Stock Options On January 1, Year 1, Green Co. granted options exercisable after December 31, Year 2, to purchase 50,000 shares of $1 par common stock for $8 per share. Using an acceptable valuation model, the options had a total fair value of $50,000. The options are to serve as compensation for services during Year 1 and Year 2. Journal Entry: January 1, Year 1—No entry required Journal Entry: To allocate compensation cost to Year 1 operations DR Compensation expense $25,000 CR APIC—stock options $25,000 Journal Entry: To allocate compensation cost to Year 2 operations DR Compensation expense 25,000 CR APIC—stock options 25,000 On January 1, Year 3, all options are exercised. Journal Entry: To record the exercise of the options Cash (50,000 × $8) 400,000 APIC—stock options 50,000 Common stock (50,000 × $1 par) 50,000 APIC in excess of par (common stock) 400,000

MCQ-09284 *The City of Lawrence has a $20,000,000 bond issue outstanding with a stated rate of 6 percent issued at 0.855 to yield 7 percent. Interest is payable on April 1 and October 1. The city also had a master lease agreement with a balance of $200,000 yielding 6 percent that qualified for treatment as lease agreement structured as a contract that transfers ownership to buy equipment used in general governmental operations. Monthly interest had not been paid year-end. The bond indenture required the use of formal debt service fund accounting. Lease payments were made through the general fund. At December 31, the governmental funds of the City of Lawrence would display accrued interest payable of:* General Fund -- Debt Service Fund A. $1,000 $300,000 B $1,000 $350,000 C. $1,000 $0 D. $10 $0

The City of Lawrence would not accrue interest payable on the debt related to governmental funds. Accrued interest would not be recorded relative to either the debt supported by formal debt service funds or the general fund. *General Rule:* Governmental funds should generally accrue fund liabilities and expenditures in the period in which the liability is incurred. Unmatured long-term indebtedness should be reported as long-term debt in government-wide financial statements rather than as governmental fund liabilities. Thus, no accrual of unmatured interest or principal is made in the fund financial statements. The application of this principle applies not only to formal debt issues that require debt service funds, such as bond issues, but also to other long-term debt, such as obligations incurred under lease agreements structured as contracts that transfer ownership, etc.

*Statement of Cash Flows (Proprietary Fund)*

The Statement of Cash Flows is only used for the proprietary funds. The *direct method is required* and it is prepared in a manner similar to the commercial version, with the following differences: - There are four categories (instead of the three) and the order of the categories is the following: 1. Operating activities. *2. Noncapital financing activities.* *3. Capital and related financing activities.* 4. Investing activities Interest income/cash receipts are reported as *"investing activities"* (not as operating activities). Interest expense/cash payments are either: capital and related financing; or noncapital financing. - Capital asset purchases are reported as *"financing activities"* (not as investing activities). - Interest expense/cash disbursements are reported as *"financing activities"* (not as operating activities). - Many transactions unique to government (e.g., transfers, property tax revenues, and special assessments) are often classified as *"non-capital financing activities."* - A reconciliation of operating income (instead of net income) to net cash provided by operations is required.

MCQ-09306 On December 30, year 1, Wayne Corporation issued 1,000 of its 8%, 10-year, $1,000 face value bonds with detachable stock purchase warrants at par. Each bond carried a detachable warrant for one share of Wayne's common stock at a specified option price of $25 per share. Immediately after issuance, the market value of the warrants was $120 per warrant.In its December 31, year 1, balance sheet, what amount should Wayne report as the book value of bonds payable? A. $1,080,000 B. $1,000,000 C. $900,000 D. $1,200,000

The bonds were originally issued with detachable warrants. Because the warrants were detachable, they represent a separate security from the bonds, and a value must be attributed to them. So the total issuance price is going to have to be allocated between the bonds and the warrants. The bonds were issued at par (no premium or discount, at least initially). Par was $1,000,000 (1,000 bonds × $1,000). The rest of the information about the life of the bonds and the stated interest rate is nice but irrelevant. In this question, both the market value of the bonds ($1,080,000) and the market value of the warrants ($120,000) were provided. The total market value is $1,200,000 of which the bonds are 90% and the warrants are 10%. Thus, 90% of the $1,000,000 or $900,000 total proceeds is allocated to the bonds. The $900,000 is reported as bonds payable (net of the discount). Note that they could just as easily have asked for the amount allocated to the warrants ($100,000) or the amount of the discount on the bonds ($100,000). Or, the bonds could have been issued at a premium or at a discount, which would not have changed the overall solution method but would have changed the numbers. The following journal entry would be recorded for this issuance: Dr Cash 1,000,000 Dr Discount 100,000 Cr Bonds payable 1,000,000 Cr APIC—Warrants 100,000

*Fair Value Adjustment/Goodwill*

The difference between the fair value of the subsidiary and the book value of the subsidiary net assets should be allocated as follows *(BIG):* B 1. Balance sheet adjustment of the *subsidiary's assets and liabilities* from book value to fair value. I 2. Identifiable *intangible assets* recorded at fair value. G 3. *Goodwill* is excess. *Under U.S. GAAP, goodwill is calculated as follows (full goodwill method):* *Goodwill = 100% Fair value of subsidiary - 100% Fair value of subsidiary's net assets* IFRS permits the use of the full goodwill method or the partial goodwill method. *Under the partial goodwill method, goodwill is calculated as follows:* *Goodwill = % Acquisition cost - % Fair value of subsidiary's net assets acquired* *Goodwill recognized in a business combination is not amortized.* Instead, it is tested for impairment, and a loss is recognized in income from continuing operations if the goodwill is impaired.

Bond Issuance Costs

Under both IFRS and U.S. GAAP, bond issuance costs are presented on the balance sheet as a direct reduction of the carrying amount of the bond, similar to bond discounts. When bonds are issued, the bond proceeds are recorded net of the bond issuance costs. Bond issuance costs are amortized as interest expense over the life of the bond using the effective interest method. Dr Cash $XXX Cr Bonds payable $XXX Cr Premium; on B/P less bond issuance costs $XXX Dr Cash $XXX Cr Discount on B/P less bond issuance costs $XXX Cr Bonds payable $XXX Issuance of Bonds Between Interest Dates When bonds are issued between interest dates, the amount of interest that has accrued since the last interest payment is added to the price of the bonds and is reimbursed at the next interest payment date to the purchaser. (The purchaser gets the full interest payment regardless of how long he/she has held the bond.) Dr Cash $XXX Dr Discount on B/P $XXX Cr Bonds payable $XXX Cr Interest expense (or payable) $XXX

MCQ-00325 *Peel Co. received a cash dividend from a common stock investment. Should Peel report an increase in the investment account if it uses the fair value method or the equity method of accounting?* Fair Value -- Equity A. Yes No B. No No C. No Yes D. Yes Yes

B. No No Choice "2" is correct, No - No. Under the fair value method, receipt of a dividend is recorded as income and does not affect the investment account. Dr Cash Cr Equity income Under the equity method, receipt of a dividend is recorded as a decrease in the investment account. Dr Cash Cr Equity investment

MCQ-04694 *Which of the following is an example of activities that would typically be excluded in research and development costs under U.S. GAAP?* A. Laboratory research aimed at discovery of new knowledge. B. Quality control during commercial production, including routine testing of products. C. Testing in search for, or evaluation of, product or process alternatives. D. Design, construction, and testing of preproduction prototypes and modes.

B. Quality control during commercial production, including routine testing of products. *Expensed as Operating/PPE Expenses (Not R&D)* - Routine periodic design changes, market research, executive salaries, quality control testing, post-production costs, and commissions. Research is the planned efforts of a company to discover new information that will help either create or improve a new product, service, process, or technique or one in current use. Items not considered research and development include: Routine periodic design changes to old products or troubleshooting in production stage, marketing research, quality control testing and reformulation of a chemical compound.

MCQ-09427 *During the fiscal year ended June 30, Year 1, Friendship Foundation, a private non-for-profit corporation, received pledges without donor restrictions for $300,000 payable in three years from institutional donors who have made contributions of this character in prior years. Amounts are deemed to be fully collectible. The foundation also received $150,000 specifically payable to various charities named by the donor. In its June 30, Year 1, financial statements, Friendship Foundation would classify these transactions as,* I. Revenue Without Donor Restrictions: II. Revenue With Donor Restrictions: I. II. A. $300,000 $150,000 B. $450,000 $0 C. $0 $300,000 D. $300,000 $0

C. $0 $300,000 Pledges without donor restrictions of $300,000 *due in three years* carry an implied time restriction. These pledges are classified as with donor restrictions. Other receipts payable to specific beneficiaries are agency transactions that would be recorded as an agency liability and not reflected as revenue. Choice "1" is incorrect. Pledges without donor restrictions of $300,000 due in three years carry an implied time restriction. These pledges are classified as with donor restrictions. Other receipts payable to specific beneficiaries are agency transactions that would be recorded as an agency liability and not reflected as revenue. Choice "2" is incorrect. Pledges without donor restrictions of $300,000 due in three years carry an implied time restriction. These pledges are classified as with donor restrictions. Other receipts payable to specific beneficiaries are agency transactions that would be recorded as an agency liability and not reflected as revenue. Choice "4" is incorrect. Pledges without donor restrictions of $300,000 due in three years carry an implied time restriction. These pledges are classified as with donor restrictions.

Pension Plan Contributions

An employer's contribution to its defined benefit pension plan(s) increases the pension benefit asset (overfunded pension plans) or decreases the pension benefit liability (underfunded pension plans). Journal entry to record pension plan contribution: DR Pension benefit asset/liability $XXX CR Cash $XXX

MCQ-09266 On January 1, Year 1, Schreiber Company purchased a $300,000 machine with a five-year useful life and no salvage value. The machine was depreciated by an accelerated method for book and tax purposes. The machine's carrying amount was $120,000 on December 31, Year 2. On January 1, Year 3, Schreiber changed to the straight-line method for financial statement purposes. Schreiber's income tax rate is 40%. Assuming that Schreiber can justify the change, in its Year 3 statement of retained earnings, what amount should Schreiber report as the cumulative effect of this change? A. $60,000 B. $36,000 C. $0 D. $24,000

C. $0 *A change in the method of depreciation, amortization, or depletion is considered to be both a change in accounting principle and a change in estimate.* These changes should be accounted for as changes in estimate and are handled prospectively. The new depreciation method should be used as of the beginning of the year of change in estimate and should start with the current book value of the underlying asset. No adjustment should be made to retained earnings.

Conventional Retail Method

Converts inventory at retail to inventory at cost. This is accomplished via a cost/retail ratio. Markups are included in the ratio, whereas, markdowns are excluded, resulting in lower of cost or market.

MCQ-00749 *Gow and Cubb formed a partnership on March 1 and contributed the following assets:* *Gow* *Cash $80,000* *Cubb* *Equipment (market value) $50,000* *The equipment was subject to a chattel mortgage of $10,000 that was assumed by the partnership. The partners agreed to share profits and losses equally. Cubb's capital account at March 1 should be:* A. $50,000 B. $45,000 C. $40,000 D. $60,000

Rule: Assets contributed by partners to a partnership are valued at fair market value of the assets, net of any related liabilities. Total --- Gow --- Cubb Cash 80,000 NA NA Equipment at FMV 50,000 NA 50,000 Total assets 130,000 NA NA Chattel mortgage on equipment (10,000) NA (10,000) Net assets contributed 120,000 = 80,000 40,000 (The fact that the partners agree to "share profits equally" does not affect their partnership capital accounts from contribution of assets.)

Interest on Self-Constructed Assets

*For the CPA Exam, it is important to remember two rules concerning capitalized interest:* *Rule 1: Only capitalized interest on money actually spent, not on the total amount borrowed.* *Rule 2: The amount of capitalized interest is the lower of:* *- actual interest incurred or,* *- computed capitalized interest (avoidable interest).* General Rule: Interest is expensed as incurred Exception: *During* (permit filed) construction for use in business *Weighted Avg. Construction Expenditures x Interest rate = Capitalized interest (not amt. borrowed) The interest capitalized cannot exceed the actual interest cost.

Investment in Equity Securities

0-20% = FV method 20-50% = Equity method > 50% = Consolidation Investments in Equity Securities: Equity investments are typically carried at fair value through net income (FVTNI), with *unrealized gains/losses included in earnings as they occur (income statement).* For equity investments that do not have a readily determinable fair value, the practicability exception allows an entity to measure an investment at cost plus/minus observable price changes of identical or similar investments, less impairment.

MCQ-00375 Derby Co. incurred costs to modify its building and to rearrange its production line. As a result, an overall reduction in production costs is expected. However, the modifications did not increase the building's market value, and the rearrangement did not extend the production line's life. Should the building modification costs and the production line rearrangement costs be capitalized? I. Building Modification Costs: II. Production Line Rearrangement Costs: I. II. A. No No B. Yes Yes C. Yes No D. Yes No

B. Yes Yes Rule: Expense ordinary repairs but capitalize expenditures, which are "additions" or "benefit several periods" or "improve efficiency" as is the case in this question.

*Changes in accounting principles, estimates, entities, and correction of errors*

Change in accounting Principle = Restate to BOY RE Net of Tax -Change in inventory valuation to other than LIFO -Change in recognition of revenue (i.e., Switch from completed contract method to percentage of completion) Change in estimate = Prospective -Change in depreciation -Change computation of warranty costs -Change in collection procedures (i.e., 2% to 1%) -Only exception Change in inventory valuation to *LIFO* Change in entity = Restate to BOY RE Net of Tax Error = Prior period adjustment = Restate to BOY RE Net of Tax -Change from cash basis to GAAP -Entirely expense insurance premium in one year rather than over its 3 year period

MCQ-09345 Pate paid $50,000 and gave a plot of undeveloped land with a carrying amount of $320,000 and a fair value of $450,000 to Bizzell Co. in exchange for a plot of undeveloped land with a fair value of $500,000. The land was carried on Bizzell's books at $350,000. The exchange is one that has commercial substance under U.S. GAAP. At what amount is the land received from Pate recorded on Bizzell's books? A. $370,000 B. $320,000 C. $500,000 D. $450,000

D. $450,000 Bizzell's journal entry: Dr Cash $50,000 (Cash received from Pate) Dr Land $450,000 (FV of land received from Pate) Cr Old land 350,000 (Bizzell removes old land) Cr Gain 150,000 (500k FV of old asset - 350k Bv of old asset)

MCQ-06458 *Penn Corp. paid $300,000 for 75 percent of the outstanding common stock of Star Co. At that time, Star had the following condensed balance sheet,* *Carrying Amounts:* *Current assets $40,000* *Plant and equipment, net 380,000* *Liabilities 200,000* *Stockholders' equity 220,000* *The fair value of the plant and equipment was $60,000 more than its recorded carrying amount. The fair value and carrying amounts were equal for all other assets and liabilities. What amount of goodwill related to Star's acquisition should Penn report on its consolidated balance sheet under the IFRS partial goodwill method?* A. $120,000 B. $20,000 C. $90,000 D. $40,000

$90,000 Choice "3" is correct. Under the IFRS partial goodwill method, goodwill is calculated as follows: Goodwill = Acquisition cost - Fair value of subsidiary's net assets acquired The fair value of the subsidiary's net assets is: FV of subsidiary's net assets = $40,000 + $380,000 + $60,000 - $200,000 = $280,000 Therefore, the goodwill to be reported under the IFRS partial goodwill method is: Goodwill = $300,000 - ($280,000 x 75%) = $300,000 - $210,000 = $90,000

Non-For-Profit Donated Services and Donated Works of Art

*Contributions of services* are recorded as revenue *some of the time.* The services must either enhance a physical asset or meet the following *(SOME)* criteria: they require *specialized skills, are otherwise needed, and are measured easily.* Services that meet the criteria are recorded as revenues and assets or expenses at their fair value as follows: Dr Expense or asset $XXX Cr Contributions—Nonoperating revenue $XXX *Donated works of art* are not required to be recorded by the non-for-profit if *all of the following criteria are met* and the capitalization policy is consistently applied: - The item is held for *public viewing;* - The work of art is *cared for* by the not-for-profit organization; and - Proceeds, if the art is sold, must be *used to purchase other works of art.*

Other Liabilities and Debt Covenants

*Current liabilities: are obligations that mature within one year or the operating cycle, whichever is longer. Current liabilities are valued at their settlement values.* The most common current liabilities are: - Trade accounts payable - Refundable deposits - Sales, use and property taxes payable - Accrued salaries, wages and bonuses - Unpaid payroll deductions and employer's share of payroll taxes *Notes payable: must be recorded at present value at the date of issuance.* - When a note contains either no interest or an unreasonable rate of interest, the present value of the obligation is shown at the appropriate market interest rate. The difference between the face value of the note and its present value is a discount or premium to be amortized over the life of the note using the effective interest method. Debt Covenants Debt covenants prohibit certain actions by debtors that might negatively affect the position of the creditor. When covenants are violated, the loan is in technical default. The creditor and borrower generally work out concessions that may include a higher interest rate to be paid by the borrower. - The market rate of interest should approximate the rate that would have been if negotiated by an independent lender.

*Eliminate 100% of Intercompany Transactions (When consolidating even if not 100% ownerhship)*

*Eliminate 100% of Intercompany Transactions (When consolidating even if not 100% ownerhship)* Payable/Receivable In a consolidated balance sheet, all intercompany payables and receivables are eliminated. Dr Account payable $XXX Cr Accounts receivable $XXX *Inventory* *Affiliated companies often sell inventory to one another. Intercompany sales and intercompany cost of goods sold should be eliminated. This entry is made if the books are open. *Any intercompany profit from the intercompany inventory transaction must also be eliminated* against the purchaser's ending inventory and cost of goods sold.* Intercompany Sales (selling affiliate) $XXX Intercompany COGS (selling affiliate) $XXX COGS (purchasing affiliate) XXX Inventory (purchasing affiliate) XXX Fixed Assets The gain or loss on the intercompany sale of a depreciable asset is unrealized from a consolidated financial statement perspective until the asset is sold to an outsider. A working paper eliminating entry in the period of the intercompany *sale eliminates the intercompany gain/loss* and adjusts the asset and the accumulated depreciation to their original balances on the date of sale. The *excess depreciation on the gain must also be eliminated.* Dr Gain $XXX Cr Equipment $XXX Cr Accumulated depreciation XXX (The three Dr eliminate intercompany sale) Dr Accumulated depreciation XXX Cr Depreciation expense (RE) XXX (The two Cr correct/reduce depreciation back to "old amounts") Bonds If one member of the consolidated group acquires an affiliate's debt from an outsider, the debt is *considered to be retired and a gain/loss is recognized.* This gain/loss on extinguishment of debt is calculated as the difference between the price paid to acquire the debt and the book value of the debt. This gain/loss is not reported on either company's books, but is recorded on the consolidated income statement through an elimination entry. All intercompany account balances are also *eliminated; e.g., bond interest payable and bond interest receivable.* Dr Bond interest payable $XXX Cr Bond interest receivable $XXX Dr Bonds payable XXX Dr Premium on bonds payable XXX Dr Loss XXX Cr Investment in bonds XXX Cr Discount on bonds payable XXX Cr Gain XXX

*Summary of Accounting Changes and Necessary Treatments*

*From one GAAP/IFRS principle to another GAAP/IFRS principle* Example: - Adopt a new standard - Change methods of Statement of Retained Earnings: Inventory costing—FIFO to Average Retrospective application, compute cumulative effect and report net of tax by adjusting beginning retained earnings of earliest year presented *Changes in principle— Exceptions (require prospective treatment)* Example: - From any inventory valuation method to LIFO (U.S. GAAP only) - Change depreciation methods —SL to SYD Income Statement: - Prospective application, the beginning inventory of the year of change is the first LIFO layer - Apply new depreciation method to remaining book value as of the beginning of the year *Changes in entity* Example: - Consolidation of a subsidiary not previously included in consolidated FS - Report consolidated FS in place of individual statements Statement of Retained Earnings: - Retrospective adjustments (plus or minus) net of tax, against the beginning balance of the retained earnings under the caption of "Prior Period Adjustments" - Restate all financial statements presented *Neither a change in principle nor a change in estimate* Examples: - Change from fair value method to equity method because an increase in ownership now qualifies as equity method investment. Adopt the equity method as of the date the investment qualifies for equity method. Income Statement: If the investment was previously accounted for as an available-for-sale security, recognize in earnings the unrealized holding gain or loss from accumulated other comprehensive income. Statement of Retained Earnings: Because the cost of acquiring the additional interest in the investee is added to the carrying value of the previously held investment, retroactive adjustments are not required. *Correction of errors* Examples: - From cash to accrual - Errors made in prior statements Statement of Retained Earnings: - Retroactive adjustments (plus or minus) net of tax, against the beginning balance of the retained earnings under the caption of "Prior Period Adjustments" - Restate all financial statements presented that are affected *Changes in estimate* Examples: - Depreciation method - Useful life of depreciable asset - Residual value - Bad debt % - Loss accruals Income Statement: - Prospective application, account for in the current statement "above the line" - No cumulative effect

Types of Non-exchange revenues

*Modified Accrual Basis of Accounting = GRaSPP* Derived Non-exchange Tax Revenues A *sales tax* or an *income tax* is considered to be "derived" tax revenue; it is a tax that comes as a result of (is derived from) economic activity. Derived non-exchange tax revenues are accrued based on the timing of receipt. Receipts due at year-end and actually received within *60 days* of year-end are accrued and recognized as revenue. Imposed Non-exchange Revenues *Fines* and *property taxes* are imposed non-exchange revenues since the taxpayer's obligation is imposed by an enforceable claim by the government. Imposed non-exchange *revenues* are typically *accrued when billed* since collection is not in doubt. Collection of fines is based upon enforcement of a penalty resulting from the violation of law (e.g., driver's licenses can be revoked, cars can be impounded, etc.). Liens on property (allowed by law) are used to enforce property tax collection. Government Mandated Non-exchange Transactions *Grants* are conveyed by one government to another, (a state, or a county) to mandate certain activities. *Revenues* are recognized when *eligibility requirements* are met and the revenues are both measurable and available. Voluntary Non-exchange Transactions Resources are *willingly conveyed* by a government to another for a particular purpose or use without an equal exchange of value. Revenues are recognized when *restrictions are met.* *Accrual Basis of Accounting = SE CIPPOE* The economic resources measurement focus is accomplished using the accrual basis of accounting where *revenues* are recorded when *earned* and *expenses* are recorded when *incurred.*

Lessor Accounting

*Operating Capital Lease = Keep asset & depreciate* With an operating lease, the lessor will keep the asset on its balance sheet, which will include depreciating it. Lease income will be recognized on a straight-line basis. *Sales-Type and Direct Financing Leases = Remove asset you sold it* For sales-type and direct financing leases, the lessor will recognize a *lease receivable* on its balance sheet. On the income statement, interest income will be recognized over the life of the lease using the effective interest method. The excess cash payments from the lessee (above the interest income) will reduce the lease receivable over the life of the lease.

MCQ-04501 *How would a 5% stock dividend affect each of the following?* Assets -- Total stockholders' equity -- Retained earnings Decrease Decrease Increase No effect Increase Increase Decrease No effect Decrease Decrease No effect No effect Decrease

*Stock Dividends (Not on CF)* A stock dividend of *less than 20-25 percent* of the outstanding capital stock is recorded at *fair value (small dividend).* *Total stockholders' equity does not change it only changes form bc APIC increases and RE decreases.* Dr RE (FV) 100 [decreases RE] Cr CS (Par) 50 [increases CS] Cr APIC 50 [increases APIC] Choice "4" is correct. No effect on assets. No effect on equity. Decrease to retained earnings. Rule: A stock dividend (less than 20-25% of the stock outstanding) transfers the FMV of the stock dividend at declaration date from retained earnings to capital stock and paid-in capital. There is no effect on total stockholders' equity because all transfers take place within stockholders' equity.

*Research and Development (R&D)*

*U.S. GAAP = Generally expensed in period incurred* Under U.S. GAAP, research and development costs must be expensed in the period incurred. In general, items to be expensed as R&D include: equipment, material, labor, overhead, design, testing, engineering, modification, and salaries of research staff. Exceptions to expensing include: *1. Software after feasibility (working product) are capitalized, not expensed* (Hardware is always expensed, GAAP only) *2. PPE has alternative future use capitalized, not expensed* Alternative Use Capitalize and then depreciate as R&D expense if alternative use on other future projects is planned (e.g., building will be used for other projects). Expensed as Operating/PPE Expenses (Not R&D) - Routine periodic design changes, market research, executive salaries, quality control testing, post-production costs, and commissions. IFRS *Under IFRS, research costs must be expensed like U.S. GAAP, but development after feasibility costs may be capitalized if certain criteria are met.* Research and development costs for computer hardware will be expensed under U.S. GAAP. Under U.S. GAAP, R&D for computer hardware cannot be capitalized. IFRS will allow for capitalization once technological feasibility has been established, but prior to that all costs will be expensed. For computer software, GAAP will require expensing all research and development (prior to technological feasibility) costs. Development post technological feasibility may be capitalized. IFRS does not have a specific accounting for "computer software" costs, but they are treated in a similar manner to other intangible assets.

*Goodwill impairment*

*US GAAP Goodwill Impairment* One-step approach: CV reporting unit < FV including Goodwill CV reporting unit - FV including Goodwill = Impairment loss The max charge is the amount of CV of goodwill. *IFRS Goodwill Impairment* Cash generating unit (CGU) Recoverable amount: Step 1: Pick higher of NRV (FV - cost to sell) or Value in use (PV of future CF), this is CGU recoverable amount (RA) Step 2: RA - CV = CGU Impairment loss

Statement of Net Position / Balance Sheet (Government-Wide Financial Statement)

- Full accrual accounting - No fiduciary funds (CIPPOE) are reported All assets and deferred outflows of resources <All liabilities and deferred inflows of resources> = Net position Three components 1. restricted 2. unrestricted 3. net investment in cap assets

Statement of Activities / Income Statement (Government-Wide Financial Statement)

- Full accrual accounting - No fiduciary funds (CIPPOE) are reported Reports expenses and revenues classified by program or function; uses separate columns for governmental activities, business-type activities, and component units

Investing Activities

-Making and collecting loans and acquiring and disposing of investments, PPE, and Intangibles. Relate to Long term assets. -*Include long term/noncurrent assets.* -Lend money (negative) -> Collect money (positive) -Buy or sell investments (debt or securities) -Buy or sell PPE / Fixed assets -Proceeds from works of art or purchasing works of art.

Derivatives and Hedging

2.1 Definition Derivatives derive their value from other securities. A derivative must have *all three* of the following characteristics: 1. One or more underlyings (Stock price, commodity price, interest rates), and one or more notional amounts (Dollar amounts used to calculate G/L) or payment provisions (or both); and 2. No initial net investment (or smaller than would be expected); and 3. Its terms *require or permit a net settlement.* - An underlying is a specified price, rate, or other variable, e.g., $10 a bushel. - A notional amount is a specified unit of measure on which the derivative is valued, e.g., 10,000 bushels. - The value or settlement amount is the amount determined by the multiplication of the notional amount and the underlying, e.g., 10,000 bushels × $10 per unit = $100,000. - Examples of common derivatives are *forward* contracts, *futures, swaps, and options.* - *Derivatives are reported as assets (receivables) or liabilities (payables) and are measured at fair value just like other financial instruments.* Hedging Instruments No Hedge Designation No hedge designation. Just speculation. *Changes in fair value are fully included in income.* *Fair Value Hedge-Own asset-inventory-risk value decreases* A fair value hedge hedges an exposure to changes in fair value of a recorded asset or liability or unrecognized firm commitment. *Changes in fair value are included in income but are offset by changes in the fair value of the hedged item.* Cash Flow Hedge *Owed money in foreign currency - risk FV increases & pay more.* *Owed money in foreign currency - risk FV decreases & receive less.* A cash flow hedge hedges an exposure to variability in the cash flows of a recognized asset or a forecasted transaction. *Changes in fair value of the "ineffective portion" of a cash flow hedge are included in income;* changes in fair value of the *"effective portion" of a cash flow hedge are included in the stockholders' equity as part of other comprehensive income (OCI) until the related cash flows are realized. (PUFI"E"R)*

MCQ-09347 The Holly Company incurred research and development costs in Year 1 as follows: Equipment acquired for use in various R&D projects $400,000 Depreciation on the above equipment 60,000 Materials used 100,000 Compensation costs of personnel 200,000 Fees to outside consulting firms 70,000 Indirect costs appropriately allocated 100,000 The total research and development expense in Holly 's Year 1 income statement under U.S. GAAP should be: 1. $930,000 2. $870,000 3. $530,000 4. $470,000

3. $530,000 Equipment acquired for use in "various R&D projects" $400,000 - Capitalize and depreciate Depreciation on the above equipment 60,000 Materials used 100,000 - R&D expense Compensation costs of personnel 200,000 - R&D expense Fees to outside consulting firms 70,000 - R&D expense Indirect costs appropriately allocated 100,000 - R&D expense *U.S. GAAP = Generally expensed in period incurred* Under U.S. GAAP, research and development costs must be expensed in the period incurred. In general, items to be expensed as R&D include: equipment, material, labor, overhead, design, testing, engineering, modification, and salaries of research staff. Exceptions to expensing include: *1. Software after feasibility are capitalized, not expensed* *2. PPE has alternative future use capitalized, not expensed*

MCQ-09281 Gonzales Company purchased a machine on January 1, Year 1 for $600,000. On the date of acquisition, the machine had an estimated useful life of six years with no salvage value. The machine was being depreciated on a straight-line basis. On January 1, Year 4, Gonzales determined that the machine had an estimated life of eight years from the date of acquisition. An accounting change was made in Year 4. What is the amount of the depreciation expense that should be recorded for the year ended Year 4? 1. $75,000 2. $100,000 3. $60,000 4. $0

3. $60,000 The accounting change is a change in accounting estimate, which is handled prospectively. Before the change, depreciation was calculated on the straight-line basis using a life of 6 years. The depreciable base was $600,000, so the accumulated depreciation for the 3 years before the change was $300,000 ($600,000/6 × 3). Thus, the carrying amount at the date of the change was also $300,000. A change in accounting estimate is handled prospectively. In this case, the remaining carrying amount of $300,000 is depreciated over the new remaining life of 5 years (8 − 3). The new depreciation expense each year is $60,000. Note that the question is asking about depreciation expense. It just as easily could have been accumulated depreciation or it could have been the net book value of the asset.

MCQ-09296 On December 31, Year 10, Brown Company changed its inventory valuation method from the weighted average method to FIFO for financial statement purposes. The change will result in an $800,000 decrease in the beginning inventory at January 1, Year 10. The tax rate is 30%. The cumulative effect of this accounting change for the year ended December 31, Year 10 in the statement of retained earnings is: 1. $0 2. $800,000 3. $240,000 4. $560,000

4. $560,000 Any change from one generally accepted accounting principle to another generally accepted accounting principle is recognized using the *retrospective approach by adjusting beginning retained earnings for the cumulative effect of the change, net of tax.* Prior period financial statements are restated (IDA). This particular change is from the weighted average to FIFO. It is a change in accounting principle. Accounting changes are reported on the statement of retained earnings net of tax. In this question, the cumulative effect before taxes is $800,000. Net of tax, it is $560,000 ($800,000 x 0.70).

MCQ-09344 The City of Richardson reported a change in fund balances of $2,002,000 in its governmental funds Statement of Revenues, Expenditures, and Changes in Fund Balances for the year ended December 31, Year 1. Additional information: 1.Capital outlay expenditures amounted to $10,000,000 in the modified accrual statement. General government fixed assets amounted to $160,000,000 excluding land and had an average life of 20 years. 2. The modified accrual statement reported proceeds from the sale of land in the amount of $1,000,000. The land had a basis of $800,000. 3. Property taxes had been levied in the amount of $20,000,000. It was estimated that 3% would be uncollected, that $1,000,000 would be collected within 60 days of year-end, and that $400,000 would be collected more than 60 days from year-end. The City had recognized the maximum permitted under modified accrual accounting. 4. $370,000 of property taxes had been deferred at the end of the previous year and was recognized under modified accrual as revenue in the current year. 5. The modified accrual statement reflected debt service expenditures in the amount of $1,000,000 for interest and $1,500,000 for principal. No adjustment was necessary for interest accruals at year-end. 6. Compensated absences charges, on the full accrual basis, amounted to $100,000 more than under the modified accrual basis. 7. The change in net position in the governmental column in the government-wide Statement of Activities for the year ended December 31, Year 1 is: 1. 4,602,000 2. 5,202,000 3. 3,832,000 4. 4,632,000

4. 4,632,000 *(CAN CPAS RIDE or SIT)* = 2,002,000 fund balance 1. $10,000,0000 Modified accrual (MA) depri. exp $160,000,000/20 yrs = 8,000,000 Full accrual (FA) depri. exp = 2,000,000 Add back excess depri. exp under MA 2. $1,000,000 MA reports proceeds from sale. You only need to report the 200,000 (1,000,000 - 800,000) gain as proceeds from sale under FA. So you must subtract 800,000 to get to 200,000 NI for FA. = 800,000 Subtract get to 200,000 NI for FA 3. $1,000,00 under MA (within 60 days after year-end). $1,400,00 under FA bc it has been earned for the period. = 400,000 Add to recognize 1,400,000 NI for FA. 4. The 370,000 that was recognized under MA this year should have been recognized in the period earned under FA. You must subtract the 370,000. = 370,000 Subtract to put it back into last year's NI. 5. Under MA principal and interest are both treated as an expenditure of 2,500,000. Under FA principal is not an expense it is a principal reduction so you must add 1,500,000 under FA. = 1,500,000 Add back to correct excess expenditure under MA. 6. Subtract 100,000 excess absence charges under FA than MA. = 100,000 Subtract excess absence charges under FA 2,002,000 + 2,000,00 - 800,000 + 400,000 - 370,000 + 1,500,000 - 100,000 = 4,632,000 Change in net postion

Form 10-Q Quarterly Report

40 large accelerated-40 accelerated-45 all others -Must be filed quarterly by U.S. registered companies (issuers). -The filing date for the 10-Q is 40 days after the end of the fiscal quarter for large accelerated filers (larger than 700 million) and accelerated filers (75 - 699 million). -The form contains unaudited financial statements prepared using GAAP, interim MD&A, and certain disclosures.

Sale-Leaseback Transactions

A sale-leaseback occurs when one party (the seller) which has control of an asset transfers it to another party (the buyer), with a subsequent lease of the same asset where the seller becomes the lessee and the buyer becomes the lessor. ASSET -> Transferred ownership from Seller to Buyer which is a *Sales-Leaseback* ASSET -> Leased by the Seller (now the Lessee) from the Buyer (now the Lessor) which is an *Operating Capital Lease* To qualify as a sale, revenue recognition requirements must be met (i.e., a contract exists and *control has transferred* from the seller to the buyer). If these criteria are not met, this will be treated as a financing transaction. Criteria to Be Considered a Sale Are Met = *Operating Leaseback* Each party must then determine whether the transaction is at fair value using a two-step process: - Determine which of the two sets of information below are more readily determinable: Set 1: Asset sale price and fair value Set 2: PV of lease payments and PV of market rental payments - Of the one that is more determinable, any difference between the two data points will require an adjustment to either the sale price or the purchase price. Two transactions upon execution of the sale-leaseback: - The sale, along with recognition of profit/loss, would be recorded *(leaseback must be an operating lease, as a finance lease equates to a repurchase and a "failed sale").* - The lease itself would be recorded. Criteria to Be Considered a Sale Are Not Met = *Financing Leaseback* Seller (lessee) and buyer (lessor) will treat this as a "failed sale" (financing transaction), which will involve the *seller (lessee) recording a financing liability* and *buyer (lessor) recording a financing receivable.* The seller will continue to recognize the asset on its books.

Troubled Debt Restructuring

A troubled debt restructuring is one in which the creditor allows the debtor certain concessions such as reduced interest rates, extension of maturity dates, reduction of the face amount of the debt, and reduction of the amount of accrued interest as a result of the debtor's financial difficulty. *Accounting by the Debtor* *Assets marked to market (to FV) through IS.* *- Recognize gain/loss on:* *FV Asset Transferred* *< NBV Asset Transferred >* *= Gain / Loss* - Recognize gain on: "Concession" is a gain Carrying Amount of the Payable < FV Asset Transferred > = Gain Transfer of Equity - Recognize gain on: Carrying Amount of the Payable < FV Equity Transferred > = Gain Accounting by the Creditor A loan restructured in a troubled debt restructuring is accounted for by the creditor as an impaired loan. Bad debt expense is recognized for the difference between the loan amount and the fair value of whatever has been received and/or the present value of expected cash to be received, discounted at the loan's historical effective interest rate.

MCQ-09329 *The County of Deutsch appropriated $45,000 in its General Fund for miscellaneous supplies for its fiscal year ended September 30, Year 1. The County found that it had paid $15,000 for miscellaneous supplies in November Year 0 and issued a $30,000 PO to a sole source vendor for miscellaneous supplies in December Year 0. By August Year 1, the County had received $20,000 related to the order but did not pay the vendor until October pending tax receipts. Appropriations do not lapse. What was the County of Deutsch's available appropriation at September 30, Year 1?* A. $0 B. $15,000 C. $10,000 D. $5,000

A. $0 Fund accounting mechanics generally focus on the accounting for the governmental funds and require knowledge of the journal entries used to record the *budget, actual activities, and encumbrances* (BAE BAE). Budgetary Activity Encumbrances The County's available appropriation is computed as the difference between the original appropriation and the funds either expended or encumbered. Appropriation 45,000 Less: Expended by November, Year 0 (15,000) Accrued by August, Year 1 (20,000) Budgetary control (purchase order) 30,000 Order received (20,000) Budgetary control (10,000) Available appropriation 0 *Budgetary:* $45,000 appropriated JE: Dr Estimated revenues 45,000 Cr Appropriation 45,000 *Activity:* - 15,000 expenditure JE: Dr Capital outlay expenditure 15,000 Cr Cash 15,000 *Encumbrance:* - 30,000 PO issued, record encumbrance + 20,000 Reverse out encumbrance - 20,000 Expenditure once encumbrance is reversed JE to record the issuance of a purchase order: Dr Encumbrances 30,000 Cr Budget control 30,000 Upon receipt of goods, the encumbrance of funds associated with the issued order is reversed and the related expenditure is recorded. Dr Budget control 20,000 Cr Encumbrnaces 20,000 Dr Expenditure 20,000 Cr Vouchers payable 20,000

On January 1, Year 1, LaGuardia Company signed a five-year non-cancelable lease for a new machine with a fair value of $80,000, requiring $8,000 annual payments at the beginning of each year. The machine has a useful life of 10 years, with no salvage value. Title did not pass to LaGuardia, nor was there any written purchase option that Laguardia was reasonably certain to exercise. LaGuardia uses straight-line depreciation for all of its plant assets. Aggregate lease payments has a present value on January 1, 20X1, of $40,000 based on an appropriate interest rate. For Year 1, LaGuardia should record depreciation (amortization) expense for the leased machine under U.S. GAAP at: A. $0 B. $7,500 C. $6,000 D. $8,000

A. $0 In this question, they want the depreciation expense for a leased machine. The first issue to address is whether the lease is a finance lease. *An operating lease is treated like renting—lease payments are considered as operating expenses. Assets being leased are not recorded on the company's balance sheet; they are expensed on the income statement as lease expense. So, they affect both operating and net income.* There are five criteria for a finance lease under U.S. GAAP. Only one has to be met. The first criterion is the transfer of ownership, and ownership does not transfer in this question. The second criterion is the existence of a written purchase option that the lessee is reasonably certain to exercise (which is not the case here), and there is no written purchase option in this question. There is no other information regarding the remaining "OWNES" criteria, which means that this will be treated as an operating lease. As such, the leased machine itself will not be depreciated on the books of the lessee. *An ROU asset will be booked, which will be amortized by the lessee.*

MCQ-09447 During the year ended June 30, Year 0, the Aberdeen Care Clinic, a not-for-profit organization, received an endowment for $2,300,000. The endowment is to remain intact with income and appreciation from the investment of the endowment to be used in support of a research program to assist the elderly. By June 30, Year 2, the endowment had accumulated $110,000 in gains. On July 1, Year 2, the company liquidated 65% of the cumulative appreciation of the endowment for use in the research program. During the year ended June 30, Year3, the endowment earned $30,000, which it spent on the research program but, at year end, suffered market losses that reduced the total value of the investment below its initially recorded value to $2,250,000. The amounts reported as investment losses by net asset category for the year ended June 30, Year 3, would be: With Donor Restrictions -- Without Donor Restrictions A. $0 $(88,500) B. $0 $(50,000) C. $(88,500) $0 D. $(50,000) $(38,500)

A. $0 $(88,500) Not-for-profit organizations account for assets with donor restrictions that are perpetual in nature in endowment funds. Generally, a benefactor will donate an amount and direct that earnings and not principal be used for specific or general purposes of the not-for-profit. An underwater endowment is a donor-restricted endowment fund for which the fair value at the reporting date is less than either the original gift amount or the amount required to be maintained by the donor or by law. Underwater endowment funds will report accumulated losses together with the endowment fund in net assets with donor restrictions. Losses are not recorded in the permanently restricted net asset category; they are recorded as temporarily restricted, to the extent that temporarily restricted assets have not been used for their intended purpose, and as unrestricted for any amount not previously absorbed by temporarily restricted net assets. The value of investments at the beginning of the year was $2,410,000. Investments experienced a $160,000 reduction in value accounted for in part by the sale of assets ($110,000 × 65% = $71,500) and a loss of $88,500 ($160,000 − $71,500). The allocation of the loss between categories is computed as follows: *With DonorRestrictions* Beginning of Year 2,300,000 Accumulated gains 110,000 Liquidation (71,500) Losses *(plug)* (88,500) = End of Year 2,250,000

MCQ-09399 On January 1, Year 1, the Bagel Conservatory received a bequest of marketable securities that was valued at $600,000. The bequest represented a permanent endowment whose earnings were to be used to install advanced oven systems for advanced bagel research. At December 31, Year 1, the value of the investment had increased by $40,000 by the end of Year 2, the fair value of the investment had declined to $580,000. No transactions other than the unrealized changes in investment value occurred during the two year period. As a result of these changes net asset balances at December 31, Year 2 would be as follows: Without Donor Restrictions -- With Donor Restrictions A. $0 $580,000 B. ($60,000) $640,000 C. ($20,000) $600,000 D. $0 $620,000

A. $0 $580,000 Not-for-profit organizations account for assets with donor restrictions that are perpetual in nature in endowment funds. Generally, a benefactor will donate an amount and direct that earnings and not principal be used for specific or general purposes of the not-for-profit. An underwater endowment is a donor-restricted endowment fund for which the fair value at the reporting date is less than either the original gift amount or the amount required to be maintained by the donor or by law. Underwater endowment funds will report accumulated losses together with the endowment fund in net assets with donor restrictions. At the end of Year 2, the endowment is underwater because the value of $580,000 is less than the original amount of the gift. Underwater endowment funds are reported together with accumulated losses in net assets with donor restrictions.

MCQ-08602 *A company incurred the following costs to complete a business combination in the current year:* *Issuing debt securities $30,000* *Registering debt securities 25,000* *Legal fees 10,000* *Due diligence costs 1,000* *What amount should be reported as current-year expenses, not subject to amortization?* A. $11,000 B. $1,000 C. $66,000 D. $36,000

A. $11,000 Choice "1" is correct. *Legal fees and due diligence costs are expensed in the period incurred for business combinations.* Debt securities create liabilities, and debt security registration costs are capitalized and amortized. *Note: Do not get this confused with Finite life intangible assets which is different from business combinations, explanation below.* Finite Life Intangibles: For intangible assets with finite lives, the cost of the asset less its residual value, is amortized over its useful life, generally using the straight-line method. *Succesful defense patent legal fees, legal and other patent registration fees can be capitalized. *Goodwill cannot be amortized, but is subject to the impairment test.* Indefinite Life Intangibles: Intangible assets that have no legal or economic lives are considered to have indefinite useful lives. *These intangible assets are not amortized but are reviewed for impairment periodically.* *If a trademark is expected to be renewed indefinitely, there will be no amortization expense on the books. Amortization is only recorded for intangible assets with a definite life.* Choice "2" is incorrect. This choice incorrectly excludes the legal fees from the expense total. Choice "3" is incorrect. The costs associated with the issuance and registration of debt securities are not expensed. Choice "4" is incorrect. The debt security registration costs are not expensed; rather, they are capitalized and amortized.

MCQ-09396 The following information relates to the pension plan for the employees of Neal​ Co.: Jan . 1 Year 4 -- Dec . 31 Year 4 -- Dec 31, Year 5 Accum benefit obligation $4,400,000 $4,600,000 $6,000,000 Project benefit obligation 4,650,000 4,980,000 6,670,000 Fair value of plan assets 4,250,000 5,200,000 5,740,000 Market-related value of assets 4,100,000 5,160,000 5,650,000 Unrecognized net (gain) or loss 0 (720,000) (800,000) Settlement rate (for year) 11% 11% Expected rate of return (for year) 8% 7% Neal estimates that the average remaining service life is 16 years.​ Neal's contribution was​ $315,000 in Year 5 and benefits paid were​ $235,000. The amount of unrecognized net gain amortized in Year 5​ is: A. $12,750 B. $12,500 C. $9,688 D. $8,314

A. $12,750 The company is required to use the Corridor approach to determine the amount of gains and losses to recognize (amortize) in pension expense each period. The corridor approach is a technique used to reduce the amounts of gains and losses to be recognized as an adjustment to pension expense. *It requires recognition of certain gains and losses in excess of 10 percent of the greater of the projected benefit obligation or the market-related asset value. The 10 percent is referred to as the corridor.* *Any excess over the 10 percent should be amortized over the average remaining service period of active employees expected to participate in the plan.* This amount represents the minimum amount a company can recognize. If the gains or losses are not in excess of 10 percent of the appropriate amount, then no gain or loss may be recognized. The purpose of the corridor is to reduce the volatility of the pension expense. Unrecognized net (gain) or loss 720,000 Less market-related value of assets 516,000 Equal = 204,000 Divided by service life (given) 16 years Net gain amortized = 12,750

MCQ-09415 At the end of Year 6, the tax effects of temporary differences reported in Apple Company's year-end financial statements were as follows: Deferred tax assets (liabilities) Accelerated tax depreciation $(120,000) Warranty expense 80,000 NOL carryforward 200,000 Net change $160,000 A valuation allowance was not considered necessary. Apple anticipates that $40,000 of the deferred tax liability will reverse in Year 7, that actual warranty costs will be incurred evenly in Year 8 and Year 9, and that the NOL carryforward will be used in Year 7. On Apple's December 31, Year 6 balance sheet, what amount should be reported as a non-current deferred tax asset under U.S. GAAP? A. $160,000 B. $200,000 C. $240,000 C. $280,000

A. $160,000 Accelerated tax depreciation $(120,000) DTL = PPE, Non-current Warranty expense 80,000 DTA = Current operating cycle NOL carryforward 200,000 Orphan DTA = Reverse next year Net change $160,000 Accelerated tax depreciation is DTL because it causes BI > TI. Warranty expense is paid upfront before it is used Dr prepaid warranty expense Cr account payable (always assume accrual accounting is used). This causes BI < TI resulting in a DTA. NOL carryforward is a DTA because it is a deduction in the future for businesses.

MCQ-09362 *During Year 1, the James Company incurred costs to develop and produce a routine, low-risk computer software product, as follows:* *Detailed program design $30,000 Coding and testing to establish technological feasibility 40,000 Coding after establishment of technological feasibility 20,000 Other testing after establishment of technological feasibility 30,000 Producing product masters for training materials 50,000 Duplication of software and training materials 15,000 Packaging 10,000* *In James' December 31, Year 1 balance sheet, what amount should be reported in inventory under U.S. GAAP?* A. $25,000 B. $185,000 C. $70,000 D. $90,000

A. $25,000 Detailed program design $30,000 =Expense as R&D Coding and testing to establish technological feasibility 40,000 =Expense as R&D Coding after establishment of technological feasibility 20,000 =Capitalized as software Other testing after establishment of technological feasibility 30,000 =Capitalized as software *Producing product masters* for training materials 50,000 =Capitalized as software *Duplication of software* and training materials 15,000 =Inventory Packaging 10,000 =Inventory Until technological feasibility has been established for a product, costs incurred in creating a computer software product should be charged to research and development expense when incurred. Technological feasibility is established upon completion of a detailed program design or a working model. In this question, $70,000 would be recorded as research and development expense ($30,000 for detailed program design and $40,000 for coding and testing to establish technological feasibility). Costs incurred from the point of technological feasibility until the time when product costs are incurred are capitalized as software costs. In this question, $100,000 ($20,000 + $30,000 + $50,000) is capitalized as software cost. Product costs that can be easily associated with the items to be inventoried are reported as a part of the inventory cost (in this question, $15,000 for duplication of software and training materials and $10,000 for packaging, for a total of $25,000).

MCQ-09671 On January 1 of the current year, the balance sheet of PharmaTech Corporation included patents with a carrying value of $25 million. During the current year, the following transactions took place related to the company's patents: - $4 million in legal costs defending patents that were challenged in infringement suits of $2 million of which was for the successful defense of a patent. - $5 million in research cost, $3 million in development costs, and $1 million in legal fees related to a new drug that was granted a patent during the year. As a result of the above transactions, December 31 carrying value of the patents before amortization or impairment under U.S. GAAP is: A. $28 million B. $30 million C. $31 million D. $36 million

A. $28 million Under U.S. GAAP, capitalizable costs related to intangible assets include legal costs related to the successful defense of an intangible asset and legal fees incurred to register internally developed patents. Research and development costs and costs related to the unsuccessful defense of an intangible asset must be expensed when incurred. On December 31, the carrying value of PharmaTech's patents before amortization or impairment is calculated as follows: December 31 Carrying Value = $25 million beginning carrying value + $2 million cost of successful defense + $1 million legal fees for new patent = $28 million.

MCQ-05404 Zen Transportation Inc.'s pension trustee provided the company with the following information for its defined benefit pension plan at December 31: Projected benefit obligation $2,500,000 Fair value of pension plan assets 1,950,000 Unrecognized prior service cost 375,000 Unrecognized net gain 50,000 Unrecognized transition obligation 135,000 Net periodic pension cost 440,000 What amount would Zen report in accumulated other comprehensive income related to its pensions plan on its December 31 balance sheet under U.S. GAAP? A. $460,000 B. $190,000 C. $290,000 D. $560,000

A. $460,000 + *S*ervice cost (current item - goes to compensation) + *I*nterest cost (on the projected benefit obligation) − *R*eturn on plan assets (expected or actual) + *A*mortization of unrecognized prior service cost ± (*G*ains) and losses ± Amortization of *E*xisting net (asset) or obligation = Net periodic pension cost IR = Current items expensed *AGE -> AOCI -> Amortized out to I/S* Choice "1" is correct. Under U.S. GAAP, unrecognized prior service cost, unrecognized transition obligations and unrecognized net gains or losses must be reported in accumulated other comprehensive income until recognized as a component of net periodic pension cost through amortization. Unrecognized prior service cost, transition obligations and net losses all increase net periodic pension cost when recognized and are therefore recorded as a debit to accumulated OCI. Unrecognized transition assets and net gains decrease net periodic pension cost when recognized and are therefore recorded as a credit to accumulated OCI. For Zen, the total pension related amount to be reported in accumulated OCI (before tax) is: Unrecognized prior service cost 375,000 Unrecognized transition obligation 135,000 Unrecognized net gain (50,000) Total 460,000 Choice "2" is incorrect. This amount is calculated by incorrectly subtracting the unrecognized transition obligation. Choice "3" is incorrect. This amount is calculated by incorrectly subtracting the unrecognized transition obligation and incorrectly adding the unrecognized net gain. Choice "4" is incorrect. This amount is calculated by adding the unrecognized net gain to the unrecognized prior service cost and transition obligation. The unrecognized net gain should be subtracted.

MCQ-09354 Giant​ Jobs, Inc. amended its *overfunded pension plan* on December​ 31, Year​ 7, resulting in the recognition of prior service cost of​ $700,000. On December​ 31, Year​ 7, Giant​ Job's employees had an average remaining service life of 20 years. The company has an effective tax rate of​ 30%. How will the amortization of the prior service cost affect Gain job's December​ 31, Year 8 financial​ statements under U.S. GAAP? A. $490,000 increase in net periodic pension cost. A. $490,000 decrease in other comprehensive income. B. $700,000 decrease in net income. C. $700,000 increase in pension benefit asset.

A. $490,000 decrease in other comprehensive income. U.S. GAAP requires that a change in the funded status of a pension plan due to the incurrence of prior service cost from a plan amendment be reported in other comprehensive income with related tax effects in the period incurred. *Reclassification adjustment to record amortization of net gain or net transition asset to net periodic pension cost:* *= SIR"AGE" -> OCI -> % age of "AGE" each year is expensed* Dr Other comprehensive income 700,000 Cr Pension benefit asset 700,000 Dr Deferred tax asset (700k x .30) 210,000 Cr Deferred tax benefit—OCI 210,000 The debit to other comprehensive income of $700,000 is offset by the $210,000 credit to deferred tax benefit—OCI, resulting in a net decrease (debit) to other comprehensive income of $490,000 at December 31, Year 7. Note that under IFRS, the prior service cost would be reported on the Year 7 income statement. Choice "1" is incorrect. The prior service cost is not recorded as a component of net periodic pension cost when incurred, but is instead amortized to net periodic pension cost over the average remaining service life of the company's employees. Choice "3" is incorrect. The prior service cost is not recorded on the income statement when incurred, but is instead amortized to net periodic pension cost over the average remaining service life of the company's employees. Additionally, the amortization of prior service cost affects the income statement on an after-tax basis. Choice "4" is incorrect. As demonstrated in the journal entry above, the prior service cost decreases, not increases, the pension benefit asset.

MCQ-09406 On December 30, year 1, Wayne Corporation issued 1,000 of its 8%, 10-year, $1,000 face value bonds with detachable stock purchase warrants at par. Each bond carried a detachable warrant for one share of Wayne's common stock at a specified option price of $25 per share. Immediately after issuance, the market value of the warrants was $120 per warrant.In its December 31, year 1, balance sheet, what amount should Wayne report as the book value of bonds payable? A. $880,000 B. $900,000 C. $1,000,000 D. $1,120,000

A. $880,000 Because the market value of the warrants only is given, the bonds and warrants will be recorded as follows: Dr Cash 1,000,000 Dr Bond discount 120,000 Cr Bonds payable 1,000,000 Cr APIC - Stock warrants 120,000 Therefore, the book value of the bond on the date of issuance is $880,000 ($1,000,000 - 120,000). Calculation: Issue price of bonds and warrants together 1,000,000 Less: Market price of the warrants (120,000) [1,000 * $120] = Issue price of the bonds 880,000 Choice "2" is incorrect. This answer is not supported by the facts presented in this problem. Choice "3" is incorrect. This is the amount of cash received from the issuance of these bonds, not the carrying amount of the bond. See the explanation above. Choice "4" is incorrect. This answer incorrectly adds the bond discount to determine the carrying value of the bond.

MCQ-09696 On June 30 of the current year Huff Corp. issued one thousand of its 8%, $1,000 bonds at 99. The bonds were issued through an underwriter to whom Huff paid bond issue costs of $35,000. On June 30 of the current year, Huff should report the bond liability on the balance sheet at: A. $955,000 B. $965,000 C. $990,000 D. $1,000,000

A. $955,000 Bond issue costs reduce the cash received from the bond issuance and are deducted from the carrying value of the liability. This bond was issued at a discount for $990,000 ($1,000,000 x 99%). The total discount on the bond is $45,000 ($10,000 discount on issuance + $35,000 bond issue costs). The bond issuance is recorded as follows: Dr Cash 955,000 Dr Discount 45,000 Cr Bond liability 1,000,0000 The bond liability reported on the balance sheet is: $1,000,000 bond liability - $45,000 discount = $955,000.

MCQ-09761 *Which of the following statements is most inaccurate in regards to variable interest entities (VIE)?* A. A VIE tends to be self-supportive regardless of its relatively thin equity capitalization. B. The parties who own the majority of voting rights may not exercise significant power over the VIE. C. A significant amount of power to direct the VIE's economic activities will likely lead to consolidation. D. A continuous reassessment is required to assess a VIE's primary beneficiary for consolidation purposes.

A. A VIE tends to be self-supportive regardless of its relatively thin equity capitalization. Choice "1" is correct. This question asks for the inaccurate statement. One of the hallmarks of a VIE classification is that the entity itself is not self-supporting; it requires the financial support of an investor company in order to fund its activities. A thin equity capitalization is often found with VIEs, but that will tend to coincide with a lack of sufficient internal support. Choice "2" is incorrect. This is a true statement, as the majority of voting rights does not necessarily tie to which parties exercise actual power and control over the VIE. Choice "3" is incorrect. A significant amount of power, along with being a primary beneficiary of the VIE, will lead to the requirement for the investor company to consolidate the VIE in its financial statements. Choice "4" is incorrect. This is a true statement, as a VIE's primary beneficiary may change from one period to the next and the investor recognized as the primary beneficiary should be the company that is held accountable for consolidation.

MCQ-08998 *Which of the following costs should a nongovernmental not-for-profit organization report as a supporting service expense?* A. Cost for the annual fund-raising dinner. B. Printing cost incurred to create educational flyers on the prevention of illness. C. Salary paid to a program director. D. Cost incurred to advertise the programs of the organization.

A. Cost for the annual fund-raising dinner. Support services typically involve items such as, - fundraising (i,e., salaries for fundraisers) - administration, - management, - and membership development. Choice "1" is correct. A program service expense is any expense that relates to the activities for which an organization is chartered, while a supporting service expense is any expense not classified as a program service expense. The cost for the annual fundraising dinner and other fundraising expenses would be classified as a supporting service expense. Choice "2" is incorrect. The printing cost incurred to create educational flyers on the prevention of illness would be classified as a program expense rather than as a supporting service expense. The flyers are assumed to directly benefit a program objective (prevention of illness) and do not represent a general promotion or public service announcement in support of the organization. Choice "3" is incorrect. The salary of a program director would be classified as a program expense rather than as a supporting service expense. The efforts of the program director specifically benefit a program. Choice "4" is incorrect. The cost incurred to advertise the programs of the organization would be classified as a program expense rather than as a supporting service expense. The advertisements are assumed to directly benefit programs (program expense) rather than provide general publicity for the organization (support service).

MCQ-09299 *The Township of Thomasville recorded more appropriations than estimated revenues for the coming fiscal year. In integrating its adopted budget with its financial accounting records, the town would:* A. Debit budgetary control. B. Credit budgetary control. C. Debit reserve encumbrances. D. Credit reserve encumbrances.

A. Debit budgetary control. Integration of budgetary controls into the accounting records is accomplished with the following journal entry with the estimated revenues and approved expenditures posted on the opposite side of T-accounts compared to actual amounts and any balancing amount posted to budgetary control: Dr Estimated revenue control XXX Dr Budgetary control (if deficit) XXX Cr Appropriations control XXX Cr Budgetary control (if surplus) XXX Choice "2" is incorrect. A deficit budget would be recorded with a balancing debit entry to budgetary control. Choices "3" and "4" are incorrect. Encumbrance accounts are used to account for the issuance of purchase orders and are debited when the purchase order is issued and credited when the liability is incurred. Encumbrance accounts are not impacted by recording the budget. In addition, the term "reserve" is no longer used for external reporting and would likely not appear in the accounts of a government.

MCQ-07284 The employer net pension liability for the Golf City Fire Department decreased by $200,000 from Year 1 to Year 2 as a result of a change in actuarial assumptions used to value the employer net pension liability. As a result of this change, Golf City will display the following on its Year 2 government-wide financial statements: A. Deferred inflows of resources of $200,000 B. Pension expense of $200,000 C. Deferred outflows of resources of $200,000 D. Note disclosures only

A. Deferred inflows of resources of $200,000 Choice "1" is correct. Changes in a government's pension liability that result from changes in actuarial assumptions are accounted for as deferred outflows and deferred inflows. The reduction in the liability is a deferred inflow presented between liabilities and net position on the statement of net position. Choices "2", "3" and "4" are incorrect, per the explanation above.

MCQ-04674 A not-for-profit voluntary health and welfare organization should report a contribution for the construction of a new building as cash flows from which of the following in the statement of cash flows? A. Financing activities. B. Capital financing activities. C. Investing activities. D. Operating activities.

A. Financing activities. Choice "1" is correct. Cash received with donor-imposed restriction limiting its use to long-term purposes (such as construction of a new building) is displayed as a financing activity on the statement of cash flows of a not-for-profit organization. Choice "2" is incorrect. This is a distracter. There is no classification in the statement of cash flows used for not for profit organizations entitled "Capital financing activities." This classification is used in the statement of cash flows prepared for a proprietary fund of a government. Choice "3" is incorrect. Cash received with donor imposed restrictions limiting its use to long term purposes is a financing activity, not an investing activity, on the statement of cash flows of a not-for-profit organization. Investing activities include sources of cash from the sale of long lived assets or uses of cash associated with the acquisition of long lived assets. Choice "4" is incorrect. Cash received with donor imposed restrictions limiting its use to long term purposes is a financing activity, not an operating activity, on the statement of cash flows of a not-for-profit organization. Operating activities include cash received for purposes without donor restrictions or used in the central ongoing activities of the organization.

MCQ-09425 Which of the following statements regarding the accounting for derivatives is correct? I. A derivative must have one or more underlyings or one or more notional amounts, require little or no net investment, and permit or require a net settlement. II. Derivatives are always reported on the balance sheet as assets at fair value. III. If a derivative is speculative (no hedge designation), changes in fair value are reported on the income statement. IV. If a derivative is used as a fair value hedge, then changes in fair value are reported in other comprehensive income. A. III only. B. I and III C. I, II, III. D. I, II, III, IV.

A. III only. Statement III is the only correct statement. Statement I is incorrect because a derivative must have one or more underlyings and one or more notional amounts. Statement II is incorrect because derivatives can be reported as assets or liabilities. Statement IV is incorrect because changes in the fair value of a fair value hedge are reported on the income statement. Choices "2", "3", and "4" are incorrect, based on the above explanation. Derivatives derive their value from other securities. A derivative must have *all three* of the following characteristics: *1. One or more underlyings (Stock price, commodity price, interest rates), and one or more notional amounts (Dollar amounts used to calculate G/L) or payment provisions (or both);* and 2. No initial net investment (or smaller than would be expected); and 3. Its terms *require or permit a net settlement.*

MCQ-01093 *Frome City signed a 20-year office property lease for its general staff. Frome could terminate the lease at any time after giving one year's notice, but termination is considered a remote possibility. The lease meets the criteria for a lease other than a short-term lease and contract that transfers ownership. What is the effect of the lease on the asset amount in Frome's capital assets and the liability amount in Frome's governmental activities column of the government-wide statement of net position?* Asset amount -- Liability amount A. Increase Increase B. No effect Increase C. No effect No effect D. Increase No effect

A. Increase Increase The lease is neither a short-term lease nor a contract that transfers ownership. It would be recorded as a right-of-use asset and with an associated liability at inception. Dr Right-of-use asset Cr Lease liability Rule: Leases other than short-term leases and contracts that transfer ownership are recognized as a right-of-use asset with related debt for net present value of lease payments. Choice "1" is correct. The lease liability for a 20-year office property recorded as a lease other than short-term lease and contracts that transfer ownership should be reported in the government-wide statement of net position as an increase in assets and liabilities in the governmental activities column. Capital assets and non-current liabilities are not recorded in the governmental funds.

MCQ-09763 Which of the following statements is most accurate for a company with a significant amount of investments categorized within Level 3 for fair value purposes? A. It is more difficult to truly ascertain the fair value of the investment portfolio. B. These investments will outperform a comparable portfolio classified under Level 2. C. Classifications under Level 1 tend to be less accurately quantified than Level 3 investments. D. Less communication with investment managers is required than would be need for Level 1 and 2.

A. It is more difficult to truly ascertain the fair value of the investment portfolio. Level 1 investments are the "easiest" to assign fair values to because they are based on readily observable inputs in active markets. Although Level 1 represents one extreme, Level 3 represents the other extreme. For Level 3 investments, there is no active market and there are no readily observable inputs. As a result, a company will have a more difficult time determining fair value. For a portfolio with a significant amount of investments in Level 3, assessing the overall fair value of the portfolio can be difficult. Choice "2" is incorrect. Investment performance is not linked to the classifications (by level) of the investments themselves. Choice "3" is incorrect. Level 1 investments are more accurate to quantify than Level 3 investments because there are observable quotations (prices) readily available for Level 1 investments. Choice "4" is incorrect. Because Level 3 investments do not trade in active markets and readily observable market inputs do not exist, these investments will require more (rather than less) communication with investment managers to determine appropriate reporting values.

MCQ-01047 Shared revenues received by an enterprise fund of a local government for operating purposes should be recorded in fund financial statements as: A. Non-operating revenues. B. Other financing sources. C. Interfund transfers. D. Operating revenues.

A. Non-operating revenues. Choice "1" is correct. Shared revenues are revenues levied by one government but shared on a predetermined basis with another government. Shared revenues received by a proprietary fund should be recorded as non-operating revenue if received for operations or if received for either operations or capital expenditures at the discretion of the recipient governmental unit. Choice "2" is incorrect. "Other financing sources" is not a proper classification used in the financial statements of enterprise funds. Choice "3" is incorrect. Shared revenues are not classified as interfund transfers. Choice "4" is incorrect. Shared revenues are revenues levied by one government but shared on a predetermined basis with another government. Shared revenues received by proprietary fund types are not classified as operating revenues.

MCQ-01185 Todd City uses an internal service fund to account for the operations of its data processing center, which provides services to Todd's other governmental units. To record the billing for data processing services provided to Todd's other governmental units, the internal service fund should credit: A. Operating revenues. B. Intergovernmental transfers. C. Interfund exchanges. D. Data processing departmental expenses.

A. Operating revenues. Choice "1" is correct. To record the billing for data processing services provided to other governmental units, the internal service fund should credit operating revenues, just like commercial accounting.

MCQ-09326 Lore Co. changed from the cash basis of accounting to the accrual basis of accounting during the current year. The cumulative effect of this change should be reported in Lore's current year financial statements as a: A. Prior period adjustment resulting from the correction of an error. B. Prior period adjustment resulting from the change in accounting principle. C. Component of income from continuing operations. D. Component of non operating gains.

A. Prior period adjustment resulting from the correction of an error. Error Corrections (Restatement Approach) *Error corrections require retroactive restatement by adjusting the beginning balance of retained earnings, net of tax, in the earliest year presented.* If the error occurred in a year presented, the error is corrected in those prior financial statements. *(i.e., Non-GAAP = Error)* Under IFRS, when it is impracticable to determine the cumulative effect of an error, the entity is required to restate information prospectively from the earliest date that is practicable. U.S. GAAP does not have an impracticality exemption for error corrections.

MCQ-00945 *Gains and losses from changes in the fair value of a derivative designated and qualified as a fair value hedge should be:* A. Recognized in current net income in the period in which the fair value of the derivative changes. B. Recognized as a deferred debit or deferred credit in the balance sheet until the derivative is settled. C. Recognized as a component of other comprehensive income in the period in which the fair value of the derivative changes. D. Disregarded until the derivative is settled.

A. Recognized in current net income in the period in which the fair value of the derivative changes. A cash flow hedge hedges an exposure to variability in the cash flows of a recognized asset or a forecasted transaction. *Changes in fair value of the "ineffective portion" of a cash flow hedge are included in income;* changes in fair value of the *"effective portion" of a cash flow hedge are included in the stockholders' equity as part of other comprehensive income (OCI) until the related cash flows are realized. (PUFI"E"R)* Choice "1" is correct. Gains and losses resulting from the change in fair value of derivatives designated and qualified as fair value hedges should be recognized in net income in the period in which the fair value of the derivative changes. Any gain or loss for the period on the hedged item also is recognized in net income. If the hedge is fully effective, the gain or loss on the derivative will exactly offset the loss or gain on the hedged item. Choices "4", "2", and "3" are incorrect, because gains and losses on fair value hedges must be recognized in current income.

MCQ-09758 Churchill Inc. is a public company operating under U.S. GAAP that offers its former employees postretirement benefits. Which of the following statements is most accurate regarding Churchill's nonpension benefits? A. The EPBO for the benefits will be equal to or greater than the APBO as a result of any nonvested benefits. B. Although the assumed health care trend rate is critical to the company's estimates, it is not a required disclosure. C. Any differences between the expected and actual returns must be recognized in the income statement in the period incurred. D. The cost associated with retiree benefits may be accrued before services are rendered, as long as they can be reasonably estimated.

A. The EPBO for the benefits will be equal to or greater than the APBO as a result of any nonvested benefits. The APBO (accumulated postretirement benefit obligation) represents the present value of the liability associated with vested future benefits. The EPBO (expected postretirement benefit obligation) is equal to the APBO, plus the present value of the liability associated with nonvested future benefits. If there are no nonvested benefits, the APBO and EPBO will be equal; if there are nonvested benefits, the EPBO will be greater than the APBO. Choice "2" is incorrect. Churchill is a public company under U.S. GAAP, which means the health care trend rate assumed by the company must be included as a disclosure. Choice "3" is incorrect. If Churchill uses the expected rate of return in order to calculate net postretirement benefit expense/cost, then any differences between the expected return and actual return will result in a gain or loss that can either be booked as an expense in the current period (on the income statement) or booked in other comprehensive income (OCI) and amortized over time. Choice "4" is incorrect. In order to accrue for these costs, they must represent benefits earned as a result of services already rendered.

MCQ-09268 *The Felix Nursing Home Inc. is a health care provider organized as a not-for-profit organization whose activities are regulated by state licensure rules. The financial information that the Felix Nursing Home Inc. is required to produce are,* *Statement of Financial Position:* *Statement of Activity:* *Statement of Cash Flows:* *Functional Expense Disclosure:* A. Yes Yes Yes Yes B. Yes Yes Yes No C. Yes Yes No No D. Yes Yes No Yes

A. Yes Yes Yes Yes Required Financial Statements All not-for-profit entities are required to prepare three basic financial statements on the full accrual basis: √ Statement of Financial Position (Balance sheet) √ Statement of Activities (Income Statement) √ Statement of Cash Flows Disclosure of Functional Expenses (Difference bt NFP & Commercial Accounting) Not-for-profit organizations are required to present disclosure of functional expenses, analyzed by object classification. The objective of the functional expense disclosure is to present the *programmatic and support expenses* displayed horizontally on the statement of activities in separate columns and to analyze the *expenses by object (natural classifications).* Disclosure of expenses by natural classification is done on the statement of activities, or footnotes.

Non-For-Profit Split-Interest Agreements

Agreements such as *charitable remainder* trusts represent donor contributions structured to simultaneously donate assets to the not-for-profit organization and share those assets with a beneficiary. *Split-interest agreements* are *displayed separately* on the not-for-profit organization's financial statements, *measured at their fair value or present value* at acquisition, and classified as donor-restricted. Dr Assets held in trust $XXX Cr Liability to beneficiary $XXX Cr Contribution revenue (with donor restrictions) XXX Disbursements associated with split-interest agreements are *classified as financing activities* on the statement of cash flows. All expenses are classified as without donor restrictions in the statement of activities.

Asset Retirement Obligation (ARO)

An asset retirement obligation (ARO) is a legal obligation associated with the retirement of a tangible long-lived asset that results from the acquisition, construction, development and/or normal operation of a long-lived asset. - An ARO qualifies for recognition when it meets the definition of a liability. Under IFRS, an asset retirement obligation is called a decommissioning liability. A decommissioning liability is initially measured at the best estimate of the expenditure to settle the obligation. U.S. GAAP requires initial measurement at fair value. Measurement of Amounts - An entity records an asset (asset retirement cost—ARC) and a liability (asset retirement obligation—ARO) equal to the fair value of the asset retirement obligation. Dr Asset Retirement Cost (Asset) CR Asset Retirement Obligation (Liability) *- Accretion expense (Like interest exp. on bond) is the increase in the ARO liability due to the passage of time.* *- Depreciation expense (divided by Yrs = depri exp.) decreases the ARC asset reported on the balance sheet.*

Sum-of-Years'-Digits Depreciation

Asset cost of $160,000. The expected useful life of 5 years. Asset sold for $10,000. This means that the total amount of depreciation will be $150,000 spread over its useful life of 5 years. Instead of adding the individual digits in the years of the asset's useful life, the following formula can be used to compute the sum of the digits: n(n+1) divided by 2, where n = the useful life in years. Using this formula for our example, we have: 5(5+1)/2 = 5(6)/2 = 30/2 = 15. [If the formula is used for an asset having a useful life of 10 years, the digits will sum to the following: 10(10+1)/2 = 10(11)/2 = 110/2 = 55. In the first year of this asset's useful life, the depreciation will be 10/55 of the amount to be depreciated. The second year will use 9/55 and the tenth year will use 1/55.]

Cavan Company prepared the following reconciliation between book income and taxable income for the current year ended December 31, Year 1: Pretax accounting income $1,000,000 Taxable income (600,000) Difference $400,000 Differences: Interest on municipal bonds $100,000 Lower financial depreciation 300,000 Total $400,000 Cavan's effective income tax rate for Year 1 is 21%. The depreciation difference will reverse equally over the next three years at enacted rates as follows: Year Tax rate Year 2 30% Year 3 25% Year 4 25% In Cavan's Year 1 Income statement, the deferred portion of its provision for income taxes should be: A. $84,000 B $80,000 C. $100,000 D. $63,000

B $80,000 Remember, for the deferred portion, look for the temporary differences and when they are scheduled to reverse and then take the tax rate for the reversal period into account. In this case, we have $400,000 of differences. However, $100,000 of the differences is interest on municipal bonds, and that is a permanent difference and can be ignored. The $300,000 left is a difference in depreciation, which is a temporary difference. If the expected future tax rates were all the same, the answer would be the $300,000 × 0.21 (the current tax rate) or $63,000. And $63,000 is one of the possible answers. In this case, however, the expected tax rates are not the same, and we have to handle each year differently. Take each temporary difference x future tax rate. Year Difference Tax Rate Deferred Year 2 100,000 x 0.30 = 30,000 Year 3 100,000 x 0.25 = 25,000 Year 4 100,000 x 0.25 25,000 Totals 300,000 NA 80,000

MCQ-09679 Sumrall Corporation owns machinery that was purchased 20 years ago. The machinery, which originally cost $ 2,000,000, has been depreciated using the straight- line method using a 40- year useful life and no salvage value and has a current carrying amount of $ 1,000,000 and a current fair value of $ 800,000. Sumrall estimates that it would incur selling costs of $ 10,000 if it sold the machine and that the present value of the future cash flows from the machine is $ 820,000. Under IFRS, Sumrall should record an impairment loss associated with the machinery of: A. $ 0 B. $ 180,000 C. $ 200,000 D. $ 210,000

B. $ 180,000 *Impairment Test for long-lived assets/Fixed assets (PPE) other than Goowill IFRS* *-Reversal allowed except Goodwill* Step 1: Pick higher of NRV (FV - cost to sell) or Value in use (PV of future CF), this is Recoverable amount (RA) NRV: 800,000 FV - 10,000 cost to sell = 790,000 PV of future CF: 820,000 Step 2: RA - CV = Impairment loss 820,000 RA - 1,000,000 CV = 180,000 Impairment loss

MCQ-09283 *Walton Farms Boys Home Inc. is a not-for-profit organization that received marketable securities from a donor with a fair value of $100,000 on July 1, Year 1. The securities were donated with the stipulation that proceeds would be used to build new dormitories.On January 8, Year 2, Walton Farms Boys Home elected to sell the securities for $110,000 and begin construction on the dormitories.On its Statement of Cash Flows for the year ended December 31,Year 2, the Walton Farms Boys Home would display cash flows from the securities transaction as:* Operating -- Investing -- Financing A. $110,000 $0 $0 B. $0 $110,000 $0 C. $10,000 $100,000 $0 D. $0 $0 $110,000

B. $0 $110,000 $0 *Investing Activities* - Include proceeds from the sale of works of art or purchases of works of art. - Include investment in equipment. - Include *proceeds from the sale of assets that were received in prior periods and whose sale proceeds were donor-restricted to investment in equipment or other LTA.* Disbursements (payment)* of these donor-restricted contributions for either investments or the purpose for which they were intended are classified as investing activities. Proceeds from the sale of assets that were received in prior periods and whose sale proceeds were donor‑restricted to investment in long-lived assets are displayed on the Statement of Cash Flows as cash flows from investing activity. Choice "1" is incorrect. Proceeds from the sale of assets that were received in prior periods and whose sale proceeds were donor-restricted to investment in long-lived assets are displayed on the Statement of Cash Flows as cash flows from investing activity, not an operating activity. Choice "3" is incorrect. Proceeds from the sale of assets that were received in prior periods and whose sale proceeds were donor-restricted to investment in long-lived assets are displayed on the Statement of Cash Flows as cash flows from investing activity, gains are not segregated as an operating activity. Choice "4" is incorrect. Proceeds from the sale of assets that were received in prior periods and whose sale proceeds were donor-restricted to investment in long-lived assets are displayed on the Statement of Cash Flows as cash flows from investing activity, not a financing activity.

*On January 1, Year 1, Sunshine Corporation acquired an 80% ownership interest in Grey Sky Enterprise by purchasing 400,000 of Grey Sky's 500,000 voting common shares outstanding for $10,000,000. Additional information regarding Grey Sky as of January 1, Year 1, follows:* *Book Value Fair Value* *Net Assets $11,200,000 $11,500,000* *On the acquisition date, what is the goodwill that will be reported on the consolidated balance sheet of Sunshine Corporation under U.S. GAAP?* A. $800,000 B. $1,000,000 C. $1,040,000 D. $1,500,000

B. $1,000,000 The difference between the fair value of the subsidiary and the book value of the subsidiary net assets should be allocated as follows (BIG): B 1. Balance sheet adjustment of the *subsidiary's assets and liabilities from book value to fair value. I 2. Identifiable intangible assets recorded at fair value. G 3. Goodwill is excess. Under U.S. GAAP, goodwill is calculated as follows (full goodwill method): *Goodwill = 100% Fair value of subsidiary - 100% Fair value of subsidiary's net assets* The fair value of the subsidiary is calculated as follows: FV subsidiary x 80% = $10,000,000 10,000,000 / 0.80 = $12,500,000 100% FV subsidiary Therefore, goodwill is: Goodwill = $12,500,000 - $11,500,000 = $1,000,000 IFRS permits the use of the full goodwill method or the partial goodwill method. Under the partial goodwill method, goodwill is calculated as follows: *Goodwill = % Acquisition cost - % Fair value of subsidiary's net assets acquired* Goodwill recognized in a business combination is not amortized.* Instead, it is tested for impairment, and a loss is recognized in income from continuing operations if the goodwill is impaired.

MCQ-09302 Posteriori Company. had a credit translation (not remeasurement) adjustment of $30,000 for the year ended December 31, year 2. The functional currency of Posteriori's subsidiary is the currency of the country in which it is located. Additionally, Posteriori had a receivable from a foreign customer payable in the local currency of the customer. On December 31, year 1, this receivable for 200,000 local currency units (LCU) was correctly included in Post's balance sheet at $110,000. When the receivable was collected on February 15, year 2, the United States dollar equivalent was $120,000. In Posteriori's year 2 consolidated income statement, how much should be reported as foreign exchange transaction gain? A. $0 B. $10,000 C. $30,000 D. $40,000

B. $10,000 *Gains and losses from foreign currency transactions and foreign currency remeasurement are reported on the income statement. Gains and losses from foreign currency translation are reported in other comprehensive income.* In this case, there is a translation adjustment of $30,000. Just ignore that amount since the question is asking about what is included in earnings. There is also a receivable denominated in a foreign currency. The amount of that receivable was $110,000 at the beginning of the year and was $120,000 at the time it was collected in 20X1, the year we are asked about. The gain was $10,000 ($120,000 - $110,000) and it would be a transaction gain and included in earnings.

MCQ-00865 The December 31, Year 1 condensed balance sheet of Mason & Gross, a partnership, follows: Current assets $125,000 Equipment (net) 15,000 Total assets $140,000 Liabilities 10,000 Mason, Capital 80,000 Gross, Capital 50,000 Total liabilities and capital $140,000 Market values at December 31, Year 1 are as follows: Current assets $90,000 Equipment 30,000 Liabilities 10,000 On January 2, Year 2, the partnership was incorporated and 1,000 shares of $5 par value common stock were issued. What amount should be credited to additional contributed capital? A. $0 B. $105,000 C. $135,000 D. $125,000

B. $105,000 Rule: Assets contributed by a partnership (or sole proprietorship) to a corporation in its formation are valued at the assets *fair market value, less any related liabilities assumed* by the corporation (e.g., mortgage note on real property). Stock issued is credited at par value and any difference is credited to additional paid-in capital. Fair market value of assets contributed 120,000 ($90,000 + 30,000) Less liabilities assumed (10,000) Net assets 110,000 Common stock (1,000 shares @ $5 par) 5,000 Additional paid-in capital squeeze 105,000 Shareholders' equity 110,000

MCQ-09432 *On January 1, Year 1, Black Dog Corp. began operations and issued 30,000 shares of $5 par common stock for $9/share. On June 30, the company bought back 10,000 shares of $8/share. Then, On September 15, the company resold 5,000 shares for $12/share. What amount of total additional paid-in capital should Black Dog report on its December 31, Year 1 balance sheet if Black Dog uses the par value method to account for its treasury stock?* A. $10,000 B. $125,000 C. $120,000 D. $165,000

B. $125,000 Under the par value method, Black Dog would record the following journal entries for its Year 1 stock transactions: *Par Value Method - G/L calculated upon repurchase* Jan. 1, Year 1—Issue 30,000 shares of $5 par common stock for $9/share: $9 SP - $4 Cost = $4 Gain increases Equity Dr Cash (30,000 x $9) 270,000 Cr Common stock (30,000 x $5 par) 150,000 Cr APIC—CS (30,000 x $4 gain) 120,000 June 30, Year 1—Repurchase 10,000 shares for $8/share: Dr Treasury stock (10,000 x $5 par) 50,000 Dr APIC—CS (10,000 x $4 gain) 40,000 Reverse original entry using $4 gain Cr Cash 80,000 Cr APIC—TS (plug) 10,000 *The 10,000 APIC-TS (Gain) increases SE and is used to absorb future losses.* Sept. 15, Year 1—Resell 5,000 shares for $12/share: Dr Cash (10,000 x $12 SP) 60,000 Cr Treasury stock (5,000 x $5 par) 25,000 Cr APIC—CS 35,000 Therefore, total APIC on Black Dog's balance sheet at Dec. 31, Year 1 would be $125,000 ($120,000 from original issuance − $40,000 from repurchase using par method + $10,000 gain on repurchase + $35,000 from resale). Choice "1" is incorrect. *This is the amount of APIC—treasury stock that would be reported on the Dec. 31, Year 1 balance sheet. The question asks for total APIC, which would include APIC—common stock and APIC—treasury stock. (Know this also)* Choice "3" is incorrect. This is the amount of APIC—common stock recorded as a result of the original issuance of common stock on Jan. 1, Year 1. Choice "4" is incorrect. This is the amount of total APIC that would be reported on the Dec. 31, Year 1 balance sheet under the cost method.

MCQ-09376 Sell2All Inc. accounts for its revenue under the installment sales method. In Year 1, Sell2All sold inventory with a cost of $300,000 for $400,000 and collected $100,000. In Year 2, Sell2All sold inventory with a cost of $500,000 for $750,000 and collected $400,000, including $100,000 related to the Year 1 sales and $300,000 related to the Year 2 sales. What amount of earned gross profit should Sell2All report on its December 31, Year 2, balance sheet? A. $100,000 B. $125,000 C. $133,200 D. $400,000

B. $125,000 When the installment method is used, gross profit is recognized when cash is collected using the formula: *Earned gross profit = Cash collections x Gross profit %* In Year 2, Sell2All received cash collections related to both Year 1 and Year 2 sales, so it is necessary to calculate the gross profit percentages for both years: Year 1 gross profit percentage = ($400 - $300)/$400 = 25% Year 2 gross profit percentage = ($750 - $500)/$750 = 33.3% Therefore: Year 2 earned gross profit = ($100,000 x 25%) + ($300,000 x 33.3%) = $125,000 Choice "1" is incorrect. This is the amount of gross profit to be recognized on the sales made in Year 2 ($300,000 cash received from Year 2 sales x 33.3% = $100,000). This amount incorrectly excludes the gross profit to be recorded for the $100,000 cash received in Year 2 related to Year 1 sales. Choice "3" is incorrect. *This answer applies the Year 2 gross profit percentage of 33.3% to the entire $400,000 in cash collected in Year 2. Of the $400,000 collected in Year 2, only $100,000 is payment on Year 1 sales. This $100,000 must be multiplied by the Year 1 gross profit percentage to determine the gross profit earned in Year 2 on sales made in Year 1. I'm assuming the 100,000 from the year 1 sale is only restated twice once when it's collected and the other that it's already collected the 100,000 to confuse you if you don't read carefully.* Choice "4" is incorrect. This is the amount of cash collected in Year 2, which must be multiplied by the appropriate gross profit percentages to determine earned gross profit.

MCQ-09291 On November 1, Year 1, Dixon Corporation issued $800,000 of its 10-year, 8% term bonds dated October 1, Year 1. The bonds were sold to yield 10%, with total proceeds of $700,000 plus accrued interest. Interest paid every April 1 and October 1. What amount should Dixon report for interest payable in its December 31, Year 1 balance sheet? A. $17,500 B. $16,000 C. $11,667 D. $10,667

B. $16,000 Issuance of Bonds Between Interest Dates When bonds are issued between interest dates, the amount of interest that has accrued since the last interest payment is added to the price of the bonds and is reimbursed at the next interest payment date to the purchaser. (The purchaser gets the full interest payment regardless of how long he/she has held the bond.) Dr Cash $716,000 Dr Discount on B/P $XXX Cr Bonds payable $XXX Cr Interest expense (or payable) $XXX In this question, they want to know the amount of interest payable at the end of the year. Going back to the facts, Dixon issued $800,000 of 8% bonds dated 10/01/Year 1. One month's worth of interest would be $5,333 ($800,000 × 0.08 × 1/12), and three months' worth of interest would be $16,000.

MCQ-12700 *The pretax financial income and taxable income of Zeus Corporation were the same for the following years (i.e., there were no permanent or temporary differences):* Income -- Tax Rate 2017 $20,000 30% 2018 15,000 25% 2019 (100,000) 21% 2020 70,000 21% 2021 50,000 18% *Assume that Zeus elects to not use the five-year carryback. What amount of income tax benefit will Zeus Corporation record in 2019 under U.S. GAAP?* A. $21,000 B. $20,100 C. $19,680 D. $23,400

B. $20,100 The $100,000 2019 NOL will be carried forward to offset 2020 and 2021 taxable income, subject to an 80 percent of taxable income limitation in 2021, without regard to the NOL deduction. The deferred tax asset can be calculated using 2020 and 2021 tax rates: 2020: $70,000 × 21% = 14,700 2021: $30,000 × 18% = 5,400 Total deferred tax asset 20,100 For 2020, the NOL deduction is $70,000, which is the lesser of the NOL carryforward of $100,000, and the NOL deduction limitation of $70,000 (taxable income prior to NOL). For 2021, the NOL deduction is $30,000, which is the lesser of the NOL carryforward of $30,000 ($100,000 − $70,000), and the NOL deduction limitation of $40,000 ($50,000 taxable income prior to NOL × 80% limitation = $40,000). Zeus's 2019 income tax journal entry will be: Dr Deferred tax asset 20,100 Income tax benefit 20,100 Choice "1" is incorrect. This is the $100,000 NOL multiplied by the 2019 tax rate. This is not the method used to calculate the income tax benefit from an NOL. Choice "3" is incorrect. This answer incorrectly applies the 80 percent of taxable income deductible limitation to both 2020 and 2021, instead of just 2021. Choice "4" is incorrect. This answer incorrectly carries back the NOL to 2017 and 2018 and also ignores the 80 percent of taxable income limitation in 2020.

MCQ-09269 Property taxes for the Town of Farrell of $25,000,000 were assessedin October of Year 1 to fund budgeted operations for the fiscal year ended September 30, Year 2. Some $24,000,000 are collected from November Year 1 through March Year 2 with liens of $1,000,000 applied to properties with unpaid property tax bills in May Year 2. Properties subject to delinquent property taxes were auctioned for taxes of $800,000. At September 30, Year 2, government would record property tax revenue of: A. $25,000,000 B. $24,800,000 C. $24,200,000 D. $24,000,000

B. $24,800,000 Property tax revenue is an imposed non-exchange revenue that would be recorded when levied to the extent that it is both measurable and available. The fact pattern indicates that the following revenues were recognized: Tax revenue collected 24,000,000 Delinquent taxes collected 800,000 = Total tax revenue available 24,800,000 Uncollected revenue 200,000 = Total tax levy 25,000,000 General Rule: Property taxes generally meet the definition of non-exchange revenues and are recognized in the period in which an enforceable legal claim to the taxes arises and the revenues are available. Availability is defined as collection within the period levied or soon enough to pay liabilities of the current period (60 days after year-end). Governments may refer to the date the enforceable legal claim arises as either the lien date or the assessment date. Property taxes received in advance are displayed as deferred inflows of resources.

MCQ-09260 On December 31, Year 1, Eve Company leased a machine under a finance lease for a period of 10 years, contracting to pay $50,000 on signing the lease and $50,000 annually on December 31 of each of the next nine years. The present value at December 31, Year 1 of the 10 lease payments discounted at 10% was $338,000. At December 31, Year 2, Eve's total finance lease liability is: A. $303,980 B. $266,800 C. $259,200 D. $243,000

B. $266,800 Cash pmt -- Int exp -- Principal -- CV NA NA NA 288,000 50,000 28,800 21,200 266,800 In this question, they want to know the finance lease liability. They are paying $50,000 every year for the next 10 years, and they provide the present value of the minimum lease payments of the lease as of December 31, Year 1. Watch the dates; they are there for a reason. The first payment is made on signing the lease, i.e., at the beginning of the period. The liability after that payment is $288,000 ($338,000 − $50,000). The question asks what the long-term liability is one year later. $288,000 × 10% is $28,800. That amount is the interest component of the second payment. The difference between the $50,000 payment and the $28,800 is $21,200, and the $21,200 is the reduction of the long-term liability of the second payment; therefore the liability after the second payment is $266,800 ($288,000 − $21,200). It all started with the first payment. All the other answers have a wrong turn.

The City of Lawrence's United Way received a donation of a vehicle from a concerned citizen on January 1, Year 1, who directed that the vehicle was to be donated to the Lawrence Day Care Center, a not-for-profit organization, and went on to stipulate that the Lawrence United Way could use the vehicle for a period of one year prior to the transfer. The vehicle has a fair value of $9,000 and a remaining useful life of three years. The Lawrence Day Care is a United Way organization but does not have the ability to influence United Way Policy. The United Way elects to use the vehicle. On their December 31, Year 1 financial statements, each organization should display the following Net Asset changes based on this transaction: I. Lawrence United Way A) With Donor Restrictions: B) Without Donor Restrictions: II. Lawrence Day Care A) With Donor Restrictions: B) Without Donor Restrictions: I. A) B) II. A) B) A. $0 $0 $0 $0 B. $3,000 $0 $0 $6,000 C. $0 $9,000 $0 $0 D. $3,000 $6,000 $0 $9,000

B. $3,000 $0 $0 $6,000 The donation of a vehicle, including the use of a vehicle, is considered the donation of property, not service. The asset donation is recognized as an increase in net assets at its fair market value for the amount of the value received (by the United Way $3,000) or receivable (by the Day Care $6,000). The receivable by the Day Care has an implied temporary (time) restriction since the vehicle will not be transferred for one year. Choices "1" and "3" are incorrect. The asset donation is recognized as an increase in net assets at its fair market value for the amount of the value received. Choice "4" is incorrect. The recipient organization would not recognize revenue with donor restrictions related to its liability to the beneficiary.

MCQ-09742 Dig Inc. has several natural gas containment systems that must be disposed of in four year at an estimated cost of $200,000. The company plans to book an asset retirement obligation using straight-line depreciation and a 7 percent accretion rate. Accretion Depreciation A. $47,421 $200,000 B. $47,421 $152,579 C. $152,579 $47,421 D. $200,000 $47,421

B. $47,421 $152,579 Accretion is the increase in the ARO over time. At the end of the four years, the cumulative accretion and accumulated depreciation should combine to equal the asset retirement obligation (ARO) of $200,000. The present value of a four-year obligation at a 7 percent rate and a future value of $200,000 is equal to $152,579. This asset must be fully depreciated over four years, while the liability must accrue from $152,579 to $200,000, a difference of $47,421. (The present value can be calculated this way: $200,000 / 1.074. This is equal to $200,000 / 1.3108. Or, if you were given a present value table, you would look up the factor for the present value of $1, 4 periods, 7 percent to obtain 0.7629. Multiply this factor by $200,000 to obtain $152,579.) Choice "1" is incorrect. The accretion is correct, but the asset will be booked at a present value of $152,579 and this becomes the amount that will ultimately be depreciated over four years. Choice "3" is incorrect. The combined total is correct, but the amounts in each column are reversed. Choice "4" is incorrect. The combined total must equal the ultimate liability of $200,000.

MCQ-09361 *Big Wheel Construction had the following two construction projects, which began in Year 1:* *Project 1 -- Project 2* *Contract price $690,000 $425,000* *Costs incurred during Year 1 200,000 225,000* *Estimated costs to complete 400,000 225,000* *Billed to customers during Year 1 230,000 175,000* *Received from customers during Year 1 100,000 150,000* *What amount of gross profit (loss) would Big Wheel report on its Year 1 income statement?* *Percentage of Completion -- Completed Contract* A. $5,000 $(12,500) B. $5,000 $(25,000) C. $17,500 $(12,500) D. $17,500 $(25,000)

B. $5,000 $(25,000) *Calculation of Current Gross Profit* *Step 1* *Contract price - Total estimated cost = Total gross profit* *Step 2* *Cost to date / Total estimated cost = % Completed* *Step 3* *Step 1 × Step 2 = GP earned to date* *Step 4* *GP earned to date - GP previously recognized = Current GP* Year 1 gross profit (loss) is calculated as follows: Project 1 -- Project 2 Contact price 690,000 425,000 Estimated total cost (600,000) (450,000) = Gross profit 90,000 (25,000) Under the completed contract method, only the Project 2 estimated total loss of $25,000 will be recognized in Year 1. The Project 1 gain will not be recognized until the project is complete. Under the percentage of completion method, the Year 1 gain is calculated as follows: Year 1 Percent complete = Cost to date / Total estimated cost: Project 1 = $200,000 / 600,000 = 33.3% *Project 2 = Irrelevant—losses are recognized in total when incurred* Gross profit earned to date = (GP × % complete) − GP recognized in prior periods: Project 1 = ($90,000 × 33.3%) − $0 = 30,000 Project 2 = [$(25,000) × 100%] − $0 = (25,000) = 5,000 Choice "1" is incorrect. The gross loss under the completed contract method is incorrectly calculated based on the fact that Project 2 is only 50% complete. Under both the completed contract and percentage of completion methods, losses are recognized in full, whether or not the project is complete. Choice "3" is incorrect. The gross profit (loss) under both the percentage of completion and completed contract methods are incorrectly calculated based on the fact that Project 2 is only 50% complete. Under both the completed contract and percentage of completion methods, losses are recognized in full, whether or not the project is complete. Choice "4" is incorrect. The gross profit (loss) under the percentage of completion method is incorrectly calculated based on the fact that Project 2 is only 50% complete. Under both the completed contract and percentage of completion methods, losses are recognized in full, whether or not the project is complete.

MCQ-09674 On December 31, Star Corp. had a reporting unit that had a book value of $950,000, including goodwill of $130,000. As part of the company's annual review of goodwill impairment, Star determined that the fair value of the reporting unit was $890,000. Star assigned $840,000 of the reporting units fair value to its assets and liabilities other than goodwill. What is the impairment loss to be reported on December 31 under US GAAP? A. $50,000 B. $60,000 C. $80,000 D. $110,000

B. $60,000 US GAAP Goodwill Impairment One step approach: 950,000 CV reporting unit including GW < $890,000FV including GW *950,000 CV reporting including GW - 890,000 FV including GW = 60,000 Impairment loss* *The max charge is the amount of 130,000 CV of goodwill.*

MCQ-09379 Two independent situations are described below. Each situation has future deductible amounts and/or future taxable amounts produced by temporary differences: Situation 1 -- 2 Taxable income $40,000 $80,000 Amounts at year-end: Future deductible amounts 5,000 10,000 Future taxable amounts 0 5,000 Balances at beginning of year: Deferred tax asset 1,000 4,000 Deferred tax liability 0 1,000 The enacted tax rate is 21% for both situations. Determine the income tax expense for the year. Situation 1 Situation 2 A. $8,400 $16,800 B. $8,350 $18,750 C. $8,450 $14,850 D. $7,350 $15,750

B. $8,350 $18,750 Current income tax expense/benefit is equal to the taxable income for the current year, multiplied by the current tax rate. Situation 1 TI: $40,000 x .21 = 8,400 Income tax expense EOY Future deductible amounts: 5,000 = 5,000 x 0.21 = 1,050 BOY DTA: 1,000 (has already been multiplied by tax rate) BOY DTA 1,000 *+ 50 (plug)* EOY 1,050 DTA A DTA lowers your future tax expense. Since the DTA increased, this causes an additional 50 of savings for this Yr thus lowering Income tax expense to $8,350 (8,400 - 50). Situation 2 TI: $80,000 x .21 = 16,800 Income tax expense EOY Future deductible amounts: 10,000 EOY Future taxable amounts: 5,000 10,000 - 5,000 = 5,000 = 5,000 x 0.21 = 1,050 EOY DTA BOY DTA: 4,000 (has already been multiplied by tax rate) BOY DTL: 1,000 (has already been multiplied by tax rate) 4,000 - 1,000 = 3,000 BOY DTA BOY DTA 3,000 *- 1,950 (plug)* EOY 1,050 DTA A DTA lowers your future tax expense. Since the DTA decreased at EOY this increases tax expense by 1,950 for the current Yr to $18,750 (16,800 + 1,950).

MCQ-12700 *The pretax financial income and taxable income of Zeus Corporation were the same for the following years (i.e., there were no permanent or temporary differences):* Income -- Tax Rate 2018 $20,000 25% 2019 15,000 21% 2020 (100,000) 21% 2021 50,000 21% *Assume that Zeus elects to not use the five-year carryback. What amount of income tax benefit will Zeus Corporation record in 2020 under U.S. GAAP, assuming that it is more likely than not that there will be no taxable earning after 2021?* A. $10,500 B. $8,400 C. $12,600 D. $21,000

B. $8,400 $100,000 of the 2020 NOL is carried forward, resulting in a deferred tax asset of $21,000 ($100,000 × 21% = $21,000). However, because the use of an NOL is limited to 80 percent of taxable income in 2021 (without regard to the NOL deduction) and there will be no taxable income after 2021, a valuation allowance must be recorded to offset a portion of the deferred tax asset. The valuation allowance is calculated as follows: NOL carryforward 100,000 Less: 2021 income (50,000 × 80% limitation) (40,000) Unusable carryforward 60,000 Tax rate × 21% DTA valuation allowance 12,600 Zeus's 2020 income tax journal entry will be: Dr Deferred tax asset 21,000 Cr Income tax benefit 8,400 Cr Valuation allowance 12,600 Choice "1" is incorrect. This answer ignores the 80 percent of taxable income limitation for the NOL deduction.

MCQ-09429 Far Out Producers is involved in two product liability lawsuits anda third lawsuit that the company brought against a competitor for patent infringement. At December 31, Year 1, the company's attorneys informed management of the following: - It is probable that Far Out will lose one of the product liability lawsuits, although the actual settlement could be as low as $800,000 and as high as $2,000,000. - It is possible that the company could lose $1,000,000 in the second product liability lawsuit. - It is probable that Far Out will win $500,000 in the patent infringement case. What should Far Out report on its December 31, Year 1 balance sheet for these contingencies? A. $300,000 contingent liability. B. $800,000 contingent liability. C. $1,800,000 contingent liability. D. $3,000,000 contingent liability.

B. $800,000 contingent liability. Only the probable loss from the first product liability lawsuit would be recorded on the balance sheet. Because a range of losses is given, the lowest amount in the range is recorded on the balance sheet as a contingent liability and the range is disclosed in the footnotes along with the details of the case. The possible loss from the second product liability lawsuit and the probable contingent gain are not recorded on the balance sheet, but instead are disclosed in the footnotes. Choice "1" is incorrect. This answer incorrectly offsets the probable gain of $500,000 against the probable loss of $800,000. Probable gains cannot be accrued on the balance sheet, but can only be disclosed in the footnotes. Choice "3" is incorrect. This answer incorrectly accrues the $1,000,000 possible loss on the second product liability lawsuit. A possible loss should not be accrued on the balance sheet, but should instead be disclosed in the footnotes. Choice "4" is incorrect. This answer incorrectly accrues the $1,000,000 possible loss on the second product liability lawsuit. A possible loss should not be accrued on the balance sheet, but should instead be disclosed in the footnotes. Additionally, this answer accrues the $2,000,000 high end of the range of probable losses on the first product liability lawsuit. Generally, the low end of the range is accrued.

MCQ-09414 Pico Corp. owns 100% of Sepulveda Inc.'s common stock. DuringYear 1, Pico sold inventory to Sepulveda for $400,000, which was 25% above cost. Sepulveda sold half of the inventory purchased from Pico in Year 1. At year-end, before consolidation, Pico reported inventory of $390,000 and Sepulveda reported inventory of $480,000. All other inventory transactions for both companies were with outside customers and suppliers. At December 31, Year 1, what should Pico Corp. report as consolidated inventory on its balance sheet? A. $670,000 B. $830,000 C. $870,000 D. $910,000

B. $830,000 Pico-record the sale to Sepulveda JE: Dr Account receivable $400,000 Cr Sales revenue $400,000 Dr COGS (400,000/1.25) 320,000 Cr Inventory 320,000 Sepulveda records the purchase form Pico JE: Dr Inventory 400,000 Cr Cash 400,000 Because this is an intercompany transaction, the intercompany sales revenue, cost of goods sold and profit on inventory must be eliminated. In this problem, the intercompany profit on inventory is $80,000 ($400,000 cost of inventory on Sepulveda's books − $320,000 original cost of inventory on Pico's books). Because Sepulveda has sold half of the inventory purchased from Pico, half of the intercompany profit must be eliminated from cost of goods sold and half of the intercompany profit must be eliminated from ending inventory. The journal entry to record this elimination is as follows: Dr Sales revenue 400,000 Cr Cost of goods sold 360,000 Cr Inventory 40,000 *Therefore, at year-end, consolidated inventory is equal to Pico's inventory + Sepulveda's inventory − Eliminated intercompany profit = $390,000 + 480,000 − 40,000 = $830,000.* Choice "1" is incorrect. This answer option incorrectly calculates ending inventory as follows: Sepulveda [$480,000 − (50% × $400,000)] 280,000 Pico 390,000 Total = 670,000 Choice "3" is incorrect. This is the sum of the inventory amounts on each of the balance sheets without the elimination entry for the intercompany profit of $40,000 [$390,000 + $480,000 = $870,000]. Choice "4" is incorrect. This answer option incorrectly adds (rather than subtracts) the intercompany profit on the inventory of $40,000 to the ending balance sheet amounts of inventory for each [$390,000 + $480,000 + $40,000].

MCQ-09333 Port, Inc. owns 100% of Salem Inc. On January 1, Year 1, Port sold Salem delivery equipment at a gain. Port had owned the equipment for two years and used a five-year straight-line depreciation rate with no residual value. Salem is using a three-year straight-line depreciation rate with no residual value for the equipment. In the consolidated income statement, Salem's recorded depreciation expense on the equipment for Year 1 will be decreased by: A. 20% of the gain on sale. B. 33 1/3% of the gain on sale. C. 50% of the gain on sale. D. 100% of the gain on sale.

B. 33 1/3% of the gain on sale. Depreciation expense will be decreased by 33 1/3% of the gain on sale, the amount that depreciation expense has been overstated. Example: Original purchase price by Port 100 two years' depreciation ($100 / 5 = $20 per year x 2) (40) Net book value at date of sale 60 Sale price to Salem 75 Gain on sale 15 Depreciation expense recorded by Salem 25 ($75 / 3 year life) Consolidated depreciation expense 20 ($100 / 5 year life) Elimination of excess depreciation 5 ($15 gain × 1/3) Excess depreciation is calculated by 25 - 20 = 5. *Take the 5 of excess depreciation and divide it by the gain on sale of 15 in order to calculate the 1/3.*

MCQ-09373 Darius Doogood promised to contribute $500,000 to the Daberville Day Care, a not-for-profit corporation organized to provide child care services to disadvantaged families, to construct a satellite facility. Doogood further stipulates in his bequest that the Daberville Day Care must secure operating funds from the Federal Headstart program and ensure that municipal bus routes are expanded to include the location to be used for the day care prior to the beginning of construction. Darberville requests, and receives, $50,000 from Dogwood as evidence of his good faith to fund the construction and to demonstrate community support as part of its Federal grant application. As a result of the above transactions, the Darberville Day Care would record: A. An increase in revenue with donor restrictions of $500,000. B. A refundable advance of $50,000. C. An increase in net assets with donor restrictions of $50,000 and conditional contributions of $450,000. D. Conditional contributions with donor restrictions of $500,000.

B. A refundable advance of $50,000. Conditional promises to give are not recorded as revenue. Good faith deposits are recorded as a liability (a refundable advance) and are not recorded as revenue. Doogood's $50,000 is a refundable advance. Doogood's conditional promises would not result in any accounting entry. Choice "1" is incorrect. Good faith deposits do not increase revenue, they increase liabilities. Choice "3" is incorrect. Good faith deposits do not increase revenue, they increase liabilities. Conditional contributions are not recorded as revenue. Choice "4" is incorrect. Conditional contributions are not recorded as revenue.

MCQ-09255 Lennon Inc. is the lessor on a 10-year (120-month) operating lease on warehouse space at $1,500 per month. Lennon is offering the lessee the first four months free and the next eight months after that at half off ($750 per month), with the remaining nine years at the full rate. After the first year of the lease, the monthly journal entry will show each of the following entries except, a: A. Debit to cash of $1,500. B. Debit to depreciation of $1,500. C. Credit to rent receivable of $100. D. Credit to rental income of $1,400.

B. Debit to depreciation of $1,500. Fist 4 months rent: 0 Next eight months at half off: $750x8= $6,000 Remaining 9yrs at full rate: [$1,500 x (9x12)]= $162,000 162,000 + 6,000 = 168,000 Total rent 168,000 / 120months = 1,400 rent per month The journal entry for each of the first four months is: Dr Rent receivable $ 1,400 Cr Rental income$ 1,400 Total rental income must be pro-rated equally to show the promotion over 120 months which is why the lessor still recognizes 1,400 as rental income. The lessor allowed the first four-months to be free and cannot record any cash received, thus must record a receivable of 1,400. The journal entry for each of the next eight months is: Dr Cash $750 Cr Rent receivable 650 Cr Rental income $1,400 Now the lessor paid 750 in cash for 8 months and the remaining balance goes to rent receivable. The total cumulative balance in rent receivables is equal to: ($1,400 × 4) + ($650 × 8) = $10,800. The journal entry that Lennon will book every month after the first year of the lease is: Dr Cash $1,500 Cr Rent receivable $100 Cr Rental income 1,400 *The following year when rent returns to normal at $1,500 you will begin reducing the excess in your A/R with excess $100 of cash received.* *Depreciation will be booked every month for the warehouse asset itself, but we don't have enough information to determine the amount.*

MCQ-09747 The Richards Corporation has debt of $900,000 from a 6 percent, four-year note dated June 30, Year 2. Interest is payable quarterly, and as a result of financial difficulties, Richards has accrued interest of $40,500 as of June 30, Year 4. A troubled debt restructuring allows Richards to settle the note (and accrued interest) in exchange for two office buildings with a fair value $730,000 and a carrying value of $740,000. For the restructuring portion alone, Richards will book a: A. Gain of $200,500. B. Gain of $210,500. C. Loss of $200,500. D. Loss of $210,500.

B. Gain of $210,500. Choice "2" is correct. The total gain overall for Richards is equal to the difference between the debt forgiven and the carrying value of the asset given up. $940,500 is the total debt forgiven (principal and accrued interest) and $740,000 is the carrying value of the office buildings, which nets a total gain of $200,500. This gain must be allocated between the gain/loss on the buildings themselves and the gain/loss on the restructuring. For the buildings, the difference between the fair value ($730,000) and the carrying value ($740,000) results in a holding loss of $10,000. The restructuring gain is equal to the difference between the debt forgiven ($940,500) and the fair value of the office buildings ($730,000), or $210,500. Accounting by the Debtor Assets marked to market (to FV) through IS. - Recognize gain/loss on: 940,500 FV Asset Transferred < 730,000 NBV Asset Transferred > = $210,500 Gain / Loss The journal entry is shown below: Dr Notes payable 900,000 Dr Interest payable 40,500 Dr Loss on buildings 10,000 Cr Buildings 740,000 Cr Gain on restructuring 210,500 Choice "1" is incorrect. The total gain is $200,500, which must be further allocated between the buildings themselves and the restructuring portion. Choice "3" is incorrect. The company has a gain overall, although the building portion actually represents a loss because the fair value is below the carrying value. Choice "4" is incorrect. The amount is correct, but it is a gain rather than a loss.

MCQ-09403 Which of the following statements regarding the process of foreign currency translation is/are correct? I. After the translation process is complete, a subsidiary's financial statements must be adjusted to conform to U.S. GAAP. II. The process of translation is done to convert financial statements from a functional currency to the reporting currency. III. The process of remeasurement is done when the local currency of a subsidiary is different than the functional currency. IV. Translation gains and losses are reported on the income statement, while gains and losses from remeasurement are reported in other comprehensive income. A. I and IV. B. II and III. C. I, II and III. D. II, III, and IV.

B. II and III. Remeasurement: G/L goes on IS Local (foreign) -> Functional Translation: G/L - PU*F*IER (OCI shows up in equity BS) Functional -> Reporting I. *"Before"* the translation process is complete, a subsidiary's financial statements must be adjusted to conform to U.S. GAAP. IV. Translation gains and losses are reported on the balance sheet in the equity section under OCI (PU"F"IER), while gains and losses from remeasurement are reported in Income statementent.

Which of the following statements regarding the acquisition method is/are correct? I. An acquisition method business combination represents a legal combination of the parent company and the subsidiary company. II. In an acquisition method term-174business combination, the subsidiary can be acquired using any agreed upon form of consideration. III. Financial statements consolidated using the acquisition method do no show the equity of the subsidiary company. IV. In an acquisition method business combination, the financial statements of the parent and subsidiary are consolidated as of the beginning of the year of acquisition. A. I,IV. B. II, III. C. I, II, III. D. I, II, III, IV.

B. II, III. Acquisition Method In a business combination accounted for as an acquisition, the subsidiary may be acquired for cash, stock, debt securities, etc. The investment is valued at the fair value of the consideration given or the fair value of the consideration received, whichever is the more clearly evident. The accounting for an acquisition begins at the date of acquisition. Both Statements II and III are correct statements. Statement I is incorrect because an acquisition method business combination reflects economic reality, but not a legal combination of the parent and subsidiary. Statement IV is incorrect because, in the year of acquisition, the acquisition method consolidation is done starting on the date of acquisition. Choices "1", "3", and "4" are incorrect, based on the above explanation.

MCQ-09358 *Andrew contributed $250,000 to an endowment in the name of his late father, Phillip, at Ray Private University on January 2, Year 1. The terms of the bequest require that principal be retained intact and that the earnings from the principal be used to fund the Phil Anthorpist chair for accountancy. If Ray Private University earns $15,000 during Year 1 from the donation, the college would record the following for Year 1:* A. Increases in net assets without donor restrictions of $265,000. B. Increases in net assets with donor restrictions of $265,000. C. Increases in net assets without donor restrictions of $250,000 and increases in net assets with donor restrictions of $15,000. D. Increases in net assets without donor restrictions of $15,000 and increases in net assets with donor restrictions of $250,000.

B. Increases in net assets with donor restrictions of $265,000. The initial contribution in which the donor stipulates that the principal must remain intact would be classified as an increase in net assets with donor restrictions along with the earnings, whose use is restricted but which the university can satisfy by implementing programming (the chair for accountancy). No distinction is made for purposes of financial statement display between the items restricted by the donor in perpetuity and the items for which restrictions are temporary in nature. Choice "1" is incorrect. Contributions for an endowment with earnings restricted for use to fund a chair in accountancy would be reported as an increase to net assets with donor restrictions. Choice "3" is incorrect. Both the contribution to fund the endowment and related earnings restricted as to use would be accounted for as an increase in net assets with donor restrictions. Choice "4" is incorrect. Both the contribution to fund the endowment and related earnings restricted as to use would be accounted for as an increase in net assets with donor restrictions.

MCQ-06819 *Jordon Township's Water & Sewer fund received interest earnings off its invested funds. What category would Jordon use to classify cash receipts from these earnings on its Water & Sewer enterprise fund statement of cash flows?* A. Operating activities. B. Investing activities C. Capital and related financing activities. D. Noncapital financing activities

B. Investing activities Interest income/cash receipts are reported as *"investing activities"* (not as operating activities). Interest expense/cash payments are either: capital and related financing; or noncapital financing. Choice "2" is correct. Investing activities include making and collecting loans and acquiring and disposing of debt or equity instruments including interest and dividend income. Choice "1" is incorrect. Operating activities generally result from providing services and producing or delivering goods and all other transactions not defined by other classifications. Payment for services to other funds, including payments in lieu of taxes, are treated as operating. Choice "3" is incorrect. Capital and related financing activities include acquiring and disposing of capital assets, borrowing money for capital acquisitions and repaying amounts borrowed (including interest) as well as capital grants, special assessments levied for capital acquisitions, etc. Choice "4" is incorrect. Noncapital financing activities include borrowing for noncapital purposes as well as cash receipts from grants or subsidies, property taxes, etc.

MCQ-09762 Which of the following choices correctly matches the type of investment with its most appropriate fair value classification? Investment Type -- Level A. U.S. Treasury bill -- Level 3 B. Mortgage-backed securities -- Level 2 C. Limitedpartnerships -- Level 1 D. Large public company common stock -- Level 3

B. Mortgage-backed securities -- Level 2 An asset-backed security is an investment whose value and income payments are driven by a pool of underlying assets. A mortgage-backed security is a type of asset-backed security that is secured by a mortgage or mortgages. Level 2 investments may trade in active markets, but tend to be based on dealer quotations or alternative sources supported by observable investments or inputs that trade in inactive markets. A mortgage-backed security will most likely fall under a Level 2 classification. Choice "1" is incorrect. A U.S. Treasury bill represents a short-term fixed obligation of the United States government, which means it has a very active market and quoted market prices are readily available. As a result, this type of investment will be categorized under Level 1. Choice "3" is incorrect. Limited partnerships allow investors to take advantage of opportunities without incurring the risks and liabilities of a general partnership. These types of investments trade infrequently (if at all), and as a result have significant unobservable inputs that therefore requires that they fall under Level 3. Choice "4" is incorrect. A large company's common stock will have a very active market and therefore have quoted market prices. As a result, the most appropriate categorization is Level 1.

MCQ-05099 Palmetto Inc. is currently using the equity method to account for its 30% investment in Royal Company. In the acquisition last year of Royal Co. common stock, Palmetto calculated $1,000,000 of goodwill. The correct accounting for this goodwill during the current year is: A. Amortization over the anticipated holding period of the Royal Company stock. B. No accounting necessary. C. Amortization over 40 years. D. Test for impairment at year-end.

B. No accounting necessary. Choice "2" is correct. Any goodwill created in an investment accounted for under the equity method is ignored. It is neither amortized nor tested for impairment. The entire investment (using the equity method) is subject to the impairment test. Choices "3" and "1" are incorrect. Goodwill is no longer amortized in any situation. Choice "4" is incorrect. Purchased goodwill is only tested for impairment in an acquisition of a controlling interest in another company. As Palmetto acquired only 30% of Royal, no consolidation will occur; the investment will be correctly accounted for under the equity method.

MCQ-06026 The County Road & Bridge fund is funded by gas taxes whose use is restricted by law to road construction and is properly classified as a special revenue fund. The fund maintains inventories of road signage and road construction materials. Resources associated with those inventories would be classified within fund balance as: A. Restricted. B. Non-spendable. C. Assigned. D. Unassigned.

B. Non-spendable. Choice "2" is correct. Inventories are considered non-spendable. Non-spendable fund balances represent resources in a form that cannot be spent (e.g., inventories or prepaid expenditures) or are legally or contractually required to remain whole (e.g., permanent fund principal). Choice "1" is incorrect. Inventories are considered non-spendable, not restricted. Restricted fund balances represent resources whose use has been limited by external sources such as creditors (e.g., debt covenants), contributors, other governments, laws, constitutional provisions or enabling legislation. Choice "3" is incorrect. Inventories are considered non-spendable, not assigned. Assigned fund balances are constrained by the government's intent to be used for specific purposes but are neither restricted nor committed. Choice "4" is incorrect. Inventories are considered non-spendable, not unassigned. Unassigned fund balance is the residual classification for the general fund. This classification represents fund balance that has not been assigned to other funds and that has not been restricted, committed or assigned to specific purposes within the general fund.

MCQ-09253 Under U.S. GAAP, the lessor and the lessee may classify the same lease transaction in each of the following ways, except: Lessor -- Lessee A. Operating Operating B. Operating Finance C. Finance Operating D. Finance Finance

B. Operating Finance Choice "2" is correct.If the lessee treats the lease as a finance lease, it is because at least one of the OWNES criteria has been met. If this is the case, the lessor will also categorize the lease as a finance lease (either sales-type or direct financing). Choice "1" is incorrect. Both the lessor and lessee may classify a lease as an operating lease. Choice "3" is incorrect. It is possible for the lessee to classify a lease as operating while the lessor classifies the same lease as a finance lease. This situation will occur if none of the OWNES criteria are met but the PC criteria are met (in which case it will be a direct financing lease). Choice "4" is incorrect. Both the lessor and lessee may classify a lease as a finance lease.

MCQ-09388 *Geartyville established the Cox Administration Corporation, a nonprofit corporation for the purpose of administering employee welfare benefits for the employees of Geartyville. The city appoints the corporation's Board of Directors. The financial statements of the Cox Administration Corporation should be:* A. Presented discretely as a component unit. B. Presented as a blended component unit. C. Only disclosed in the notes to the financial statements. D. Displayed as a stand alone corporation.

B. Presented as a blended component unit. Step 1: Primary Governments and Component Units A government is viewed as a stand-alone or primary government if it has a separately elected governing board, it is a legal entity and it is financially independent (SELF). SE: separate elected governing board - No bc the city appoints the corporation's board of directors L: legal entity - Yes bc it is a non for profit affiliated with a government entity F: financially independent - No bc it exists for the purpose of administering welfare benefits for the government thus funded by the government. *If government meets all four criteria it is a Primary Government.* Step 2: Component units are presented either discretely or in a blended format. *Discrete:* Component units are presented as discrete (separate columns) on the primary government's financial statements. *Blended:* Blended presentations are made when the component unit either exclusively serves the primary government or when the component unit's governing body is substantially the same as the primary government's governing body. Blending involves consolidation of activities. Rule: Component units include legally separate organizations (governmental, not-for-profit, or for profit) for which elected officials of the primary government are financially accountable. A primary government is financially accountable if it appoints a voting majority of the organization's governing body. A component unit should be included in the reporting entity financial statements using the blending method when the component unit provides services entirely or almost entirely to the primary government or otherwise exclusively or almost exclusively benefits the primary government even though it does not provide services directly to it. The Cox Administration Corporation is a component unit since its governing body is appointed by the City. The corporation would be presented as a blended component unit since it was established by the primary government to administer its employee benefit programs in a manner that exclusively benefits the primary government even though it provides services to the employees rather than directly to the primary government itself. Choice "1" is incorrect. The Cox Administration Corporation primarily serves the primary government and would be presented as a blended not discrete component unit. Choice "3" is incorrect. The Cox Administration Corporation's financial statements are presented as a blended component unit, not simply disclosed. Choice "4" is incorrect. The Board of Directors of the Cox Administration Corporation is appointed by the primary government; the corporation is a component unit, not a stand alone corporation.

MCQ-09434 The Village of James has elected to use the modified approach for reporting infrastructure for the year ended December 31, Year 1. Using the modified approach, the government's asset management system does not need to include: A. Inventory of assets. B. Summary of condition assessments. C. Estimated net realizable value of community infrastructure. D. Amount necessary to maintain and preserve the condition of the assets.

B. Summary of condition assessments. If the government is unable to arrive at the cost data for its infrastructure, the use of a modified approach (no capitalization needed) is acceptable, provided that supplementary information describing the infrastructure, its condition, and estimation of expenses needed to maintain condition is included in the RSI. A complete new professional assessment of the infrastructure condition is necessary every three years.

MCQ-09449 *The least restrictive classification of governmental fund balances is titled:* A. Unreserved B. Unassigned C. Unspendable D. Committed

B. Unassigned Five government fund balances NUCAR. N Nonspendable U Unrestricted C Committed A Assigned R Restricted *Unassigned* Unassigned fund balances is the *residual classification for the general fund.* This classification represents fund balance that has not been assigned to other funds and that has not been restricted, committed or assigned to specific purposes within the general fund. *The general fund should be the only fund that shows a positive unassigned fund balance amount.* Over- expenditure of resources in other governmental funds may, however, result in a reported negative unassigned fund balance. Choice "1" is incorrect. "Unreserved" is older terminology. Although unreserved and undesignated fund balance classifications approximated the meaning of the "unassigned" fund balance prior to implementation of GASB 54, the terminology is now obsolete and a distracter. Choice "3" is incorrect. There is no such classification as "unspendable." This is a distracter. There is a classification titled "nonspendable." These residual amounts may indeed be unassigned, but they are not available and would not be less limited as to use than resources classified as unassigned. Choice "4" is incorrect. Committed funds are internally limited by formal action of the government's highest level of decision-making authority and would not be less limited as to use than resources classified as unassigned.

MCQ-00389 *A business combination is accounted for as an acquisition. Which of the following expenses related to the business combination should be included, in total, in the determination of net income of the combined corporation for the period in which the expenses are incurred?* *Fees of finders and consultants:* *Registration fees for equity securities issued:* A. Yes Yes B. Yes No C. No Yes D. No No

B. Yes No Choice "2" is correct. Fees of finders and consultants are expensed in the period incurred. Registration fees for equity securities issued decrease additional paid-in capital (stockholders' equity).

Bonds Payable (LT liability)

Bond Terminology A bond indenture is a contract that specifies the terms between the bond issuer and the bondholders. Among the elements of the contract: - Face Value: The total dollar amount of the bond. Bonds are generally sold in denominations of $1,000 and are quoted in 100s. (Always pay off the face) - Stated/Nominal/Coupon Rate: The interest to be paid to the bondholders. - Life of the Bond: The number of periods from the bond date to the maturity date. - Frequency of interest payments (annual, semiannual). Bond Issuance *The selling price of a bond is equal to the present value of the future cash payments related to the bond, including both the principal and interest payments using the effective rate of interest.* *Stated rate > Mkt rate = Premium* *Stated rate < Mkt rate = Discount*

On July 1, Year 1, Plato Inc. sold $500,000, 10% ten-year bonds at 103. On July 5, Year 1, Socrates Inc., a wholly owned subsidiary purchased half of Plato Inc.'s bonds at 101. What amount of gain (loss) related to this transaction should Plato recognize on its income statement at December 31, Year 1? A. $0 B. $(2,500) C. $(5,000) C. $5,000

Bonds If one member of the consolidated group acquires an affiliate's debt from an outsider, the debt is *considered to be retired and a gain/loss is recognized.* This gain/loss on extinguishment of debt is calculated as the difference between the price paid to acquire the debt and the book value of the debt. This gain/loss is not reported on either company's books, but is recorded on the consolidated income statement through an elimination entry. All intercompany account balances are also *eliminated; e.g., bond interest payable and bond interest receivable.* Plato's issuance of bonds JE: Cr Cash ($500,000 x 1.03) 515,000 CrBonds payable 500,000 Dr BP premium 15,000 Dr Bond interest payable $XXX Cr Bond interest receivable $XXX Dr Bonds payable 250,000* Dr Premium on bonds payable 7,500** Cr Investment in bonds 252,500*** Cr Gain on extinguishment 5,000*** * $500,000 face amount x ½ = $250,000 ** [($500,000 x 103%) x ½ (half bonds sold) = $7,500 *** $250,000 x 101% = $252,500 ****($250,000 + $7,500) - $252,500 = $5,000

MCQ-09317 *Coco and Chanel are partners who share profits and losses in the ratio of 3:2, respectively. On August 31, their capital accounts were as follows:* *Coco $70,000* *Chanel 60,000* *On that date, they agreed to admit Chance as a partner with a one-third interest in the capital and profits and losses for an investment of $50,000. The new partnership will begin with a total capital balance of $180,000.* *Immediately after Chance's admission, what are the capital balances of the partners?* Coco -- Chanel -- Chance A.. $60,000 $60,000 $60,000 B. $63,333 $56,667 $60,000 C. $64,000 $56,000 $60,000 D. $70,000 $60,000 $50,000

Bonus method: 'B" Balance in capital accounts* 180,000 x 1/3 = 60,000 - 50,000 Chance's inv. = 10,000 Profit sharing 3:2 Coco (3/5): 10,000 x .60 = 6,000 Chanel (2/5): 10,000 x .40 = 4,000 Capital accounts balances Coco: $70,000 - 6,000 = 64,000 Chanel: 60,000 - 4,000 = 56,000 Chance: 60,000 Chance gets 1/3 of the 180,000 balance under the Bonus method.

MCQ-06571 Northstar Co. acquired a registered trademark for $600,000. The trademark has a remaining legal life of five years, but can be renewed every 10 years for a nominal fee. Northstar expects to renew the trademark indefinitely. What amount of amortization expense should Northstar record for the trademark in the current year? A. $40,000 B. $15,000 C. $0 D. $120,000

C. $0 Because the trademark is expected to be renewed indefinitely, there will be no amortization expense on the books. Amortization is only recorded for intangible assets with a definite life. Choice "1" is incorrect. The 15 year useful life here is equal to the remaining legal life (5 years) and the first 10 year renewal. This is not applicable here due to the expectation of indefinite renewal. Choice "2" is incorrect. This amount represents the value of the acquired trademark amortized over 40 years. There will be no amortization due to the expectation that the trademark will be renewed indefinitely. Choice "4" is incorrect. This answer assumes amortization over the remaining legal life, which is not applicable because the company expects to renew indefinitely.

MCQ-09444 Refund Ridge Township issued General Governmental Refunding Bonds for $1,000,000 at 98 during the year ended June 30, Year 1. The entire amount of the difference between bond proceeds and face amount relates to the original issue discount. The Township would report the following balances in the indicated accounts in its Debt Service Fund financial statements at and for the year ended June 30, Year 1: Bonds Payable -- Other Financing Sources -- Other Financing Uses A. $1,000,000 $0 $980,000 B. $0 $20,000 $1,000,000 C. $0 $1,000,000 $20,000 D. $0 $0 $0

C. $0 $1,000,000 $20,000 New debt proceeds are recorded as *other financing sources,* a resource inflow: Dr Cash $9800,00 Dr Other financing uses (discount) $20,000 Cr Other financing sources—debt proceeds $1,00,000 Proceeds from refunding debt would be displayed at face value as "Other Financing Sources" in the debt service fund associated with the refunded debt with any original issue discount displayed as "Other Financing Uses." Underwriter's fees (as distinct from original issue discount) netted against bond proceeds would have been charged to expenditures. The transaction associated with the issuance of the debt would appear on the Statement of Revenues, Expenditures and Changes in Fund Balance. Choice "1" is incorrect. The non-current debt, bonds payable, would not be recorded in the governmental fund financial statements, it would only be recorded in the government-wide financial statements under the governmental activities column. "Other Financing Sources" would be displayed at the face amount of the bond and any difference between the face amount and proceeds associated with the original issue discount would be displayed as "Other Financing Uses." Choice "2" is incorrect. Refund bonds would be recorded in the debt service fund associated with the refunded debt. The face amount of the bonds would be displayed as proceeds under the "Other Financing Sources" classification while any discount associated with an original issue discount would be displayed beneath the "Other Financing Uses" classification. Charges associated with underwriter's fees that are netted against bond proceeds would be displayed as an expenditure. Choice "4" is incorrect. The issuance of refunding bonds would be recorded in the debt service fund financial statements as bond proceeds under the category "Other Financing Sources" at face value with any original issue discount reported as "Other Financing Uses." It would not be either recorded exclusively on the government-wide financial statements or recorded in another fund.

MCQ-09441 *Narli State University benefits from the donated services of numerous volunteer faculty members. The performance of both regular compensated and uncompensated volunteer faculty members is regularly and similarly evaluated. Both groups must meet Narli standards and provide services in the same way. During the June 30, Year 1 fiscal year, Narli develops the following statistics:* *Salaries and fringe benefits of compensated faculty $1,000,000* *Value of uncompensated volunteer services 200,000* *Reimbursement expenses for volunteer faculty 45,000* *The amount reported for total salary and fringe benefit expense for Narli University for the year ended June 30, Year 1 would be:* A. $1,000,000 B. $1,045,000 C. $1,200,000 D. $1,245,000

C. $1,200,000 *Contributions of services* are recorded as revenue *some of the time.* The services must either enhance a physical asset or meet the following *(SOME)* criteria: they require *specialized skills, are otherwise needed, and are measured easily.* Services that meet the criteria are recorded as revenues and assets or expenses at their fair value as follows: Dr Expense or asset $XXX Cr Contributions—Nonoperating revenue $XXX The valuation of salaries and fringes would include the amounts paid for salaries and fringe benefits to compensated staff and the fair value of the services contributed by volunteers. Services provided by the volunteers require specialized skills, would otherwise have been purchased by the university and can be measured by the rate of pay associated with compensated staff. *Both groups must meet Narli standards and provide services in the same way.* The volunteer staff meets the SOME criteria and would be included as both revenue and expense on the Narli State University financial statements. Choice "1" is incorrect. The valuation of salaries and fringes would include the amounts paid for salaries and fringe benefits to compensated staff and the fair value of the services contributed by volunteers. The amount shown by this choice only values salaries and fringes at the amount paid to compensated staff. Choice "2" is incorrect. The valuation of salaries and fringes would include the amounts paid for salaries and fringe benefits to compensated staff and the fair value of the services contributed by volunteers. The amount shown by this choice only values salaries and fringes at the amount paid to compensated staff plus non compensatory expense reimbursements paid to volunteer staff. Choice "4" is incorrect. The valuation of salaries and fringes would include the amounts paid for salaries and fringe benefits to compensated staff and the fair value of the services contributed by volunteers. The amount shown by this choice over values salaries and fringes at the amount paid to compensated staff and the value of volunteer services plus non compensatory expense reimbursements paid to volunteer staff.

MCQ-09255 Lennon Inc. is the lessor on a 10-year (120-month) operating lease on warehouse space at $1,500 per month. Lennon is offering the lessee the first four months free and the next eight months after that at half off ($750 per month), with the remaining nine years at the full rate. After the first year of the lease, Lennon will have booked a cumulative balance in rent receivables of: A. $5,200 B. $6,000 C. $10,800 D. $16,800

C. $10,800 Fist 4 months rent: 0 Next eight months at half off: $750x8= $6,000 Remaining 9yrs at full rate: [$1,500 x (9x12)]= $162,000 162,000 + 6,000 = 168,000 Total rent 168,000 / 120months = 1,400 rent per month The journal entry for each of the first four months is: Dr Rent receivable $ 1,400 Cr Rental income$ 1,400 *Total rental income must be pro-rated equally to show the promotion over 120 months which is why the lessor still recognizes 1,400 as rental income. The lessor allowed the first four-months to be free and cannot record any cash received, thus must record a receivable of 1,400.* The journal entry for each of the next eight months is: Dr Cash $750 Cr Rent receivable 650 Cr Rental income $1,400 *Now the lessor paid 750 in cash for 8 months and the remaining balance goes to rent receivable.* The total cumulative balance in rent receivables is equal to: ($1,400 × 4) + ($650 × 8) = $10,800. The journal entry that Lennon will book every month after the first year of the lease is: Dr Cash $1,500 Cr Rent receivable $100 Cr Rental income 1,400 The following year when rent returns to normal at $1,500 you will begin reducing the excess in your A/R with excess $100 of cash received. Choice "1" is incorrect. $5,200 reflects the balance if the last eight months were booked at $650, with no recognition the first four months. Choice "2" is incorrect. $6,000 reflects the balance if the last eight months were booked at the monthly cash received amount of $750, with no recognition the first four months. Choice "4" is incorrect. $16,800 reflects the balance if all 12 months were booked at $1,400.

MCQ-09447 During the year ended June 30, Year 0, the Aberdeen Care Clinic, a not-for-profit organization, received an endowment for $2,300,000. The endowment is to remain intact with income and appreciation from the investment of the endowment to be used in support of a research program to assist the elderly. By June 30, Year 2, the endowment had accumulated $110,000 in gains. On July 1, Year 2, the company liquidated 65% of the cumulative appreciation of the endowment for use in the research program. During the year ended June 30, Year3, the endowment earned $30,000, which it spent on the research program but, at year end, suffered market losses that reduced the total value of the investment below its initially recorded value to $2,250,000. The amounts reported as net assets released from donor restrictions for the year ended June 30, Year 3 would be: A. $30,000 B. $71,500 C. $101,500 D. $151,500

C. $101,500 The amount of assets released from restrictions would be equal to the *earnings and liquidated investments expended in accordance with the bequest*: Earnings 30,000 Liquidated gains ($110,000 × 65%) 71,500 = Total 101,500 Choice "1" is incorrect. The amount of the assets released from restrictions is not purely equal to the investment earnings used in the program. Choice "2" is incorrect. The amount of the assets released from restrictions is not purely equal to the investments sold whose proceeds were used in the program. Choice "4" is incorrect. The amount of the assets released from restrictions is not equal to the combined value of the resources used in the program and the investment loss. The investment loss does not represent assets released from restriction.

MCQ-09432 *On January 1, Year 1, Black Dog Corp. began operations and issued 30,000 shares of $5 par common stock for $9/share. On June 30, the company bought back 10,000 shares of $8/share. Then, On September 15, the company resold 5,000 shares for $12/share. What amount of total additional paid-in capital should Black Dog report on its December 31, Year 1 balance sheet if Black Dog uses the cost method to account for its treasury stock?* A. $20,000 B. $120,000 C. $140,000 D. $165,000

C. $140,000 Under the cost method, Black Dog would record the following journal entries for its Year 1 stock transactions: *Cost method - G/L recorded upon reissue* Jan. 1, Year 1—Issue 30,000 shares of $5 par common stock for $9/share: SP $9 - $5 par = $4 Gain Cash (30,000 x $9 SP) 270,000 Common stock (30,000 x $5 pair ) 150,000 APIC—CS (30,000 x $4 Gain) 120,000 June 30, Year 1—Repurchase 10,000 shares for $8/share: Treasury stock (10,000 x $8 PP) 80,000 Cash 80,000 *No G/L yet* Sept. 15, Year 1—Resell 5,000 shares for $12/share: Cash (5,000 x $12) 60,000 Treasury stock (5,000 x $8 PP) 40,000 APIC—TS (Plug) 20,000 *The 20,000 APIC-TS (Gain) is used to absorb future losses.* Therefore, total APIC on Black Dog's balance sheet at December 31, Year 1 would be $140,000 ($120,000 from original sale + $20,000 from sale of treasury stock). Choice "1" is incorrect. This is the amount of APIC—Treasury Stock recorded as a result of the resale of treasury stock on Sept. 15, Year 1 (see the explanation above). The question asks for total APIC. Choice "2" is incorrect. This is the amount of APIC—Common Stock recorded as a result of the original issuance of common stock on Jan. 1, Year 1. Choice "4" is incorrect. This is the amount of total APIC that would be reported on the Dec. 31, Year 1 balance sheet under the par value method.

MCQ-09683 *On January 1, Year 1, Sunshine Corporation acquired an 80% ownership interest in Grey Sky Enterprises by purchasing 400,000 of Grey Sky's 500,000 voting common shares outstanding for $10,000,000. Additional information regarding Grey Sky as of January 1, Year 1, follows:* Book Value Fair Value Net assets $11,200,000 $11,500,000 *On the acquisition date, what is the noncontrolling interest that will be reported on the consolidated balance sheet of Sunshine Corporation under the IFRS partial goodwill method?* A. $1,200,000 B. $1,500,000 C. $2,300,000 D. $2,500,000

C. $2,300,000 *Under IFRS, noncontrolling interest (and goodwill) can be calculated using either the full goodwill method, which is the method required under U.S. GAAP, or the partial goodwill method. Under the partial goodwill method, noncontrolling interest on the balance sheet is calculated as:* Noncontrolling interest (BS) = *100% Fair value of subsidiary's net assets* × % Noncontrolling interest $11,500,000 FV x (1.0 - 0.80) = $2,300,000 NCI Under the U.S. GAAP, noncontrolling interest on the balance sheet is calculated as follows: NCI = *100% Fair value of subsidiary* x % NCI The fair value of the subsidiary is calculated as follows: FV subsidiary x 80% = $10,000,000 FV subsidiary = $12,500,000 Therefore, the NCI is: NCI = $12,500,000 x 20% = $2,500,000 Note that the NCI can also be calculated as the difference between the fair value of the subsidiary and the acquisition cost: $12,500,000 FV - $10,000,000 acquisition cost = $2,500,000 NCI

MCQ-09443 Healthy Care Hospital Inc., a private not-for-profit organization, accumulated the following patient service revenue transactions in their records during the June 30, Year 1, fiscal year: Gross patient fees for all services provided at usual and customary rates $5,000,000 Contractual allowances for third party payments 800,000 Bad debts written off after an assessment of patient's ability to pay 350,000 Current year bad debt allowance 300,000 Charity care 1,300,000 The amount reported as net patient service revenue for Healthy Care Hospital Inc. for the year ended June 30, Year 1, would be: A. $5,000,000 B. $4,200,000 C. $2,900,000 D. $2,850,000

C. $2,900,000 Health Care Organization Revenue Recognition Patient service revenue should be accounted for on the accrual basis at usual and customary fees, even if the full amount is not expected to be collected. Although patient service revenue is accounted for on a gross basis, deductions are made from gross revenue for reporting purposes to display revenues net. *Charity care, the value of services that a health care organization gives away, is not displayed in the financial statements. Bad debt expense (Operating expense) is recorded when the allowance is established after an evaluation of a patient's ability to pay. Otherwise, bad debts are recorded as a deduction from revenue. When a health care organization is regularly making evaluations of a patient's ability to pay any bad debt expense is recorded as an operating expense and are not offset against revenues.* Net patient service revenues reported on a hospital statement of activities are comprised of gross earnings net of contractual allowances and exclude charity care. Net patient service revenues are computed as follows: Gross patient fees 5,000,000 Contractual allowances (800,000) Charity care (1,300,000) Net patient service revenue 2,900,000 Choice "1" is incorrect. Net patient service revenues reported on a hospital statement of activities are comprised of gross earnings net of contractual allowances and exclude charity care. This choice does not reduce patient service revenue by any of the appropriate deductions. Choice "2" is incorrect. Net patient service revenues reported on a hospital statement of activities are comprised of gross earnings net of contractual allowances and exclude charity care. This choice only reduces patient service revenue by contractual allowances. Choice "4" is incorrect. Net patient service revenues reported on a hospital statement of activities are comprised of gross earnings net of contractual allowances and exclude charity care. This choice not only reduces patient service revenue by appropriate deductions but also improperly reduces revenue by the net change in the allowance account. Bad debt expense is matched against revenue as an expense when the allowance was established after an evaluation of the patient's ability to pay. Revenue associated with doubtful accounts is recognized and would be included as a component of net patient service revenue. The change in the allowance is not relevant.

MCQ-9324 *The following information applies to Babydoll Company's defined benefit pension plan at December 31, year 8:* *Projected benefit obligation, December 31, year 7 $ 2,000,000* *Projected benefit obligation, December 31, year 8 2,220,000* *Fair value of plan assets, December 31, year 7 $1,750,000* *Fair value of plan assets, December 31, year 8 $2,025,000* *Unrecognized prior service cost, December 31, year $7,500,000* *Year 8 Service cost $200,000* *Expected benefits payable - year $500,000* *Discount rate 6%* *Expected rate of return on plan assets 8%* *The company's employees have an average remaining service life of 10 years. The company has no unrecognized net gains or losses. What is Babydoll's U.S. GAAP net periodic pension cost for Year 8?* A. $208,000 B. $221,200 C. $230,000 D. $243,00

C. $230,000 To calculate U.S. GAAP net periodic pension cost, we must first calculate interest cost, expected return on plan assets and amortization of prior service cost: Interest cost = Beginning PBO × Discount rate = $2,000,000 × 6% = $120,000 Expected return on plan assets = Beginning fair value of plan assets x expected return = $1,750,000 × 8% = $140,000 Amortization of prior service cost = $500,000 / 10 years = $50,000 Net periodic pension cost is then calculated as follows: S Service cost 200,000 I Interest cost 120,000 R (Expected return on plan assets) (140,000) A Amortization of prior service cost 50,000 G Amortization of net losses (gains) 0 E Amortization of existing obligation (asset) 0 = Net periodic pension cost 230,000

MCQ-09676 *Hi-Tech Corp. spent $300,000 on research and development to generate new product lines. Only one of the five product lines resulted in a patented item, while the remaining four were considered unsuccessful. The cost of the product that was successfully patented included $30,000 in research costs and $40,000 in development costs. Under IFRS, how much of the $300,000 should be recognized as an expense?* A. $300,000 B. $270,000 C. $260,000 D. $230,000

C. $260,000 All of the research and development costs related to the unsuccessful product lines must be expensed. The $30,000 in research costs related to the product that was successfully patented must also be expensed because all research costs are expensed under IFRS. The $40,000 in development costs will most likely be capitalized because they related to a successfully patented product and development costs can be capitalized under IFRS. Therefore, the entity should expense $260,000 ($300,000 − $40,000). IFRS *Under IFRS, research costs must be expensed like U.S. GAAP, but development after feasibility costs may be capitalized if certain criteria are met.* U.S. GAAP = Generally expensed in period incurred Under U.S. GAAP, research and development costs must be expensed in the period incurred. In general, items to be expensed as R&D include: equipment, material, labor, overhead, design, testing, engineering, modification, and salaries of research staff. Exceptions to expensing include: 1. Software after feasibility are capitalized, not expensed* 2. PPE has alternative future use capitalized, not expensed*

MCQ-00452 *Clark Co. had the following transactions with affiliated parties during Year 1:* - *Sales of $60,000 to Dean, Inc., with $20,000 gross profit. Dean had $15,000 of inventory on hand at year-end. Clark owns a 15% interest in Dean and does not exert significant influence.* - *Purchases of raw materials totaling $240,000 from Kent Corp., a wholly-owned subsidiary. Kent's gross profit on the sale was $48,000. Clark had $60,000 of this inventory remaining on December 31, Year 1.* *Before eliminating entries, Clark had consolidated current assets of $320,000. What amount should Clark report in its December 31, Year 1, consolidated balance sheet for current assets?* A. $317,000 B. $303,000 C. $308,000 D. $320,000

C. $308,000 Choice "3" is correct, $308,000 current assets in the 12/31/Year 1 consolidated balance sheet ($320,000 less 12,000 unrealized profit in inventory) The unrealized profit to be eliminated from inventory is calculated as follows: Interco. profit on inventory % x remaining inventory = ($48,000/$240,000) x $60,000 = $12,000 Note: No elimination is made related to the transaction with Dean, Inc. because Dean (owned less than 50%) is not consolidated.

MCQ-00811 *West, Inc. made the following expenditures relating to Product Y:* *-Legal costs to file a patent on Product Y $10,000* *(Production of the finished product would not have been undertaken without the patent)* *- Special equipment to be used solely for development of Product Y 60,000* *(The equipment has no other use and has an estimated useful life of four years)* *- Labor and material costs incurred in producing a prototype model 200,000* *- Cost of testing the prototype 80,000* *What is the total amount of costs that will be expensed when incurred under U.S. GAAP?* A. $295,000 B. $350,000 C. $340,000 D. $280,000

C. $340,000 *Succesful defense patent legal fees, legal and other patent registration fees can be capitalized. *Goodwill cannot be amortized, but is subject to the impairment test.* Choice "3" is correct. $340,000 will be expensed when incurred. Capitalize Legal costs to file a patent on Product Y [$10,000]. Production of the finished product would not have been undertaken without the patent (patents should be capitalized). 10,000 Expense Special equipment to be used solely for development of product Y [$60,000]. The equipment has no other use and has an estimated useful life of four years (capitalize only if it has alternative use). 60,000 Expense Labor and material costs incurred in producing a prototype model [$200,000] (R&D should be expensed). 200,000 Expense Cost of testing the prototype [$80,000] (R&D should be expensed). 80,000 Total Expense: 340,000 Total Capitalize: 10,000

MCQ-09271 *During Year 1, Meriwether Construction Co. started a construction job with a contract price of $3,000,000. The job was completed in Year 2, and the company uses the percentage of completion method. The following information is available for Year 1 and Year 2:* Year 1 -- Year 2 *Cost incurred to date $500,000 $2,400,000* *Estimated cost to complete 1,500,0000* *Billing to date 300,000 1,800,000* *Collection to date 100,000 1,600,000* *What amount of gross profit should Meriwether recognize for this job for Year 2?* A. $250,000 B. $1,000,000 C. $350,000 D. $600,000

C. $350,000 *Calculation of Current Gross Profit* *Step 1* *Contract price - Total estimated cost = Total gross profit* *Step 2* *Cost to date / Total estimated cost = % Completed* *Step 3* *Step 1 × Step 2 = GP earned to date* *Step 4* *GP earned to date - GP previously recognized = Current GP* The total cost for the contract was $2,400,000. The contract price was $3,000,000. So Meriwether ended up making a $600,000 ($3,000,000 - $2,400,000) profit cumulative for the two years together. To determine how much was recognized in the second year, a determination of how much was recognized in the first year must be made. Meriwether incurred $500,000 of cost in the first year and expected to incur $1,500,000 more. At the end of Year 1, Meriwether expected to make a profit of $1,000,000 on the job ($3,000,000 - $2,000,000). Meriwether had incurred 25 percent ($500,000 / $2,000,000) of the total estimated cost and would have recognized $250,000 of profit. In Year 2, total costs were higher. Total profit is now $600,000 for the entire project. Since Meriwether had already recognized $250,000 in Year 1, $350,000 is still to be recognized in Year 2 ($600,000 - $250,000).

MCQ-09420 Nadaf University received $4,000,000 in tuition revenue for the year ended December 31, Year 1. In addition to the tuition received, the university offered $500,000 in scholarships but had to refund $200,000 for canceled classes. For the year ended December 31, Year 1, Nadaf would record gross revenue for tuition of: A. $3,800,000 B. $4,000,000 C. $4,300,000 D. $4,500,000

C. $4,300,000 University and Institutions of Higher Learning Revenue Recognition Student tuition and fees should be reported at gross amount (Total income earned before deductions). Scholarships, tuition waivers, and similar reductions are considered either expenses or a separately displayed allowance reducing revenue. Read the requirements first. Notice the dates. Gross revenues for tuition would be computed as follows: Tuition revenue 4,000,000 Scholarship 500,000 Refunds (200,000) Total 4,300,000 Choice "1" is incorrect. Gross revenues includes scholarships. Choice "2" is incorrect. Gross revenues include scholarships net of refunds. Choice "4" is incorrect. Gross revenues are net of refunds.

MCQ-09265 Boone Corporation's outstanding capital stock at December 15 consisted of the following: - 30,000 shares of 5% cumulative preferred stock, par value$10 per share, fully participating as to dividends. No dividends were in arrears. - 200,000 shares of common stock, par value $1 per share. On December 15, Boone declared dividends of $100,000. What was the amount of dividends payable to Boone's common stockholders? A. $10,000 B. $34,000 C. $40,000 D. $47,500

C. $40,000 There are both common and preferred shareholders. The dividends are $100,000, and some of it has to go to the preferred shareholders since none of the answers is $100,000. The preferred goes first (since it is preferred) at 5% of par value or $15,000 (5% × 30,000 × $10). *Since the preferred is "fully participating," the common shares are initially treated the same way as the preferred. Fully participating CS is shared equally and then pro-rata.* The common thus gets its 5% or $10,000 (5% × $200,000 × $1). Then everybody shares in relation to their relative par values. PS: 30,00 x $10 par x .05 = 15,000 PS dividend CS: 200,000 x $1 par x .05 = 10,000 CS stock dividend Total par value common 200,000 (200,000 shares × $1 par) Total par value preferred 300,000 (30,000 shares × $10 par) = Total par value 500,000 The remaining 75,000 of CS dividend is allocated pro-rata. $300,000 PS / 500,000 = .60 x 75,000 = 45,000 Additional PS dividend $200,000 CS / 500,000 = .40 = 30,000 Additional CS divident PS: 15,000 + 45,000 = 60,000 Total PS dividend *CS: 10,000 + 30,000 = 40,000 Total CS dividend*

MCQ-09328 Faith Church decided to replace its electric organ with a multiple rack pipe organ. The church purchased the organ itself for $250,000 and a congregation member stepped forward to assemble the organ and perform the necessary carpentry work for $5,000. The congregation member is a skilled craftsman who normally charges $40,000 for this work. Other congregation members stepped forward to help with general labor assistance valued at $7,000. As a result of the transaction above, Faith Church should record revenues from contributed services of: A. $35,000 B. $40,000 C. $42,000 D. $47,000

C. $42,000 Rule: Not-for-profit organizations record donated services SOME of the time, when services meet the following criteria: *S*pecialized skill *O*therwise needed *M*easured *E*asily *Services are also recognized when a long-lived asset is enhanced.* The services for the construction of the pipe organ meet all criteria. The construction requires a specialized skill, the musical instrument is necessary equipment and there is a market price for the service. In addition, unskilled labor was used to *enhance a long-lived asset.* The church would record $35,000 in revenues from contributed specialized services per the rule above (40,000 fair value minus $5,000 paid) and would record $7,000 in revenue for the general labor used to enhance the long-lived asset. Value for specialized service 40,000 Amount paid (5,000) Net specialized service donated 35,000 General labor used to enhance long-lived asset 7,000 Total donated service revenue 42,000

MCQ-09309 At December 31, Year 8 Stephens Brothers Inc. has the following pension plan information: Fair value of plan assets, beginning of year $1,500,000 Fair value of plan assets, end of year 1,590,000 Contributions 350,000 Benefits paid 425,000 Expected return on plan assets 120,000 The expected return on plan assets was used to calculate net periodic pension cost. No actuarial gains or losses were incurred during Year 8. Stephens Brothers' effective tax rate is 40%. What is the net gain to be reported in Year 8 other comprehensive income under US GAAP? A. $0 B. $27,000 C. $45,000 D. $165,000

C. $45,000 Absent any actuarial gains or losses, the net pension gain incurred during an accounting period is the difference between the expected return on plan assets recorded in net periodic pension cost and the actual return on plan assets earned during the accounting period. *Under U.S. GAAP, this amount is reported in other comprehensive income, after-tax, in the period incurred.* The actual return on plan assets is calculated as follows: Beginning fair value of plan assets 1,500,000 + Contributions 350,000 - Benefits paid 425,000 + Actual return on plan assets 165,000 (Plug) = Ending fair value of plan assets 1,590,000 *Therefore, the difference between the actual return on plan assets and expected return on plan assets is $45,000 ($165,000 actual - $120,000 expected), which on an after-tax basis is $27,000 [$45,000 x (1-40%)].* This amount will be amortized using the corridor approach starting in Year 9. Choice "1" is incorrect. As explained above, there is a net pension gain equal to the after-tax difference between actual and expected return on plan assets. Choice "3" is incorrect. This is the net gain before tax. U.S. GAAP requires that the difference between actual and expected return on plan assets be reported in other comprehensive income net of tax. Choice "4" is incorrect. This is the actual return on plan assets, not the net pension gain to be reported in Year 8 other comprehensive income.

*Erika's Surf Shop had taxable income in Year 2 of $500,000 and pretax financial income of $600,000. The company had a cumulative $200,000 difference between its taxable income and pretax financial statement income at December 31, Year 1. These differences were solely related to accelerated depreciation methods used for income tax purposes. The enacted tax rate increased to 30 percent in Year 2 compared to an enacted rate of 20 percent in the prior year. At December 31, Year 2, the company would record a deferred tax* expense of: A. $150,000 B. $90,000 C. $50,000 D. $40,000

C. $50,000 I think the easiest way to do this is through a T-account for the deferred tax asset/liability. It has a beginning credit balance of $40,000 ($200,000 cumulative difference * 20% PY rate). Two things are going to be added to the beginning balance to reach the ending balance, the first is this year's temporary difference of $100,000 (pretax financial income-taxable income) times this year's rate of 30%, so $30,000. This is a liability because taxable income is less than pretax income, so that $100,000 difference will eventually be taxed in the future. The second item that is added to the liability balance is the change in beginning balance due to the rate change. The new rate is 30%, meaning the existing DTL would actually be worth $200,000*30%, which is $60,000. Subtracting the beginning balance from this, you get a change equal to $20,000 (60,000-40,000). In total, the DTL account increases by $30,000+20,000=$50,000, which is the deferred expense.

MCQ-05054 *Carter Components is computing the components of its net periodic pension cost for the current year ended December 31. Carter has calculated that its service cost is $60,000 and has computed interest cost as $42,000. The average remaining service life of its employees is 8 years. The return on $500,000 in plan assets was anticipated to be 8 percent but was actually 8.5 percent. The pension benefit obligation at the beginning of the year was $560,000 and, at the end of the year, $602,000. The company has an unrecognized gain of $60,000. To what extent will the unrecognized gain reduce current- year net periodic pension cost under U.S. GAAP?* A. $1,250 B. $750 C. $500 D. $25

C. $500 Under U.S. GAAP, unrecognized pension gains or losses are amortized over the average remaining service period if, at the beginning of the year, the gain or loss exceeds 10% of the greater of the beginning of the year PBO or the beginning of the year market-related value of plan assets (we will use the fair value of the plan assets in this example as the market-related value is not given and these amounts are approximately equal). At the beginning of the year, Rhino's PBO exceeds the fair value of the plan assets, so the net loss amortization is calculated as: Unrecognized gain $60,000 Less: 10% of Greater of Beg. PBO/Plan Assets ($560,000) = $560,000 PBO × 10% Excess $56,000 Excess / Average Remaining Service Life = $4,000 / 8 = $500

MCQ-09294 Do It Right Inc.'s actuary provided the company with the following information regarding its defined benefit pension plan for the year ended December 31, Year 7: Fair value of plan assets $5,580,000 Accumulated benefit obligation 3,400,000 Projected benefit obligation 4,930,000 Unrecognized prior service cost 400,000 Unrecognized transition obligation 275,000 Unrecognized net gain 140,000 Expected benefit obligation - Year 8 250,000 The company reported net periodic pension cost of $310,000 on its income statement and made a $500,000 contribution to the pension plan during Year 7. The company's effective tax rate is 40%. What amount should Do It Right record as a pension asset/liability on the December 31, Year 7, balance sheet under U.S. GAAP? A. $321,000 B. $489,000 C. $535,000 D. $815,000

C. $535,000 Under U.S. GAAP, unrecognized prior service cost, unrecognized transition obligations or assets and unrecognized net gains or losses must be reported in accumulated other comprehensive income, net of tax, until recognized as a component of net periodic pension cost through amortization. Unrecognized prior service cost, transition obligations and net losses all increase pension expense when recognized and are therefore recorded as a debit to accumulated OCI. Unrecognized transition assets and net gains decrease pension expense when recognized and are therefore recorded as a credit to accumulated OCI. For Do It Right, the total pension related amount to be reported in accumulated OCI (before tax) is: Unrecognized prior service cost 400,000 Unrecognized transition obligation 275,000 Unrecognized net gain (140,000) = 535,000 *= SIR"AGE" -> OCI -> % age of "AGE" each year is expensed* U.S. GAAP requires that this amount be reported on an after-tax basis: $535,000 × (1 − 40%) = $321,000. Note that this would be a $321,000 offset (debit) to accumulated OCI. Choice "2" is incorrect. This amount is calculated by adding the unrecognized net gain to the unrecognized prior service cost and transition obligation. The unrecognized net gain should be subtracted. Choice "3" is incorrect. This is the total of the unrecognized amounts that must be reported in accumulated other comprehensive income on a pre-tax basis. U.S. GAAP requires that these amounts be reported in accumulated OCI after-tax. Choice "4" is incorrect. This amount is calculated by adding the unrecognized net gain to the unrecognized prior service cost and transition obligation. The unrecognized net gain should be subtracted. Additionally, this is a pre-tax amount. U.S. GAAP requires that these amounts be reported in accumulated OCI after-tax.

MCQ-00496 Nolan owns 100% of the capital stock of both Twill Corp. and Webb Corp. Twill purchases merchandise inventory from Webb at 140% of Webb's cost. During Year 1, merchandise that cost Webb $40,000 was sold to Twill. Twill sold all of this merchandise to unrelated customers for $81,200 during Year 1. In preparing combined financial statements for Year 1, Nolan's bookkeeper disregarded the common ownership of Twill and Webb. What amount should be eliminated from cost of goods sold in the combined income statement for Year 1? A. $40,000 B. $24,000 C. $56,000 D. $16,000

C. $56,000 Choice "3" is correct, $56,000 elimination from cost of goods sold. Webb sold inventory with a cost of $40,000 to Twill at 140% of cost, or $56,000. This transaction resulted in intercompany revenue of $56,000, intercompany COGS of $40,000, and intercompany profit on the sale of inventory of $16,000 ($56,000 - $40,000). All intercompany amounts must be eliminated. Because Twill sold all of the inventory purchased from Webb, the intercompany profit is eliminated from Twill's cost of goods sold. The eliminating entry is: Dr Intercompany revenue - Webb 56,000 Dr Intercompany COGS - Webb 40,000 Cr Cost of goods sold -Twill 16,000 The total elimination from COGS is $56,000.

MCQ-09435 *Backdoor Inc. has 200,000 shares of $5 common stock outstanding. The company declared a stock dividend of 30,000 shares when the market price was $25. By how much did additional paid-in capital increase when the dividend was declared?* A. $0 B. $150,000 C. $600,000 D. $750,000

C. $600,000 *This stock dividend is an example of a small stock dividend as 30,000 shares is 15% of the 200,000 shares outstanding. A small stock dividend (< 20-25% of outstanding stock) is recorded at the fair value of the stock on the date of declaration, as follows:* *Dr Retained earnings @ FV 750,000* *Cr Common stock (30,000 x $5 par) 150,000* *Cr Additional paid-in capital 600,000* *Choice "1" is incorrect. When a stock dividend is large (> 20-25% of outstanding stock), the dividend is recorded at par value and no additional paid in capital is recorded. In this problem, the dividend is a small stock dividend, which is recorded using the fair value of the stock on the date of declaration (see the explanation above). (know this also)* *Dr RE @ par 150,000* *Cr CS 150,000* Choice "2" is incorrect. This is the amount of common stock at par value that would be recorded on the date the dividend is declared. Choice "4" is incorrect. This is the total value of the stock dividend, which is recorded as a reduction of retained earnings.

MCQ-09279 Do It Right Inc.'s actuary provided the company with the following information regarding its defined benefit pension plan for the year ended December 31, Year 7: Fair value of plan assets $5,580,000 Accumulated benefit obligation 3,400,000 Projected benefit obligation 4,930,000 Unrecognized prior service cost 400,000 Unrecognized transition obligation 275,000 Unrecognized net gain 140,000 Expected benefit obligation - Year 8 250,000 The company reported net periodic pension cost of $310,000 on its income statement and made a $500,000 contribution to the pension plan during Year 7. The company's effective tax rate is 40%. What amount should Do It Right record as a pension asset/liability on the December 31, Year 7, balance sheet under U.S. GAAP? A. $650,000 current liability. B. $2,180,000 non-current liability. C. $650,000 non-current asset. D. $2,180,000 current asset.

C. $650,000 non-current asset. $5,580,000 Fair value of plan assets - 4,930,000 Projected benefit obligation = $650,000 non-current asset. (Overfunded)

MCQ-01563 *On January 1, Year 1, Ward Corp. granted stock options to corporate executives for the purchase of 20,000 shares of the company's $20 par value common stock at $48 per share. All stock options were exercised on December 28, Year 1. Using an acceptable option pricing model, Ward calculated total compensation cost of $240,000. The quoted market prices of Ward's $20 par value common stock were as follows:* *January 1, Year 1 $45* *December 28, Year 1 60* *As a result of the grant and exercise of the stock options and the issuance of the common stock, Ward's additional paid-in capital increased by:* A. $740,000 B. $500,000 C. $800,000 D. $560,000

C. $800,000 Choice "3" is correct. $800,000 increase to additional paid-in capital. January 1 Journal Entry Dr Compensation expense 240,000 APIC - Stock options 240,000 December 28 Journal Entry Cash ($48 x 20,000 shares) 960,000 APIC - Stock options 240,000 Common stock (20,000 x $20) 400,000 APIC 800,000

MCQ-09746 Ansley Inc. (which uses IFRS) has debt on its books with a current value of $425,000. For the first time in its history, the company receives debt modifications that will result in future cash flows totaling $280,000. A. As a result of the modifications, the company will most likely: Handle the change prospectively, and keep the carrying value the same. B. Handle the change prospectively, and lower the carrying value over time. C. Book a gain in current operations and reduce the carrying value of the liability. D. Book a reduction in interest expense and reduce the carrying value of the liability.

C. Book a gain in current operations and reduce the carrying value of the liability. A debt modification that lowers future cash flows below the carrying value of the debt will result in a gain as well as a reduction in the carrying value of the liability, with the gain recorded in current operations. Choice "1" is incorrect. *Debt modifications are normally handled prospectively, but not when the future cash outflows are less than the carrying value of the debt.* Choice "2" is incorrect. Debt modifications are normally handled prospectively, but not when the future cash outflows are less than the carrying value of the debt. Choice "4" is incorrect. The modification results in a gain, not a reduction, of interest expense. - Recognize gain on: "Concession" is a gain 425,000 Carrying Amount of the Payable < 280,000 FV Asset Transferred > = 120,000 Gain

MCQ-09749 A major customer of the Green Company filed a lawsuit against Green in February of Year 2. Green has a fiscal yar end of December 31, and the company plans to issue their Year 2 financial statements on March 31, Year 3. The customer prevailed against Green, and the lawsuit was ultimately settled for $450,000 on January 27, Year 3. In its Year 2 financial statements, as a result of the lawsuit settlement Green will most likely: A. Reissue the financial statements for Year 2. B. Show a $450,000 expense in the Year 3 financial statements. C. Book a journal entry to reflect the $450,000 expense in Year 2. D. Disclose the nature of the event without reflecting the expense.

C. Book a journal entry to reflect the $450,000 expense in Year 2. Because the lawsuit arose during Year 2 and was settled before the Year 2 financial statements were issued, the expense should be booked in the Year 2 financial statements Choice "1" is incorrect. There is no need to reissue financial statements for Year 2 because they would not have been issued yet as of the settlement date of the lawsuit. Choice "2" is incorrect. The expense should be booked in Year 2 rather than in Year 3. Choice "4" is incorrect. The expense should be booked in Year 2 because the lawsuit arose in Year 2 and was settled before the financial statements were due to be issued.

MCQ-00867 *On April 30, Algee, Belger, and Ceda formed a partnership by combining their separate business proprietorships. Algee contributed cash of $50,000. Belger contributed property with a $36,000 carrying amount, a $40,000 original cost, and $80,000 fair value. The partnership accepted responsibility for the $35,000 mortgage attached to the property. Ceda contributed equipment with a $30,000 carrying amount, a $75,000 original cost, and $55,000 fair value. The partnership agreement specifies that profits and losses are to be shared equally but is silent regarding capital contributions. Which partner has the largest April 30 capital account balance?* A. Belger. B. All capital account balances are equal. C. Ceda. D. Algee.

C. Ceda. Choice "3" is correct. Ceda's capital account balance is the fair market value of the equipment donated. The $55,000 contribution is greater than Algee's $50,000 contribution and Belger's $45,000 contribution, ($80,000 fair value - $35,000 mortgage assumed by the partnership). Choice "1" is incorrect. Belger's capital account balance would be the fair market value of the property donated ($80,000) less the mortgage assumed with the property ($35,000). The net contribution is $45,000. Choice "2" is incorrect. The partners' capital account balances equal the fair value of assets contributed less liabilities assumed by the partnership. Choice "4" is incorrect. Algee's capital account balance would be his cash contribution of $50,000.

MCQ-09751 Which of the following statements is most accurate in regards to the capitalization of computer software costs to be licensed? A. Software costs intended to be licensed can only be expensed. B. Amortization begins once technological feasibility is established. C. Coding and testing costs after technological feasibility is established can be capitalized. D. Capitalized software costs are reported on the balance sheet at cost, regardless of market value.

C. Coding and testing costs after technological feasibility is established can be capitalized. Prior to the establishment of technological feasibility, coding and testing costs are expensed. Once technological feasibility is established, coding and testing costs are capitalized. Choice "1" is incorrect. The costs are expensed until the point of technological feasibility, after which they are capitalized. Choice "2" is incorrect. Amortization begins once the product is released for sale (to be licensed). Choice "4" is incorrect. Capitalized software costs are reported at the lower of cost or market (net realizable value).

MCQ-09442 Payback Parish levied an additional one-cent sales tax intended exclusively for the repayment of Civic Center Bonds. The earnings from this sales tax would most appropriately be recorded in the: A. General Fund. B. Special Revenue Fund. C. Debt Service Fund. D. Capital Projects Fund.

C. Debt Service Fund. Earnings from a tax to be used exclusively to repay specific debt would most likely be accounted for in the debt service fund associated with that debt. Fund financial reporting, particularly for the governmental funds, demonstrates compliance with laws rules and regulations. The display of revenues restricted for debt repayment in the debt service fund would demonstrate compliance with the law that levied the tax. Choice "1" is incorrect. The General Fund would not serve to demonstrate the use of the sales tax for repayment of debt as clearly as the debt service fund. Choice "2" is incorrect. The Special Revenue Fund would not serve to demonstrate the use of the sales tax for repayment of debt as clearly as the debt service fund. Choice "4" is incorrect. The Capital Projects Fund accounts for construction monies, not funds anticipated for the repayment of debt. The Capital Projects Fund would not be appropriate.

MCQ-09310 At its date of incorporation, The McCarty Company issued 100,000 shares of its $10 par common stock at $11 per share. During the current year, The McCarty Company acquired 30,000 shares of its common stock at a price of $16 per share and accounted for them by the cost method. Subsequently, these shares were reissued at a price of $12 per share. There have been no other issuances or acquisitions of its own common stock. What effect does the reissuance of the stock have on the following accounts? Retained earning -- Additional paid-in-capital A. Decrease Decrease B. No effect No effect C. Decrease No effect D. No effect No effect

C. Decrease No effect The first transaction is issuing shares. The Journal Entry would be as follows: Dr Cash 1,100,0000* Cr Common stock 1,000,000** Cr APIC from issuance $100,000*** * - 100,000 shares × $11 ** - 100,000 shares × $10 par *** - plug for the difference So this Journal Entry gives us an increase to APIC. The next transaction is acquiring treasury stock under the cost method. The Journal Entry would be as follows: Dr Treasury stock 480,000* Cr Cash 480,000 * - 30,000 shares × $16 No effect on either of the accounts from this Journal Entry. The next transaction is re-issuing the shares that were acquired for the treasury. The Journal Entry would be as follows: (bought them at $16 and sold them at $12) Dr Cash 360,000* Dr Retained earnings 120,000** Cr Treasury stock 480,000*** * - 30,000 × $12 ** - plug *** - got to get rid of it all *So this transaction gives us a decrease to Retained Earnings. Why Retained Earnings and not APIC? If we had some APIC from previous Treasury Stock transactions, we would have used it up before debiting Retained Earnings. But in this case, the only APIC had come from the issuance of stock and not from previous Treasury Stock transactions so we don't have any to use.* *And don't get confused by the fact that there was an increase to APIC in the transactions. The question specifically asked only for the "re-issuance" transaction. In this case, none of the answers contained an "increase" to APIC, but they could have.*

MCQ-01054 Which of the following does not affect an internal service fund's change in net position? A. Interfund transfers out. B. Depreciation expense on its fixed assets. C. Due from other funds. D. Interfund transfers in.

C. Due from other funds. Choice "3" is correct. Due from other funds appears on the balance sheet. They are not a component of changes in net position. Choice "1" is incorrect. Interfund transfers out are deductions from gross revenues in the internal service fund. Interfund transfers affect the results of operations (change in net position). Choice "2" is incorrect. The internal service fund is a proprietary fund and recognizes depreciation expense. Depreciation expense is a component of the change in net position. Choice "4" is incorrect. Interfund transfers in are additions to gross revenues in the internal service fund. Interfund transfers affect the results of operations (change in net position).

MCQ-05610 Kind Nurses Assoc. is a voluntary health and welfare organization. Nurses are paid to visit homes of elderly people and are reimbursed for mileage and supplies. Which of the following items should Kind record as a support activity expense in its statement of functional expense? A. Payment for nurses' supplies. B. Nurses' mileage expense. C. Fundraising costs. D. Payment for nurses' employee benefits.

C. Fundraising costs. Choice "3" is correct. Fundraising expenses would be appropriately classified as supporting activities. Supporting activities generally include fundraising, administrative services, and membership development expenses. Choice "1" is incorrect. Nurses' supplies expenses are not support activities of the Kind Nurses Assoc., they are program services. Supporting activities generally include fundraising, administrative services, and membership development expenses. Program services are directly related to the organization's mission. Kind Nurses Assoc. pays nurses to visit the homes of elderly people and pays reimbursement for mileage and supplies. Choice "2" is incorrect. Nurses' mileage expenses are not support activities of the Kind Nurses Assoc.; they are program services. Supporting activities generally include fundraising, administrative services, and membership development expenses. Program services are directly related to the organization's mission. Kind Nurses Assoc. pays nurses to visit the homes of elderly people and pays reimbursement for mileage and supplies. Choice "4" is incorrect. Payment for nurses' employee benefits is not a support activity of the Kind Nurses Assoc.; it is a program service. Supporting activities generally include fundraising, administrative services, and membership development expenses. Program services are directly related to the organization's mission. Payment for nurses' employee benefits is a component of compensation paid to professionals delivering services.

MCQ-09346 Which of the following statements concerning the accounting for long-term construction contracts is/are incorrect? I. The completed contract method is based on the revenue recognition principle, while the percentage of completion method is based on the matching principle II. Under the completed contract method, gains and losses are recognized only when the contract is completed III. The construction in progress account generally has a higher balance when the percentage of completion method is used IV. When the percentage of completion method is used, profit is recognized based on the ratio of current year costs to total estimated cost A.. I only B. II, III, and IV C. II and IV D. I and III

C. II and IV Statement II is incorrect because, under the completed contract method, gains are recognized at contract completion, but losses are recognized as incurred. Statement IV is incorrect because, under the percentage of completion method, profit is recognized based on the ratio of the cost incurred to date (not current year cost) to the total estimated cost. Statements I and III are both correct statements. Choices "1", "2", and "4" are incorrect, based on the above explanation.

MCQ-04502 When a property dividend is declared and the market value of the property exceeds its book value, the excess A. Increases additional paid-in-capital. B. Decreases additional paid-in-capital. C. Increases net income for the period. D. Decreases net income for the period.

C. Increases net income for the period. Property Dividends (Not on CF) Property dividends are distributions of noncash assets, such as inventories and investment securities. *Property dividends are recorded at fair value on date declared and appear as a DR to RE. Property restated from BV to FV and G/L goes on the IS* *Declaration Date:* DR Retained earnings (Reduces equity) CR Dividends payable (Increase liability) Choice "3" is correct. A property dividend is recorded at the fair value of the property to be distributed. The property has to be adjusted to fair value with the adjustment affecting earnings for the period. Additional paid-in-capital is not affected. Choices "1", "4", and "2" are incorrect, per the above discussion.

MCQ-01030 Sample City has identified the non-major funds within its fund types. In its financial report, Sample City: A. Must include combining financial statements for non-major funds for each fund type in the basic financial statements. B . Must include combining financial statements for non-major funds for each fund type in the required supplementary information. C. May include combining financial statements for non-major funds for each fund type in the supplementary information. D. Must include combining financial statement disclosures for non-major funds for each fund type in the notes to the financial statements.

C. May include combining financial statements for non-major funds for each fund type in the supplementary information. Choice "3" is correct. Sample City may report combining non-major individual fund financial statements in the supplementary information. Reporting the combining fund financial statements is optional. Choices "1", "2", and "4" are incorrect. Sample City's reporting is optional.

Balfour Animal Shelter, a not-for-profit organization, received $10,000 from Agnes Balfour to fund the acquisition of grooming equipment on December 1, Year 1. On February 2, Year 2, the shelter used $5,000 to purchase grooming equipment and on March 15, Year 2, used the remaining $5,000 to purchase more equipment as intended by the bequest. As a result of the above transaction, Balfour Animal Shelter would record the following on its December 31, Year 1, financial statements: A. Conditional revenues with donor restrictions of $10,000. B. Revenue without donor restrictions of $5,000 and net assets of $5,000. C. Net assets with donor restrictions of $10,000. D. Net assets without donor restrictions of $10,000.

C. Net assets with donor restrictions of $10,000. An unconditional contribution of funds that relate to donor stipulations that can be satisfied (purchase of equipment) would result in the recognition of net assets with donor restrictions in Year 1. The release of the assets from restriction upon satisfaction of the donor stipulation would result in a simultaneous increase in net assets without donor restrictions and decrease in net assets with donor restrictions in Year 2. Choice "1" is incorrect. The contribution is unconditional and would be recognized as with donor restrictions. Had the contribution been conditional, it would have been recorded as a refundable advance, a liability. Choice "2" is incorrect. The entire amount of the Year 1 unconditional receipt with donor stipulations would be recorded as with donor restrictions in Year 1. The use of a portion of the assets within 60 days of year-end is irrelevant. Choice "4" is incorrect. The Year 1 unconditional receipt is recorded as with donor restrictions, not without donor restrictions.

MCQ-09340 Purple Corp. acquired treasury shares at an amount greater than their par value, but less than their original issue price. Compared to the cost method of accounting for treasury stock, does the pat value method report a greater amount for additional paid-in capital and a greater amount for retained earnings? Additional paid-in capital -- Retained earnings A. Yes Yes B. Yes No C. No No D. No Yes

C. No No Compared to the cost method for reporting treasury stock, the par value method will report a lower amount for additional paid-in capital (APIC) and the same amount for retained earnings. *The asks for APIC and does not state to separate APIC-CS and APIC-TS so you net them. Remember that APIC has a regular Cr (positive) balance... APIC—CS (1,000) + 600 APIC-TS = (400) Dr (decreased) APIC* Assume: Original Issue 10,000 shares of $10 par value, CS are sold for $15 per share Dr Cash 150,000 Cr Common stock 100,000 (10,000 x $10 par) Cr APIC—CS 50,000 (10,000 x $5) *Cost method - G/L recorded upon reissue* Buy Back Below 200 shares repurchased for $12 per share Dr Treasury stock 2,400 Cr Cash 2,400 *No G/L yet* *Par Value Method - G/L calculated upon repurchase* $15 SP - $12 Cost = $3 Gain increases Equity Dr Treasury stock 2,000 (200 x $10 par) Dr APIC—CS (200 x $5) 1,000 Reverse original entry using $5 gain Cr Cash 2,400 (200 x $12PP) Cr APIC—TS 600 (200 x $3) Increases Equity

*The financial statements of governments have focused on two form of accountability. Government-wide financial statements focus the reader on accountability in which way(s):* *Fiscal Accountability -- Operational Accountability* A. Yes Yes B. No No C. No Yes D. Yes No

C. No Yes Choice "3" is correct. Governmental-wide financial statements focus on operational accountability, and funds focus on fiscal accountability. Choice "1" is incorrect. Funds, not governmental-wide financial statements, focus on fiscal accountability. Choice "2" is incorrect. Governmental-wide financial statements focus on operational accountability. Choice "4" is incorrect. Funds, not governmental-wide financial statements, focus on fiscal accountability. Governmental-wide financial statements focus on operational accountability. This answer is backwards.

Disclosure about the following kinds of risks are required for most financial instruments. Concentration of credit risk -- Market risk A. Yes Yes B. Yes No C. No Yes D. No No

C. No Yes Disclosure of Financial Instruments *Fair value must be disclosed for all financial instruments* for which it is practicable to estimate that value together with the *related carrying amounts. Disclosure of concentrations of credit risk is required.* Credit risk is the possibility of loss from the failure of another party to perform according to the terms of a contract. Disclosure of market risk is encouraged but not required under U.S. GAAP. Under *IFRS, disclosure of market risk (risk of losing money in the stock market) is required.*

*A change in the fair value of a derivative qualified as a cash flow hedge is determined to be either effective in offsetting a change in the hedged item or ineffective in offsetting such a change. How should the effective and ineffective portions of the change in value of a derivative which qualifies as a cash flow hedge be reported in financial statements?* Effective portion in -- Ineffective portion in A. Current income Current income B. Current income Other comprehensive income C. Other comprehensive income Current income D. Other comprehensive income Other comprehensive income

C. Other comprehensive income Current income Changes in (gains and losses on) the effective portion of a cash flow hedge are deferred and reported in "other comprehensive income;" changes in the ineffective portion are reported in current income. Gains and losses which are deferred and reported in "other comprehensive income" must be reclassified and recognized in income in the period(s) in which the hedged item affects income. Derivatives derive their value from other securities. A derivative must have *all three* of the following characteristics: 1. One or more underlyings (Stock price, commodity price, interest rates), *and one or more notional amounts (Used to calculate G/L) or payment provisions (or both);* and 2. No initial net investment (or smaller than would be expected); and 3. Its terms *require or permit a net settlement.*

MCQ-09410 The Cox Foundation for the Performing Arts was established to support and coordinate the fundraising appeals of the Little Tutu Dance Troupe and the Expressive Community Theater. The bylaws require thatthe governing body of the foundation include equal representation from the dance troupe and the theater consisting of two thirds of the foundation's board and that the foundation distribute current period collections in the subsequent year. Noted philanthropist L.L. Thayer contributes $250,000 to the foundation on July 1, Year 1, and states in his bequest that the entire amount of the contribution is meant to support the operations of the dance troupe. On its December 31, Year 1, financial statements, the Little Tutu Dance Troupe should: A. Only disclose the gift to the foundation. B. Record a $250,000 increase in net assets without donor restrictions and change in interest in the net assets of the foundation. C. Record a $250,000 increase in net assets with donor restrictions and change in interest in the net assets of the foundation. D. Record $250,000 in contribution revenue without donor restrictions.

C. Record a $250,000 increase in net assets with donor restrictions and change in interest in the net assets of the foundation.

MCQ-09359 The Town of Holler has a community development block grant that funds the renewal and revival of economically disadvantaged areas in its jurisdiction. The Town properly accounts for the grant in a special revenue fund. Which one of the following funds would not share the use of the current financial resources measurement focus with the Community Development Block Grant Fund? A. The Holler Bridge Fund (used to account for the bridge construction financed by general obligation debt). B. TheHollerBridgeDebtServiceFund(used to account for the accumulation of resources for the repayment of general obligation debt issued to finance the construction of the Holler Bridge). C. The Holler Sound Citizen's Initiative Fund (used to account for collection and disbursement of special assessments that pay debt service obligations which the town is barred by statute from assuming). D. The Holler Cemetery Perpetual Care Fund (that accounts for resources used for the public cemetery).

C. The Holler Sound Citizen's Initiative Fund (used to account for collection and disbursement of special assessments that pay debt service obligations which the town is barred by statute from assuming). The Holler Sound Citizen's Initiative Fund is a custodial fund that would account for the debt service associated with a special assessment for which the municipality had no obligation. A custodial fund is accounted for using the economic resources measurement focus and would not share the same basis of accounting with the Community Development Block Grant Fund. Choice "1" is incorrect. The Holler Bridge fund is a capital projects fund that would use the current financial resources measurement focus in the same manner as the special revenue fund used to account for the community development block grant. Choice "2" is incorrect. The Holler Bridge Debt Service Fund is a debt service fund that would use the current financial resources measurement focus in the same manner as the special revenue fund used to account for the community development block grant. Choice "4" is incorrect. The Holler Cemetery Perpetual Care Fund is a permanent fund established to benefit the general public and would use the current financial resources measurement focus in the same manner as the special revenue fund used to account for the community development block grant. *"P"ermanent:* Permanent funds are used to report resources that are legally restricted to the extent that income, and not principal, may be used for purposes that support the reporting government's programs.

MCQ-09374 The City of Riggsville elected to account for its very small fee- supported trash collection service through its General Fund. Payment for services provided in December, for which payment is not anticipated until March, has not been accrued. The trash collection service shares the same basis of accounting with: A. The Riggsville Utility Fund, an enterprise fund supported by user charges. B. The Riggsville Motor Pool, an internal service fund supported by internal user charges. C. The Riggsville Road and Bridge Fund, used to account for the receipt of gasoline tax funds in a Special Revenue Fund. D. The Riggsville Employee Retirement Fund, used to account for the pension fund resources that benefit the city's employees.

C. The Riggsville Road and Bridge Fund, used to account for the receipt of gasoline tax funds in a Special Revenue Fund. The trash collection service is accounted for in the General Fund, a governmental fund that uses the modified accrual basis of accounting, the basis of accounting that will not recognize revenues until measurable (amount is defined) and available (amount is collected within 60 days of year-end). The Road and Bridge Fund uses the same basis of accounting as the trash collection service. Choice "1" is incorrect. The utility fund is an enterprise fund that uses the full accrual basis of accounting that recognizes revenue when earned. Choice "2" is incorrect. The motor pool is an internal service fund that uses the full accrual basis of accounting that recognizes revenue when earned. Choice "4" is incorrect. The retirement fund is a pension fund that uses the full accrual basis of accounting that recognizes revenue when earned.

MCQ-01069 *In applying the criteria used for determination of major funds required for reporting in a government's fund financial statements, a government would consider which of the following statistics?* I .Aggregate Revenues or Expenditures/Expenses: II. Aggregate Assets or Liabilities: III. Aggregate Fund Balance/Equity: I. II. III. A. No Yes Yes B. Yes No NO C. Yes Yes No D. Yes Yes Yes

C. Yes Yes No Rule: The criteria for determining major funds includes qualification as to revenues, expenditures/expenses, assets, or liabilities that are at least 10 percent of the associated total for ALL governmental OR enterprise (as appropriate) AND at least 5 percent of the total of the associated totals for ALL governmental AND enterprise funds. Choice "3" is correct. Governments evaluate the classification of major funds using aggregate revenues and expenditures/expenses and aggregate assets and liabilities. Choices "4", "2", and "1" are incorrect. Governments evaluate the classification of major funds using aggregate revenues and expenditures/expenses and aggregate assets and liabilities per the rule above, not fund balance/equity.

MCQ-09380 Star Co. leases a building to be used for office space to support its expanding business. The sixyear nonrenewable lease will expire on December 31, year 8. In January, year 5, Star made leasehold improvements of $72,000. The estimated useful life of the improvements is 10 years. Star uses the straight-line method of amortization. What amount of amortization should Star report on its June 30, year 5 income statement? A.$3,600 B.$7,200 C.$9,000 D.$18,000

C.$9,000 *The leasehold improvements should be amortized over the lesser of the remaining lease term or the life of the improvements.* In this case, the four-year remaining life of the lease is shorter than the 10-year life of the improvements. $72,000 / 4 = $18,000 for a full year's amortization, so the January 1−June 30 amortization is $18,000 / 2 = $9,000. Choice "1" is incorrect. This answer incorrectly uses the 10-year estimated life of the improvements rather than the remaining lease term to calculate the amortization. Leasehold improvements should be amortized over the lesser of the remaining lease term or the life of the improvements. Choice "2" is incorrect. This answer incorrectly uses the 10-year estimated life of the improvements rather than the remaining lease term to calculate the amortization. In addition, this is a full year's amortization using the 10-year life. In Year 5, the asset should only be amortized over the six months beginning January 1 and ending June 30. Choice "4" is incorrect. This is a full year's amortization. In Year 5, the asset should only be amortized over the six months beginning January 1 and ending June 30.

Dividends and Stock Splits

Cash Dividends (Financing outflow on CF) Cash dividends may be declared on common and/or preferred stock. Relevant dates for cash dividends are: *Declaration Date:* DR Retained earnings (Reduces equity) CR Dividends payable (Increase liability) Record Date: The date the stockholders must own the stock in order to receive the dividend declared. *No entry* Payment Date: The date the dividends are actually disbursed. DR Dividends payable (Reduces liability) CR Cash (Now hits theFinancing outflow on CF) Property Dividends (Not on CF) Property dividends are distributions of noncash assets, such as inventories and investment securities. *Property dividends are recorded at fair value on date declared and appear as a DR to RE. Property restated from BV to FV and G/L goes on the IS* *Declaration Date:* DR Retained earnings (Reduces equity) CR Dividends payable (Increase liability) *Stock Dividends (Not on CF)* A stock dividend of *less than 20-25 percent* of the outstanding capital stock is recorded at *fair value (small dividend).* *Total stockholders' equity does not change it only changes form bc APIC increases and RE decreases.* Dr RE (FV) [decreases RE] Cr CS (Par) [increases CS] Cr APIC [increases APIC] A stock dividend of *greater than 20-25 percent* of the capital stock is recorded at *par value (large dividend).* *Total stockholders' equity does not change it only changes form bc APIC increases and RE decreases.* Dr RE (Par) [decreases RE] Cr CS (Par) [increases CS] Liquidating Dividends (No JE): A liquidating dividend is a dividend in excess of retained earnings. Stock Splits (No JE) : In a stock split, the number of shares outstanding is increased, and the par value is decreased. (In a reverse split, the opposite is true.) There is no change in the total book value of shares outstanding and a memo entry is used to acknowledge a stock split. *Number of shares issued increases, the par value decreases = No change to BV of Stated Capital *Recipient of stock dividends and stock splits recognizes no income. The basis of each share of stock is adjusted accordingly.* Stock Rights/Warrants Stock rights/warrants give the holder the right to acquire shares of stock on the payment of a defined amount. *A memo entry is made when the rights are issued.* *Under US GAAP if non-compensatory no journal entry until stock is purchased.* Then you do a regular journal entry. *US GAAP & IFRS - Compensatory* -*On the grant date the date option issued no JE, memo only.* -*Compensation expense calculated using FV estimate per Black-Scholes model and allocated over the besting (service) period in accordance with matching principle.*

Financing Activities

Cash flow activities that include (a) obtaining cash from issuing debt and repaying the amounts borrowed and (b) obtaining cash from stockholders, repurchasing shares, and paying dividends. -*Relate to long term liabilities (with interest paid) and owner equity.* -Issue of equity securities to raise capital i.e., CS -Repurchase treasury stock -Issuance of debt i.e., bonds, notes, debentures (anything with interest paid) - *Pay CASH dividend* -*Receipts from contributions restricted for the purpose of acquiring, constructing, or improving PPE and other long-lived assets.* (NFP)

*Program Revenue Category Types*

Charges for Services: Revenues based on exchange or exchange-like transactions, including: - Charges for services to customers or applicants who directly benefit from goods or services (e.g., water and sewer fees, licenses, building permits, special assessment, etc.,) - Charge for services to other governments (e.g., charges to housing prisoners, etc.) - Fines and forfeitures Operating Grants and Contributions: Mandatory and voluntary non-exchange transactions with other governments, organizations, or individuals restricted for use in a particular program. - Grant revenues in support of specific programs Capital Grants and Contributions: Mandatory and voluntary non-exchange transactions with other governments, organizations, or individuals restricted for use in a particular program. - Grant revenues in support of specific programs General Revenues: General revenues are presented separately in the same three categories and a total column as above. General revenues include taxes, interest earnings, and other revenues that are not specifically associated with a functional expense.

MCQ-00416 *Mega, Inc. was organized to consolidate the resources of Lone Co. and Small Co. in a business combination accounted for by the acquisition method. Mega issued 31,000 shares of its $10 par voting stock with a fair value of $15 per share, in exchange for all the outstanding capital stock of Lone and Small. The equity accounts of Lone and Small on the date of the exchange were:* *Common stock, at par Additional paid-in capital Retained earnings* *Lone -- Small -- Total* *$100,000 $200,000 $300,000* *12,500 17,500 30,000* *60,000 105,000 165,000* *$172,500 $322,500 $495,000* *What is the balance in Mega's additional paid-in capital account immediately after the business combination?* A. $30,000 B. $155,000 C. $12,500 D. $17,500

Choice "2" is correct. The transaction is recorded at the fair market value of the stock issued. The journal entry would be: Dr Investment in sub 465,000 (31,000 x $15 FV) Cr Common stock (par) 310,000 (31,000 x $10 par) Cr APIC 155,000 Choice "1" is incorrect. The combined APIC of Lone and Small is not the correct amount. Choice "3" is incorrect. Lone's APIC is not the correct amount. Choice "4" is incorrect. Small's APIC is not the correct amount.

MCQ-00326 *Pare Inc. purchased 10% of Tot Co.'s 100,000 outstanding shares of common stock on January 2, Year 1, for $50,000. On December 31, Year 1, Pare purchased an additional 20,000 shares of Tot for $150,000. There was no goodwill as a result of either acquisition, and Tot had not issued any additional stock during Year 1. Tot reported earnings of $300,000 for Year 1. What amount should Pare report in its December 31, Year 1, balance sheet as investment in Tot?* A. $170,000 B. $230,000 C. $200,000 D. $290,000

Choice "3" is correct: $200,000 investment in Tot at December 31, Year 1. Rule: When two or more purchases of stock cause ownership in an investee to go from less than 20% to more than 20%, the cost of acquiring the additional interest in the investee is added to the carrying value of the investment and the equity method is adopted as of the date that significant influence is acquired and going forward. On December 31, Year 1, the total carrying value of the investment in Tot is $200,000 ($50,000 original investment + $150,000 cost of additional shares to acquire significant influence). Pare will use the equity method starting on December 31, Year 1, and will add its share of Tot's earnings to the investment in subsidiary beginning in Year 2. *With 10% ownership, you make the assumption that they don't assert significant influence (between 20-50%). Therefore the FV method is used until the ownership % is within the 20-50% threshold. Given the triggering of significant influence occurring at YE, the change occurs prospectively and the subsidiary's earnings, dividends, & amortization increases/decreases the investment each year after this triggering.*

MCQ-09757 Vendor X sells a high-end furniture set to customer Y in exchange for a three year not receivable bearing no interest rate. Although prevailing market rates show that an appropriate interest rate for the transaction is 5 percent, the parties agree that a more reasonable rate for this specific transaction is 6 percent. Relative to a 5 percent interest rate, the 6 percent implied rate will result in: A. An increase in sales booked. B. A decrease in annual interest income. C. A decrease in the initial notes receivable asset. D. An increase in the annual amortization of the discount on notes payable.

Choice "4" is correct. When the initial transaction takes place, the accounting entry (from the vendor's perspective) is a debit to notes receivable for the full amount, a credit to sales for the present value of the note receivable, and a credit to the discount on notes receivable for the difference. Dr Notes receivable $XXX Cr Sales (PV of notes receivable) $XXX Cr Discount on notes receivable (plug) $XXX The higher the implied interest rate is, the lower the present value of the note is and the higher the discount amount is. As the discount is amortized over the three years, a higher initial discount amount will result in higher annual amortization. Choice "1" is incorrect. Because the sales total is equal to the present value of the note receivable, a higher interest rate will result in a lower present value and therefore a lower sales amount. Choice "2" is incorrect. Interest income is booked as the credit to match the debit for the annual amortization of the discount. Dr Amortization of discount on notes receivable $XXX Cr Interest income $XXX Since a higher interest rate results in a higher discount (and higher annual discount amortization), interest income will also be higher when the rate is higher.

Capital Stock (Common stock & preferred stock)

Common stock (CS) normally has a par (or stated) value, for instance, CS $1 par value issued at $5 per share. Dr Cash $5 Cr Common stock $1 Cr APIC—CS 4 Preferred stock (PS) may be *cumulative, noncumulative, or participating* as to the payment of dividends and generally has a preferred claim to assets on liquidation of the business. PS $10 par value is issued at $30. Dr Cash $30 Cr Preferred stock $10 Cr APIC-PS 20 Cumulative *Cumulative preferred stockholders receive current dividends and all dividends in arrears (unpaid from prior years) before dividends are paid to common stockholders.* Participating- Share equally then "pro-rata" Any amounts available for dividend distribution after both preferred and common stockholders receive a specified payment based on the same percentage is divided between preferred and common stockholders on a pro rata basis. Mandatorily Redeemable Mandatorily redeemable preferred stock is classified as a liability because it has a maturity date, similar to debt instruments.

*Long-Term Construction Contracts*

Completed Contract Method *US GAAP = Maybe can be used if the percent of completion method cannot be estimated IFRS = No* The completed contract method is used when a long-term construction contract does not meet the criteria for recognizing revenue over time. Dr Construction in progress $XXX Dr Cash/accounts payable $XXX Cr Accounts receivable $XXX Cr Progress billings $XXX *Losses (100 percent) for the completed contract method are recognized in full as they are discovered.* The completed-contract method is prohibited under IFRS. *Percentage of Completion Method = Full accrual* When a long-term construction contract meets the criteria for recognizing revenue over time, it is appropriate to use the percentage-of-completion method if the *entity can reasonably estimate profitability and can provide a reasonable measure of progress toward completion.* Revenue/income is recognized based on the ratio of the cost incurred to date to the total estimated cost. Under IFRS, if the final outcome of the project cannot be reliably measured, then the cost recovery method is required. *Losses for the percentage of completion method are recognized in full as they are discovered (e.g., 100 percent).* Dr Construction in progress $XXX Cr Cash/accounts payable XXX Dr Accounts receivable XXX Cr Progress billings XXX Dr Construction in progress XXX Cr Current gross profit XXX *Calculation of Current Gross Profit* *Step 1* *Contract price - Total estimated cost = Total gross profit* *Step 2* *Cost to date / Total estimated cost = % Completed* *Step 3* *Step 1 × Step 2 = GP earned to date* *Step 4* *GP earned to date - GP previously recognized = Current GP*

*Business Combinations*

Consolidation Consolidated financial statements are prepared when a parent-subsidiary relationship has been formed. An investor is considered to have parent status when more than *50 percent of the voting stock of the investee has been acquired.* Do not consolidate when subsidiary is in legal reorganization or bankruptcy (parent does not control the subsidiary). 0%-20% = Fair value method 20%-50% = Equity method >50% = Consolidation Acquisition Method In a business combination accounted for as an acquisition, the subsidiary may be acquired for cash, stock, debt securities, etc. The investment is valued at the fair value of the consideration given or the fair value of the consideration received, whichever is the more clearly evident. The accounting for an acquisition begins at the date of acquisition. Acquisition Method Summary Assets | 100% Fair value Liabilities | 100% Fair value Retained earnings | Parent only Income | After acquisition date Goodwill | Yes (subject to impairment adjustment) Noncontrolling interest | Yes (up to 49%) Fair value Investment in subsidiary | Eliminated Intercompany transactions | Eliminate 100% Consolidating Workpaper Eliminating Journal Entry The year-end consolidating journal entry known as the consolidating workpaper eliminating journal entry (EJE) is: Dr Common stock—subsidiary $XXX DR APIC—subsidiary XXX DR Retained earnings—subsidiary XXX (The three Dr eliminate old NBV of sub purchased) CR Investment in subsidiary XXX CR Noncontrolling interest XXX (The two Cr are the purchase of sub at FV) DR Balance sheet adjusted to fair value XXX DR Identifiable intangible asset fair value XXX DR Goodwill XXX (Adjustments to BS) *The consolidated balance sheet will report the equity of the parent company only. The parent's investment in the subsidiary is eliminated.*

*Contingent Losses and Contingent Gains*

Contingent Losses The recognition and presentation of loss contingencies depends on the classification of the contingency. Probable/Expects: *Likely to occur, and settlement can be estimated. An adjusting entry and a note disclosure are required for the probable/expected amount.* "The amount of the loss is reasonably estimated": If a range of amounts is given, adjust for the *"smaller"* amount on the balance sheet and disclose the difference in the notes to the financial statements. *In the event that a range of probable losses is given (e.g., $100,000 to $250,000), GAAP requires that the best estimate of the loss be accrued. If no amount in the range is a better estimate than any other amount within the range, the minimum amount within the range should be accrued (in this case $100,000), and a note disclosing the possibility of an additional $150,000 loss should be presented.* Reasonably Possible: A *note disclosure* is required only. Remote: *Ignore (unless guarantee of indebtedness of others, then disclose).* *Contingent Gains - No JE* Contingent gains that are probable or reasonably possible may be disclosed in the notes to the financial statements.

MCQ-00349 Day Co. received dividends from its common stock investments during the year ended December 31, Year 1, as follows: A stock dividend of 400 shares from Parr Corp. on July 25, Year 1, when the market price of Parr's shares was $20 per share. Day owns less than 1% of Parr's common stock. A cash dividend of $15,000 from Lark Corp. in which Day owns a 25% interest. A majority of Lark's directors are also directors of Day. What amount of dividend revenue should Day report in its Year 1 income statement? A. $15,000 B. $8,000 C. $23,000 D. $0

D. $0 Choice "4" is correct, $0 dividend revenue. Rules: 1. Stock dividend (more shares of stock) is not reported as revenue, only a memo entry is made. 2. Cash dividend (under the equity method) reduces the investment account but does not affect revenue.

MCQ-09675 *Hi-Tech Corp. spent $300,000 on research and development to generate new product lines. Only one of the five product lines resulted in a patented item, while the remaining four were considered unsuccessful. The cost of the product that was successfully patented included $30,000 in research costs and $40,000 in development costs. Under U.S. GAAP, how much of the $300,000 should be recognized as an expense?* A. $300,000 B. $240,000 C. $60,000 D. $0

D. $0 The entire $300,000 should be expensed because U.S. GAAP requires that R&D expenditures be expensed. *U.S. GAAP = Generally expensed in period incurred* Under U.S. GAAP, research and development costs must be expensed in the period incurred. In general, items to be expensed as R&D include: equipment, material, labor, overhead, design, testing, engineering, modification, and salaries of research staff. Exceptions to expensing include: *1. Software after feasibility are capitalized, not expensed* *2. PPE has alternative future use capitalized, not expensed* IFRS Under IFRS, research costs must be expensed like U.S. GAAP, but development after feasibility costs may be capitalized if certain criteria are met.

MCQ-09436 *During the year ended December 31, Year 0, Rich Giver notified the Salvation Church that he had remembered the Church in his will with a restricted bequest for the construction of an addition to the sanctuary conditioned upon acquisition of property adjoining the church, the site of the sanctuary expansion. Rich Giver dies in Year 3 and the probate court validates the will in Year 4 indicating that Rich Giver allocated $500,000 to the church. Salvation Church had not secured the property. Salvation Church would recognize the following revenue with donor restrictions in each year of the following years:* Year 0 -- Year 3 -- Year 4 A. $0 $0 $500,000 B. $0 $500,000 $0 C. $500,000 $0 $0 D. $0 $0 $0

D. $0 $0 $0 A conditional promise to give is not recognized as revenue until all of the pertinent conditions are met. Since Salvation Church has not met the condition of the bequest, it would not recognize any earnings in any year. Choice "1" is incorrect. Salvation Church would not recognize contribution revenue upon ratification of the will by the probate court since the contribution is subject to an unmet condition. Contribution revenue is recognized when collection is assured and conditions, if any, have been substantially met. Choice "2" is incorrect. The death of the donor does not assure the church of the contribution. Ratification of the will along with satisfaction of conditions assures the church of collection. No revenue would be recognized in Year 3. Choice "3" is incorrect. The inclusion of the church in the will does not assure the church of collection of the bequest. The will could be challenged or the will could be altered prior to the donor's death. In addition, the church must meet the conditions established by the will to recognize and collect the contribution.

MCQ-09439 Thayer Healthcare Foundation was formed to raise money for the Thayer Healthcare system, a group of three otherwise unrelated entities: a hospital, nursing home and walk-in clinic. The nursing home and walk in clinic have nearly equal net assets while the hospital's net assets are nearly three times the size of the combined net assets of all other system members. None of the entities that benefit from the foundation's activities have an agreement regarding the distribution of foundation earnings. During the year ended December 31, Year 1, AB Corporation donated $600,000 to the foundation. Relative to this transaction, entities that compromise the health care system would record in the net assets of the foundation as follows: Hospital -- Nursing Home -- Walk-in-Clinic A. $450,000 $75,000 $75,000 B. $200,000 $200,000 $200,000 C. $600,000 $0 $0 D. $0 $0 $0

D. $0 $0 $0 The unrestricted gift to the foundation serves to increase the net assets of the foundation, however, absent any agreement amongst the beneficiary organizations regarding the distribution of the foundation's resources or stipulations by the donor, none of the entities would include the interest in net assets resulting from the gift in their financial statements. Choice "1" is incorrect. Absent any agreement between the parties or a donor restriction, none of the entities record an interest in the net assets of the foundation as a result of the gift. This incorrect choice not only presumes recognition but also assumes distribution in accordance with relative net asset value, an incorrect presumption. Choice "2" is incorrect. Absent any agreement between the parties or a donor restriction, none of the entities record an interest in the net assets of the foundation as a result of the gift. This incorrect choice not only presumes recognition but also assumes equal distribution of the interest in the absence of an agreement, an incorrect presumption. Choice "3" is incorrect. Absent any agreement between the parties or a donor restriction, none of the entities record an interest in the net assets of the foundation as a result of the gift. This incorrect choice not only presumes recognition but also assumes distribution of the interest in the absence of an agreement to the entity with the largest net asset balance, an incorrect presumption.

MCQ-05618 On January 1, Year 1, Paul Corporation acquired 80% of Saul Corporation's 200,000 shares of the outstanding common stock for $5,000,000. On the date of acquisition, the $6,000,000 book value of Saul's net assets equaled the fair value of Saul's net assets. During Year 1, Saul reported net income of $550,000 and paid dividends of $165,000. What is the noncontrolling interest that will be reported on Paul Corporation's December 31, Year 1 consolidated balance sheet? A. $1,250,000 B. $1,200,000 C. $1,277,000 D. $1,327,000

D. $1,327,000 Choice "4" is correct. The noncontrolling interest to be reported on the December 31, Year 1 balance sheet is equal to the noncontrolling interest on January 1, Year 1, adjusted for the noncontrolling shareholders' share of Saul's Year 1 income and dividends. Noncontrolling interest on January 1, Year 1 is calculated as follows: Fair value of Saul × 80% = Acquisition priceFair value of Saul × 80% = $5,000,000 Fair value of Saul = $5,000,000 / 80% = $6,250,000 Using the total fair value of Saul, the noncontrolling interest in Saul on the acquisition date can be calculated: Fair value of Saul x 20% = Noncontrolling interest$6,250,000 × 20% = $1,250,000 The noncontrolling interest in Saul on December 31, Year 1 is: Beginning noncontrolling interest 1,250,000 + 20% of Saul's net income 110,000 − 20% of Saul's dividends (33,000) Ending noncontrolling interest 1,327,000 Choice "1" is incorrect. This is the fair value of the noncontrolling interest on January 1, Year 1. The question asks for the year-end value. Choice "2" is incorrect. This is the book value of the noncontrolling interest on January 1, Year 1. The noncontrolling interest is recognized at fair value. Choice "3" is incorrect. This is the book value of the noncontrolling interest on January 1, Year 1, adjusted for the noncontrolling shareholders' share of Saul's Year 1 income and dividends. Noncontrolling interest is recognized at fair value, not book value.

MCQ-09321 *On July 1, year 1, after recording interest and amortization, Wake Company's shareholders converted $1,000,000 of its 10% convertible bonds into 50,000 shares of its $1 par value common stock. On the conversion date, the carrying amount of the bonds was $1,500,000, the market value of the bonds was $1,400,000, and Wake's common stock was publicly trading at$40 per share. Using the *book value method*, what amount of *additional paid-in capital* should Wake record as a result of the conversion?* A. $500,000 B. $1,500,000 C. $1,950,000 D. $1,450,000

D. $1,450,000 In this question, they are asking about a bond conversion, and they want to know how much additional paid-in capital would be recorded using the book value method. Using the book value method, the common stock is recorded at the carrying amount of the converted bonds, less any conversion expenses. Since there are no conversion expenses in this question, the common stock is recorded at the $1,500,000 carrying amount of the converted bonds. *The par value of the stock issued is $50,000 (50,000 shares × $1) so additional paid-in capital of $1,450,000 ($1,500,000 − $50,000) is recorded.* The $1,500,000 carrying value of the bonds consists of $1,000,000 face value and a premium of $500,000. The journal entry is (Same treatment as retirement of BP): Dr Bond payable 1,000,000 Dr Premium on bonds 500,000 (1,500,000 Current CV - 1,000,000 BP issue) Cr Common stock 50,000 (50,000 CS shares x $1 par) Cr APIC 1,450,000 ($1,500,000 CV BP − $50,000) *Note that when the book value method is used, the $40 fair value of the stock is not considered, and no gain or loss is recognized.*

MCQ-09390 During Year 1, Innovative Technologies Corp. spent $900,000 developing a product which was granted a patent on June 30, Year 1. The company paid $20,000 in legal and other feees related to the patent registration process. On January 1, Year 2, the company paid $76,000 in legal fees related to the successful defense of the patent in a patent-infringement lawsuit brought by the company's main competitor. The patent's legal life is 17 years and its estimated economic life is 10 years. What will Innovative technologies report as amortization expense related to the patent on its December 31, Year 2, income statement if the company uses U.S. GAAP? A. $1,000 B. $2,000 C. $9,500 D. $10,000

D. $10,000 Under U.S. GAAP, the $900,000 spent to develop the product is considered research and development and must be expensed when incurred. However, the legal and other patent registration fees can be capitalized. Therefore, when the patent is granted on June 30, Year 1, the company will record an intangible asset in the amount of $20,000 which will be amortized over 10 years (the lesser of the patent's legal and economic lives). In Year 1, patent amortization will total $1,000 ($20,000/10 × 6/12), resulting in a 12/31/Y1 net book value of $19,000 ($20,000 cost − $1,000 amortization). When the legal fees related to the successful defense of the patent are recorded on January 1, Year 2, they will be capitalized and added to the book value of the patent: Beginning book value 19,000 - Legal fees related to successful defense 76,000 = 95,000 This amount will then be amortized over the remaining 9.5 year life of the patent on a straight line basis: $95,000 / 9.5 years = $10,000

MCQ-06458 *Penn Corp. paid $300,000 for 75 percent of the outstanding common stock of Star Co. At that time, Star had the following condensed balance sheet,* *Carrying Amounts:* *Current assets $40,000* *Plant and equipment, net 380,000* *Liabilities 200,000* *Stockholders' equity 220,000* *The fair value of the plant and equipment was $60,000 more than its recorded carrying amount. The fair value and carrying amounts were equal for all other assets and liabilities.* *What amount of goodwill related to Star's acquisition should Penn report on its consolidated balance sheet under U.S. GAAP?* A. $20,000 B. $40,000 C. $90,000 D. $120,000

D. $120,000 Choice "d" is correct. Under U.S. GAAP, goodwill is calculated as follows: Goodwill: Fair value of subsidiary - Fair value of subsidiary's net assets The fair value of the subsidiary is: FV of subsidiary x 75% = 300,000 300,000 / 0.75 = 400,000 FV of subsidiary = 400,000 The fair value of the subsidiary's net assets is: FV os subsidiary's net assets = $40,000 + $380,000 +$60,000 - $200,000 =$280,000 Therefore, the goodwill to be reported under U.S. GAAP is: Goodwill = $400,000 - $280,000 =$120,000

MCQ-10955 *On December 31, an entity's reporting unit had a net carrying value of $500,000. For the reporting unit, the entity determined the following:* *Fair value $480,000* *Value in use 475,000* *Goodwill 15,000* *What is the goodwill impairment loss that will be reported on the December 31 income statement under GAAP?* A. $20,000 B. $25,000 C. $5,000 D. $15,000

D. $15,000 *US GAAP Goodwill Impairment* One-step approach: CV reporting unit < FV including Goodwill CV reporting unit - FV including Goodwill = Impairment loss The max charge is the amount of CV of goodwill. To calculate goodwill impairment under GAAP, the carrying amount of the reporting unit is compared with the fair value of the reporting unit. If the carrying amount exceeds the fair value, there is an impairment loss. The loss will be equal to the difference between the carrying amount and the fair value, not to exceed the amount of goodwill currently reflected on the balance sheet. The carrying amount of $500,000 exceeds the fair value of $480,000 by $20,000. Because this amount exceeds the goodwill amount of $15,000, the impairment loss will be equal to $15,000. Choice "1" is incorrect. $20,000 is the calculated impairment, but only $15,000 is recognized because of the amount of goodwill currently on the balance sheet. Choice "2 is incorrect. The value in use is only relevant for IFRS calculations. Choice "3" is incorrect. $5,000 represents the amount of the calculated impairment that cannot be recognized.

MCQ-10953 *A company is completing its annual impairment analysis of the goodwill included in one of its reporting units. The company noted the following related to the reporting unit:* *Goodwill -- Patents -- Other -- Assets Total* *Historical cost $15,000 $10,000 $35,000 $60,000* *Depreciation and amortization 0 3,333 11,667 15,000* *Carrying amount, Dec. 31 $15,000 $6,667 $23,333 $45,000* *The fair value of the reporting unit, including goodwill, is $28,000. Under GAAP, the calculated impairment loss to be booked against goodwill is closest to:* A. $10,333. B. $17,000. C. $7,000. D. $15,000.

D. $15,000. Choice "4" is correct. To calculate goodwill impairment under GAAP, the carrying amount of the reporting unit is compared with the fair value of the reporting unit. If the carrying amount exceeds the fair value, there is an impairment loss. The loss will be equal to the difference between the carrying amount and the fair value, not to exceed the amount of goodwill currently reflected on the balance sheet. The carrying amount of $45,000 exceeds the fair value of $28,000 by $17,000. *With goodwill currently on the books at $15,000, the impairment loss is capped at $15,000.* Choice "1" is incorrect. $10,333 is equal to the fair value of $28,000 relative to the carrying value of $45,000 reduced by the carrying amount of the patents of $6,667. Choice "2" is incorrect. $17,000 is the difference between carrying and fair value. However, this amount exceeds the goodwill amount on the books of $15,000. Choice "3" is incorrect. $7,000 represents the difference of $17,000 noted earlier, reduced by the historical cost of the patents.

MCQ-09264 The following information pertains to Burnel Corporation's defined benefit pension plan for Year 1: Service cost $160,000 Actual and expected gain on plan assets 35,000 Unexpected loss on pension plan assets related to a Year 1 disposal of a subsidiary 40,000 Amortization of unrecognized prior service cost 5,000 Annual interest on pension obligation 50,000 What is Burnel's total net periodic pension cost for the period? A. $250,000 B. $220,000 C. $210,000 D. $180,000

D. $180,000 What should come to mind immediately are those items that make up the formula for net periodic pension cost. A number of those very items are listed in the facts of the question, and the major difficulty is remembering which of those items should be added and which should be subtracted. + 160,000 *S*ervice cost (current item - goes to compensation) + 50,000 *I*nterest cost (on the projected benefit obligation) − 35,000*R*eturn on plan assets (expected or actual) + 5,000 *A*mortization of unrecognized prior service cost ± (*G*ains) and losses ± Amortization of *E*xisting net (asset) or obligation = $180,000 Net periodic pension cost Start with the service cost of $160,000. Add the $50,000 of annual interest on the pension obligation, and that gives $210,000, a possible answer. The actual and expected gain of $35,000 would reduce the $210,000 to $175,000, and the $5,000 amortization of unrecognized prior service cost would add to it to produce $180,000. The $40,000 loss on plan assets related to the disposal of a subsidiary in Year 1 looks like it might be a factor, but that loss would have been included in the loss on the disposal of the subsidiary (probably in discontinued operations) and not the net periodic pension cost (if it had been added, the total would have $220,000).

MCQ-09683 *On January 1, Year 1, Sunshine Corporation acquired an 80% ownership interest in Grey Sky Enterprises by purchasing 400,000 of Grey Sky's 500,000 voting common shares outstanding for $10,000,000. Additional information regarding Grey Sky as of January 1, Year 1, follows:* Book Value Fair Value Net assets $11,200,000 $11,500,000 *On the acquisition date, what is the noncontrolling interest that will be reported on the consolidated balance sheet of Sunshine Corporation under U.S. GAAP?* A. $1,200,000 B. $1,500,000 C. $2,300,000 D. $2,500,000

D. $2,500,000 Under the U.S. GAAP, noncontrolling interest on the balance sheet is calculated as follows: NCI = *100% Fair value of subsidiary* x % NCI The fair value of the subsidiary is calculated as follows: FV subsidiary x 80% = $10,000,000 FV subsidiary = $12,500,000 Therefore, the NCI is: NCI = $12,500,000 x 20% = $2,500,000 Note that the NCI can also be calculated as the difference between the fair value of the subsidiary and the acquisition cost: $12,500,000 FV - $10,000,000 acquisition cost = $2,500,000 NCI Under IFRS, noncontrolling interest (and goodwill) can be calculated using either the full goodwill method, which is the method required under U.S. GAAP, or the partial goodwill method. Under the partial goodwill method, noncontrolling interest on the balance sheet is calculated as: Noncontrolling interest (BS) = *100% Fair value of subsidiary's net assets* × % Noncontrolling interest $11,500,000 FV x (1.0 - 0.80) = $2,300,000 NCI

MCQ-09348 On December 31, Saxon Corporation was merged into Philadelphia Corporation. In the business combination, Philadelphia issued 200,000 shares of its $10 par value common stock, with a market price of $18 a share, for all of Saxon's common stock. The stockholders' equity section of each company's balance sheet immediately before the combination was: Philadelphia -- Saxon Common Stock $3,000,000 $1,500,000 Additional paid-in capital 1,300,000 150,000 Retained earnings 2,500,000 850,000 Total $6,800,000 $2,500,000 In the December 31 consolidated balance sheet, additional paid-in capital should be reported at: A. $950,000 B. $1,300,000 C. $1,450,000 D. $2,900,000

D. $2,900,000 Philadelphia's JE to record purchase of Saxon Corp.: Dr Equity investment-Saxon (200,000 x $18 Mkt) 3,600,000 Cr CS (200,000 x $10 par) 2,00,000 Cr APIC 1,600,000 Beg. APIC 1,300,000 APIC 1,600,00 = 2,900,000 The conveniently named Philadelphia Corporation (P as in parent) has acquired the Saxon Corporation (S as in subsidiary) in a business combination transaction treated as a purchase. Philadelphia issued 200,000 shares of $10 par value stock valued at $18 for all of Saxon's common stock. That was $3,600,000 (200,000 × $18) in total. If 200,000 shares were issued for $8 over par, then $1,600,000 went to additional paid-in capital from that transaction. But there was already $1,300,000 there. The total of the two is $2,900,000 ($1,300,000 + $1,600,000). Saxon's additional paid-in capital is eliminated when the financial statements are consolidated.

MCQ-05607 *Barrel Corporation had service and interest costs of $50,000 related to its defined benefit pension plan for the year ended December 31, Year 7. The company's unrecognized prior service cost was $200,000 at December 31, Year 6 and the average remaining service life of the company's employees was 20 years. Plan assets earned an expected and actual return of 10% during Year 7. The company made contributions to the plan of $25,000 and paid benefits of $30,000 during the year. The pension plan had plan assets with a fair value of $300,000 at December 31, Year 6. The PBO was $400,000 at December 31, Year 6 and $420,000 at December 31, Year 7.* *What should Barrel Corporation report in accumulated other comprehensive income for this pension plan at December 31, Year 7 under U.S. GAAP?* A. $190,000 B. $30,000 C. $50,000 D. $200,000

D. $200,000 *Pension net gains or losses, prior service cost, and net transition assets or obligations* remain in accumulated other comprehensive income until recognized in net periodic pension cost through amortization ("AGE" of SIRAGE). Choice "1" is correct. The unrecognized prior service cost should be reported in accumulated other comprehensive income until recognized as a component of net periodic pension cost through amortization. *Recognition in Period Incurred (AGE): Prior service cost and pension losses decrease the funded status of the pension plan and are recorded with the following journal entry in the period incurred:* Dr Other comprehensive income 200,000 Cr Pension benefit asset/liability 200,000 At December 31, Year 7, the unrecognized prior service cost, before tax, is: 12/31/Year 6 Unrecognized prior service cost $200,000 − Amortization of prior service cost 10,000 = $200,000 / 20 years 12/31/Year 7 Unrecognized prior service cost $190,000 Journal entry to record the $10,000 ($200,000 / 20 years) reclassification adjustment from. OCI to the income statement for the Year 7 amortization of prior service cost. Dr Net periodic pension cost 10,000 Cr Other comprehensive income 10,000 Choice "2" is incorrect. This is the return on plan assets for Year 7 ($300,000 fair value of plan assets x 10% = $30,000), which is included in Year 7 net periodic pension cost. There is no difference between the expected and actual return on plan assets, so the return on plan assets does not affect accumulated other comprehensive income. Choice "3" is incorrect. The service cost and interest cost for Year 7 are included in Year 7 net periodic pension cost and are not recorded as a component of accumulated other comprehensive income. Choice "4" is incorrect. This is the amount that would be reported in accumulated other comprehensive income at December 31, Year 6.

MCQ-09290 On December 1, Year 1, Tom V. Company entered into an operating lease for office space for its executives for 10 years at a monthly rental of $200,000, increasing to $400,000 halfway through the lease. On that date, Tom V. paid the landlord the following amounts: First month's rent $200,000 Last month's rent 400,000 Installation of new carpet 600,000 Total $1,200,000 The entire amount was charged to rent expense in Year 1. What amount should Tom V. have charged to expense for the year? A. $1,200,000 B. $300,000 C. $200,000 D. $305,000

D. $305,000 Rent first 5 yrs: [$200,000x(5x12)]= 12,000,000 Rent remaining 5yrs: [$400,000x(5x12)]= 24,000,000 (12,000,000+24,000,000)/(10yrsx12)= 300,000 Installation of new carpet: 600,000/(10yrsx12)= 5,000 depriciation per yr 300,000+5,000 = 305,000 In this question, they want to know the amount charged to expense for what looks like an operating lease. An amount of $1,200,000 was paid on December 1, Year 1. $200,000 of the $1,200,000 was for actual rent. However, the lease agreement indicated that the rent would double halfway through the lease. The total rent that will be paid for the entire 10 years is thus $36,000,000, for an equivalent monthly rent of $300,000 ($36,000,000 / 120). The other amount that would be charged to expense (the question does not say rent expense, just expense) would be the amortization of the leasehold improvement. The leasehold is the office space, the improvement is the new carpet for the executives, and the amortization would be on the straight-line basis over 120 months, for $5,000 per month. The total expense would thus be $305,000.

MCQ-09369 Giant​ Jobs, Inc. amended its overfunded pension plan on December​ 31, Year​ 7, resulting in the recognition of prior service cost of​ $700,000. On December​ 31, Year​ 7, Giant​ Job's employees had an average remaining service life of 20 years. The company has an effective tax rate of​ 30%. How will the amortization of the prior service cost affect Gain job's December​ 31, Year 8 financial​ statements under U.S. GAAP? A. $24,500 decrease in other comprehensive income. B. $35,000 decrease in net income. C. $24,500 increase in pension benefit asset. D. $35,000 increase in net periodic pension cost.

D. $35,000 increase in net periodic pension cost. Prior service cost is amortized on a straight-line basis to net periodic pension cost over the average remaining service life of the company's employees: $700,000 / 20 years = $35,000/year. Pension net gains or losses, prior service cost, and net transition assets or obligations remain in accumulated other comprehensive income until recognized in net periodic pension cost through amortization ("AGE" of SIRAGE). Reclassification adjustment to record amortization of net loss, prior service cost, or net transition obligation to net periodic pension cost: Dr Net periodic pension cost 35,000 Cr Other comprehensive income 35,000 Dr Deferred tax benefit—OCI 10,500 Cr Deferred tax benefit—income statement 10,500 Choice "1" is incorrect. As demonstrated in the journal entry above, the amortization of the prior service cost will increase, not decrease, other comprehensive income by $24,500 ($35,000 credit to OCI—$10,500 debit to deferred tax benefit—OCI). Choice "2" is incorrect. As demonstrated in the journal entry above, the amortization of prior service cost decrease net income on an after-tax basis. Therefore, the total decease to net income will be $24,500 ($35,000 increase in net periodic pension cost − $10,500 deferred tax benefit). Choice "3" is incorrect. As demonstrated in the journal entry above, the amortization of prior service cost has no effect on the pension benefit asset. The prior service cost decreased Giant Job's pension benefit asset in the period of the plan amendment only.

MCQ-11096 Corbet Co. purchased a copyright near the beginning of the current year from an author for $20,000. The legal life of the copyright is equivalent to the life of the author plus 50 years. Corbet expects to sell the book for five years. What amount should Corbet report as amortization expense related to the copyright at the end of the current year? A. $400 B. $0 C. $500 D. $4,000

D. $4,000 An intangible asset such as a copyright or patent is amortized (straight-line, unless another method is indicated) over the shorter of its estimated life or remaining legal life. The estimated life is five years, and the legal life is the life of the author plus 50 years. As such, the estimated life will be used and the $20,000 cost will be amortized over five years at $4,000 per year. Choice "1" is incorrect. This choice incorrectly uses 50 years, which is not only greater than the estimated life of five years, but does not account for the legal life actually being the life of the author plus 50 years. Choice "2" is incorrect. Because the asset has a finite life, there will be amortization each period. Choice "3" is incorrect. This choice incorrectly assumes a useful life of 40 years.

MCQ-06375 Zen Transportion Inc.'s pension trustee provided the company with the following information for its defined benefit pension plan at December 31: Defined benefit obligation $2,500,000 Fair value of pension plan assets 1,950,000 Prior service cost from plan amendment 375,000 Actuarial gain 50,000 Current service cost 440,000 Under IFRS, what amount would Zen report in accumulated other comprehensive income related to its pension plan on its December 31 balance sheet? A. $325,000 B. $0 C. $440,000 D. $50,000

D. $50,000 IFRS: Under IFRS, intangible assets are reported using the *cost model (same as U.S. GAAP) or the revaluation model.* Revaluation Model Under the revaluation model, revalued intangible assets are reported at *fair value on the revaluation date less subsequent amortization and impairment.* *Revaluation losses are reported on the income statement* and *revaluation gains are generally reported in other comprehensive income.* Choice "4" is correct. Under IFRS, remeasurements of the defined benefit liability (asset), including remeasurements from actuarial gains, are reported in other comprehensive income and are not reclassified (amortized) to the income statement. Choice "1" is incorrect. Under IFRS, the actuarial gain is reported in other comprehensive income, but the prior service cost is reported on the income statement. Choice "2" is incorrect. The actuarial gain is reported in other comprehensive income under IFRS. Choice "3" is incorrect. Under IFRS, the actuarial gain is reported in other comprehensive income, while service cost (including both current and prior service cost) is reported on the income statement.

On July 1, Year 1, Planet Corporation sold Ken Company ten-year, 8% bonds with a face amount of $500,000 for $520,000. The market rate is 6%. The bonds pay interest semiannually on June 30 and December 31. for the six months ended December 31, Year 1, what should Planet report as bond interest expense and long-term liability in the balance sheet and income statement for Year 1? B/S I/S A. $511,200 $31,200 B. $500,000 $20,000 C. $504,400 $4,400 D. $515,600 $15,600

D. $515,600 $15,600 The bonds have a face amount of $500,000 and a stated rate of 8% and thus a semiannual interest payment of $20,000 ($500,000 × 0.08 × ½). The bonds were sold at a premium for $520,000 because the effective interest rate was 6% when the stated rate was 8%. They are asking about the first interest payment period. Cash Int (.04) -- Int exp (.05) -- Amort -- Unamort -- CV NA NA NA NA 520,000 20,000 15,600 4,400 The journal entry for the first interest payment is: Dr Bond interest expense 15,600 ($520,000 × 0.06 × ½) Premium amortization 4,400 Cash 20,000 ($500,000 × 0.08 × ½) The amount reported on the income statement is the bond interest expense of $15,600. If the question is worked as a journal entry, the amortization of the premium (or discount, if there had been a discount) is a plug. Journal entries are a good way to work this kind of problem even when the specific journal entry is not asked. The amount reported on the balance sheet is the adjusted carrying amount of the bond. The original carrying amount of the bond was $520,000. The amortization of the premium reduces the carrying amount of the bond to $515,600 ($520,000 − $4,400).

MCQ-09287 The Penton Corporation has the following foreign currency transactions during Year 1: Merchandise was purchased from a foreign supplier in January for the U.S. dollar equivalent of $180,000. The invoice was paid in March for the U.S. dollar equivalent of $192,000. On July 1, Penton borrowed the U.S. equivalent of $1,000,000 evidenced by a note that was payable in the lender's local currency in July of the next year. On December 31, Year 1, the U.S. dollar was equivalent of the principal amount and the accrued interest were $1,040,000 and $52,000 respectively. Interest on the note is 10% per annum. In Penton's Year 1 income statement, what amount should be inclduded as a foreign exchange loss? A. $0 B. $12,000 C. $42,000 D. $54,000

D. $54,000 Merchandise was purchased from a foreign supplier. When purchased, the merchandise cost $180,000, and, when paid, the merchandise cost $192,000. Both of the parts of the transaction were in the same year (watch out for ends of years getting in between, with part of the gain or loss then having to be recognized in the first year and the rest in the second year). That is $12,000 of loss, which is the second selection. It also eliminates the first selection. The next part is a loan. On a loan, both principal and interest must be considered. Looks like the principal cost $40,000 more ($1,040,000 - $1,000,000). That gets the foreign exchange loss to $52,000, which is not any of the answers. But what about the interest? The original interest would have been $50,000 ($1,000,000 x .10 x ½) and the actual interest was $52,000. Another $2,000 to add to the loss brings the loss to $54,000.

MCQ-09295 On January 1, Year 1, Sweeney Company granted an employee options to purchase 100 shares of Sweeney's common stock at $40 per share. The options became exercisable on December 31, Year 1, after the employee had completed one year of service, and were exercised on that date. Market prices of the stock and fair values of the options were as follows: Market Price -- Fair Value January 1, Year 1 $50 $61 December 31, Year 1 $65 $75 What amount should Sweeney recognize as compensation cost for Year 1? A. $0 B. $2,100 C. $4,000 D. $6,100

D. $6,100 Compensatory stock options should be valued at the fair value of the option on the grant date. On the grant date (January 1, Year 1), the option had a fair value of $61 per option, so the total compensation cost is $6,100 ($61 x 100 options). Journal Entry: To allocate compensation cost to Year 1 operations DR Compensation expense $6,100 CR APIC—stock options $6,100

Cavan Company prepared the following reconciliation between book income and taxable income for the current year ended December 31, Year 1: Pretax accounting income $1,000,000 Taxable income (600,000) Difference $400,000 Differences: Interest on municipal bonds $100,000 Lower financial depreciation 300,000 Total $400,000 Cavan's effective income tax rate for Year 1 is 21%. The depreciation difference will reverse equally over the next three years at enacted rates as follows: Year Tax rate Year 2 30% Year 3 25% Year 4 25% In Cavan's Year 1 Income statement, the current portion of its provision for income taxes should be: A. $84,000 B $80,000 C. $100,000 D. $63,000

D. $63,000 The taxable income of $600,000 is hiding in the first paragraph of the facts. The current tax rate is 21%. The tax currently payable is then $126,000; Taxable income × Current tax rate ($600,000 × 0.21).

MCQ-09378 *Post Company paid $100,000 for all the assets and liabilities of Script Corporation. Script Corporation's assets had a book value of $200,000 and a fair value of $210,000. Script's liabilities had a book value (equal to fair value) of $40,000. How much gain should Post recognize from this acquisition?* A. $0 B. $10,000 C. $50,000 D. $70,000

D. $70,000 When a subsidiary is acquired for less than the fair value of 100 percent of the underlying assets acquired, the acquisition cost is first allocated to the fair value of 100 percent of the balance sheet accounts and the fair value of 100 percent of the identifiable intangible assets acquired. This creates a negative balance in the acquisition cost account, which is recognized as a gain in the period of the acquisition. Gain/Goodwill = 100% Fair value of subsidiary - 100% Fair value of subsidiary's net assets *100,000 PP/FV Sub - [(210,000 - 40,000) x 100%] FV NA = 70,000 Gain*

MCQ-09413 On November 1, Year 1, Western Traders sold goods to an Italian company for 25,000 euros. Western was paid by the company in Euros on February 15, Year 2. The exchange rates to convert euros to dollars were: 11/1/Y1 $1.19/euro 12/31/Y1 $1.16/euro 2/15/Y2 $1.23/euro How much should Western report as a foreign exchange gain or loss on its December 31, Year 1 and Year 2 financial statements? Year 1 -- Year 2 A. $750 gain $1,000 loss B. $750 gain $1,750 loss C. $750 loss $1,000 gain D. $750 loss $1,750 gain

D. $750loss $1,750gain On November 1, Year 1, Western would record the transaction as follows: Dr Accounts receivable (25,000 euros × $1.19) 29,750 Cr Revenue 29,750 On December 31, Year 1, Western would record a foreign exchange transaction loss of $750 [25,000 euros × ($1.19 − 1.16)] due to the decrease in the exchange rate: Dr Foreign exchange transaction loss 750 Cr Accounts receivable 750 On February 15, Year 1, when Western receives the payment from its customer, the company would record a foreign exchange transaction gain of $1,750 [25,000 euros × ($1.23 − 1.16)]: Dr Cash (25,000 euros × $1.23) 30,750 Cr Accounts receivable ($29,750 − 750) 29,000 Cr Foreign exchange transaction gain 1,750 Choice "1" is incorrect. This answer incorrectly identifies the Year 1 loss as a gain. Additionally, it incorrectly identifies the Year 2 gain as a loss and incorrectly calculates the Year 2 amount by multiplying the $0.04 difference between the 11/1/Y1 exchange rate and the 2/15/Y2 exchange rate ($1.23 − $1.19 = $0.04) by 25,000 euros ($0.04/euro × 25,000 euros = $1,000). Choice "2" is incorrect. This answer incorrectly identifies the Year 1 loss as a gain. Additionally, it incorrectly identifies the Year 2 gain as a loss. Choice "3" is incorrect. This answer incorrectly calculates the Year 2 gain by multiplying the $0.04 difference between the 11/1/Y1 exchange rate and the 2/15/Y2 exchange rate ($1.23 − $1.19 = $0.04) by 25,000 euros ($0.04/euro × 25,000 euros = $1,000).

MCQ-04536 The Westwind Township reported a change in fund balance on its governmental fund financial statements of $75,000. On the face of its statement of revenues, expenditures, and changes in fund balance, the town reported capital outlay expenditures of $60,000, principal payments on debt of $50,000, interest payments on debt of $30,000, and other financing sources associated with proceeds on new debt of $100,000. The town's balance sheet displayed $25,000 in deferred inflows associated with measurable but unavailable revenue. The town's subsidiary records show incurred but unpaid interest of $15,000 and depreciation expense of $10,000. What would the town report as their change in net position for governmental activities? A. $65,000 B. $70,000 C. $100,000 D. $85,000

D. $85,000 Choice "4" is correct. Governmental activities change in net position is computed as follows: Change in fund balance 75,000 Capital outlay 60,000 Principal payments on debt 50,000 Asset disposal adjustment N/A Sources (other financing, debt proceeds) (100,000) Revenue accrual 25,000 Interest accrual (15,000) Depreciation Expense (10,000) Internal Service fund change in net position 0 = Change in net position 85,000 Choice "1" is incorrect. The proposed solution incorrectly subtracts capital outlay and principal payments and adds debt proceeds. Expenditures associated with non-current assets or liabilities are accounted for on the balance sheet and, therefore, added back while proceeds of non-current debt are also accounted for on the balance sheet and must be subtracted. Choice "2" is incorrect. The proposed answer excludes interest expense from the reconciliation in error and accounts for additional revenue as a subtraction. Additional revenue should be added, not subtracted. Interest that is incurred but not due is accrued on the government-wide financial statements but is not recognized in governmental fund financial statements. Interest accruals are, therefore, recognized as a reduction of the change in fund balance. Choice "3" is incorrect. The proposed answer excludes interest expense from the reconciliation in error. Interest that is incurred but not due is accrued on the government-wide financial statements but is not recognized in governmental fund financial statements. Interest accruals are, therefore, recognized as a reduction of the change in fund balance.

MCQ-09387 *The Darberville Day Care collected $45,000 in daycare fees from families during the year ended December 31, Year 1, to provide services. During the same period, the Day Care collected $30,000 from the United Way to provide ongoing funding for the Day Care's early childhood development programming and $10,000 from the Federal School Lunch Program to defray the cost of meals served to children. The Day Care's accounting policies consistently allow for the direct recording of contribution revenue with donor restrictions as contribution revenue without donor restrictions when the terms and conditions of the contributions are met in the year of donation. As a result of the above transactions, the Day Care would record revenues and support without donor restrictions of:* A. $40,000 B. $45,000 C. $75,000 D. $85,000

D. $85,000 Revenues and support without donor restrictions would consist of: Revenues Fees (exchange transactions) 45,000 Support United Way support 30,000 Federal grant earned as a result of satisfying restrictions in the same year the grant was awarded 10,000 Total revenue and support without donor restrictions 85,000 Choice "1" is incorrect. Revenues and support without donor restrictions include fees per above. Choice "2" is incorrect. Revenues and support without donor restrictions include both the United Way support and grant monies whose restrictions are satisfied in the same year as the award. Choice "3" is incorrect. Revenue and support without donor restrictions include grants with donor restrictions whose restrictions are satisfied in the same period in which the revenues are awarded.

MCQ-09332 In June, Year 1, Westchase Corporation became involved in product litigation. As a result of this litigation, it is probable that Westchase will have to pay $900,000. In August, a competitor commenced a suit against Westchase alleging violation of antitrust laws seeking damages of $100,000,000. Westchase denies the allegations, and the likelihood of Westchase paying any damages is remote. In September, Harris County brought action against Westchase for $9,000,000 for polluting. It is reasonably possible that Harris County will be successful, but the amount of damages Westchase will have to pay is not reasonably determinable. Westchase's tax rate is 40%.What amount, if any, should be accrued by a charge to income in Year 1? A. $109,000,000 B. $109,000,000 C. $9,900,000

D. $900,000 The only amount that should be recognized as a liability and charged against income is contingent losses that are probable and for which an amount can be reasonably estimated. In this question, the only contingency that falls into this category is the product litigation. The antitrust litigation is "remote," and is *ignored (unless guarantee of indebtedness of others, then disclose).* The pollution litigation is "reasonably possible." a *note disclosure* is required.

MCQ-01196 Elm City was founded in Year 1 and contributes to and administers a single-employer defined benefit pension plan on behalf of its covered employees. The plan is accounted for in a pension trust fund. Actuarially determined employer contribution requirements and contributions actually made for the past three years, along with the percentage of annual covered payroll, were as follows: Contribution made Actuarial requirement Amount -- Percent -- Amount -- Percent Year 3 $11,000 26 $11,000 26 Year 2 5,000 12 10,000 24 Year 1 5,000 12 10,000 24 None None 8,000 20 In the required supplementary information to Elm's Year 3 financial statements, the actuarially determined contributions should be shown to the extent available for a minimum of: A. 1 year. B. 10 years. C. 2 years. D. 3 years.

D. 3 years. The schedule of actuarially determined contributions displayed in the required supplementary information should be presented to the extent available (for a minimum of 10 years). Although the requirement is to display 10 years of data as required supplementary information, the city has only been in existence for 3 years.

MCQ-09448 The Palmetto Township established the Palmetto Township Building Authority to finance the construction of office buildings for Palmetto Township. The authority is a legally constituted entity whose governing body is the Town Council. The Authority is properly accounted for using a governmental accounting model and should be reported as: A. A primary government. B. A discrete presented component unit of the Palmetto Township. C. A blended component of the Palmetto Township by consolidation of the Authority's General Fund with the Township's General Fund. D. A blended component unit of the Palmetto Township by presentation of the Authority's General Fund as a Special Revenue Fund of the Township.

D. A blended component unit of the Palmetto Township by presentation of the Authority's General Fund as a Special Revenue Fund of the Township. The Authority is considered a component unit and would be blended since it exclusively services the Town and the governing bodies of the Town and the Authority are the same. The general fund of a blended component unit should be reported as a special revenue fund. Choice "1" is incorrect. A primary government is a jurisdiction that can stand by itSELF with a separately elected governing board, status as a legally separate entity and does not represent a financial burden or benefit to others. The Authority is legally separate, but the governing body for the Authority is identical to the Town. The Authority fails the SELF test, it is controlled by the Town and should be reported as a component unit. Choice "2" is incorrect. The Authority exclusively services the Town. It is a candidate for blending rather than discrete presentation. Choice "3" is incorrect. Blending of governmental models generally involves consolidation of like funds between the blended component unit and the primary government. The general fund of the component unit, however, is displayed as a special revenue fund.

MCQ-09313 On December 30, Year 1, Albert Altruistic donated $200,000 to the Carton Museum under the terms and conditions of a charitable remainder trust that guarantees Mr. Altruistic a life-time tax free annuity of $20,000 per year and bequeaths the remainder to Carton for use in their operations in furtherance of the mission of the museum. Independent actuaries have estimated that the museum's liability has a present value of $84,250. As a result of his contribution, the Carton Museum would record the following on its December 31, Year 1, financial statements: A. An increase in net assets without donor restrictions of $200,000. B. An increase in net assets with donor restrictions of $200,000. C. An increase in net assets without donor restrictions of $115,750. D. An increase in net assets with donor restrictions of $115,750.

D. An increase in net assets with donor restrictions of $115,750. Assets of a charitable remainder trust are measured at fair value when received and the liability is measured at the present value of expected future cash flows to be paid to the beneficiary. Contributions are not available until a future period (when the liability is fully satisfied) and is therefore an increase in net assets with donor restrictions. Carton would use the following entry. To record acquisition of assets subject to a charitable remainder trust: Dr Assets held in trust $200,000 Cr Liability to beneficiary $84,250 Cr Contribution revenue (with donor restrictions) $115,750 Choice "1" is incorrect. The net assets of this transaction relate to the contribution less the value of the liability, not the value of the asset itself. Choice "2" is incorrect. The net assets of this transaction relate to the contribution less the value of the liability, not the value of the asset itself. Choice "3" is incorrect. The net assets of this transaction relate to the contribution less the value of the liability, and is subject to donor restrictions associated with time requirements, not classified as an increase in net assets without donor restrictions.

MCQ-09021 A city government levies a tax on its citizens for improvements to roads. How should the city report the tax in its statement of activities? A. In special items reported separately from general revenue. B. As program-specific contributions in program revenues. C. By type of tax in program revenues. D. By type of tax in general revenues.

D. By type of tax in general revenues. Choice "4" is correct. Nonexchange revenues in support of governmental activities, such as taxes for improvements to roads, are classified as "general revenues" on the government-wide statement of activities. Choice "1" is incorrect. Special items are material unusual orinfrequent transactions (not both) that were executed by management (e.g., the sale of government property at a gain or loss, termination benefits resulting from workforce reductions, etc.) and would not include taxes. Taxes associated with a levy for road improvements would be classified as general revenues on the government-wide statement of activities. Choice "2" is incorrect. Program revenues displayed on the government-wide statement of activities include charges for services, operating grants, and capital grants (SOC), and would not include levies for improvements to roads. Taxes associated with a levy for road improvements would be classified as general revenues on the government-wide statement of activities. Choice "3" is incorrect. Program revenues displayed on the government-wide statement of activities include charges for services, operating grants, and capital grants (SOC), and would not include levies for improvements to roads. Taxes associated with a levy for road improvements would be classified as general revenues on the government-wide statement of activities.

MCQ-09748 Drexler Corp. has a June 30 fiscal year-end and plans to issue its annual (Year 3) financial statements by September 30, Year 3. On August 15, a warehouse fire destroys an estimated $250,000 of inventory. As a result of the fire, in its Year 3 financial statements Drexler should: A. Book a journal entry with a disclosure. B. Book a journal entry with no associated disclosure. C. Disclose the nature of the event with no estimated financial impact. D. Disclose the nature of the event along with the estimated financial impact.

D. Disclose the nature of the event along with the estimated financial impact. Although the fire occurred after the balance sheet date of June 30, the nature of the event and estimated financial impact were known before the financial statements were due to be issued on September 30. Therefore, the correct approach for Drexler is to disclose the nature of the event along with the estimated financial impact. Choice "1" is incorrect. The entirety of the event occurred after June 30, so there is no need to book a journal entry to reflect it in the Year 3 balance sheet/income statement. The entry will be booked in the Year 4 statements. Choice "2" is incorrect. The entirety of the event occurred after June 30, so there is no need to book a journal entry to reflect it in the Year 3 balance sheet/income statement. The entry will be booked in the Year 4 statements. In addition, a disclosure should also be included to help users of the financial statements understand the nature and impact of the event. Choice "3" is incorrect. Because an estimated financial impact is known, the nature of the event should be disclosed along with the estimated financial impact.

MCQ-09428 The Municipality of Sykes prepares its financial statements for the year ended December 31, Year 1. The municipality has elected to display all funds in its report including nonmajor funds. The Municipality of Sykes has the option of: A. Displaying each nonmajor fund in a combining statements of fund balance sheets and statement of revenues, expenditures, and changes in fund balance. B. Disclosing nonmajor fund activity in the notes to the financial statements. C. Describing each nonmajor fund in required supplementary information. D. Displaying a summary of nonmajor funds in the fund financial statements and a combining statement of nonmajor funds in other supplementary information.

D. Displaying a summary of nonmajor funds in the fund financial statements and a combining statement of nonmajor funds in other supplementary information.

MCQ-08517 Which of the following should be included in the introductory section of a local government's comprehensive annual financial report? A. Management letter. B. Auditor's report. C. Engagement letter. D. Letter of transmittal.

D. Letter of transmittal. 1. Introductory Section (unaudited) Letter of transmittal Organizational chart List of principal officers Choice "4" is correct. The introductory section of a local government's comprehensive annual financial report would include a letter of transmittal along with an organization chart and list of principal officers. Choice "1" is incorrect. The management letter would likely be included after all standard sections of the comprehensive annual financial report or perhaps presented separately from the report but would not be included in the introductory section. The introductory section of a local government's comprehensive annual report would include a letter of transmittal along with an organization chart and list of principal officers. Choice "2" is incorrect. The auditor's report would be included in the financial section of a government's comprehensive annual financial report, not the introductory section. The introductory section of a local government's comprehensive annual report would include a letter of transmittal along with an organization chart and list of principal officers. Choice "3" is incorrect. The engagement letter between the auditor and the local government would be included of the administrative files of the auditor and the local government and would not be included in the comprehensive annual financial report. The introductory section of a local government's comprehensive annual report would include a letter of transmittal along with an organization chart and list of principal officers.

MCQ-06586 *How should state appropriations to a state university choosing to report as engaged only in business-type activities be reported in its statement of revenues, expenses, and changes in net position?* A. Other financing sources. B. Capital contributions. C. Operating revenues. D. Nonoperating revenues.

D. Nonoperating revenues. Choice "4" is correct. Receiving state appropriations represents a non-exchange transaction and will be treated as nonoperating revenues. Operating grants and subsidies represent non-exchange transactions that are not derived from operations. They are treated as nonoperating revenues. Choice "1" is incorrect. Other financing sources is not a classification on the financial statements of proprietary funds used for business-type activities. Operating grants and subsidies (such as a state appropriation) represent non-exchange transactions that are not derived from operations. They are treated as nonoperating revenues. Choice "2" is incorrect. Operating grants and subsidies (such as a state appropriation) represent non-exchange transactions that are not derived from operations. They are treated as nonoperating revenues, not contributions. Choice "3" is incorrect. Operating grants and subsidies (such as a state appropriation) represent non-exchange transactions that are not derived from operations. They are treated as nonoperating revenues.

MCQ-09760 *An investor (company) will most likely avoid consolidating a potential variable interest entity (VIE) if:* A.The investor absorbs at least 50 percent of the VIE's expected gains. B. The VIE requires significant financial support from the investor. C. Current VIE equity holders do not have substantive voting rights. D. The investor does not absorb over half of theVIE's expected losses.

D. The investor does not absorb over half of theVIE's expected losses. Choice "1" is correct. This question asks for the inaccurate statement. One of the hallmarks of a VIE classification is that the entity itself is not self-supporting; it requires the financial support of an investor company in order to fund its activities. A thin equity capitalization is often found with VIEs, but that will tend to coincide with a lack of sufficient internal support. Choice "2" is incorrect. This is a true statement, as the majority of voting rights does not necessarily tie to which parties exercise actual power and control over the VIE. Choice "3" is incorrect. A significant amount of power, along with being a primary beneficiary of the VIE, will lead to the requirement for the investor company to consolidate the VIE in its financial statements. Choice "4" is incorrect. This is a true statement, as a VIE's primary beneficiary may change from one period to the next and the investor recognized as the primary beneficiary should be the company that is held accountable for consolidation.

MCQ-09430 Mighty Inc. purchase 90% of the common stock of of Puny Co. in a stock-for-stock transaction on January 1, Year 1. Which of the following statements regarding the preparation of Mighty Inc.'s consolidated financial statements is most accurate? A. The consolidated financial statements can be prepared using either the acquisition method or the pooling-of-interests mehtod. B. The consolidated income statement will include 90% of Puny Co.'s revenues and expenses. C. The consolidated balance sheet will include noncontrolling interest in Puny equal to the fair value of Puny multiplied by 90%. D. The noncontrolling interest in the net assets of Puny recorded on the consolidated balance sheet will be shown as a separate line item in the equity section of the balance sheet.

D. The noncontrolling interest in the net assets of Puny recorded on the consolidated balance sheet will be shown as a separate line item in the equity section of the balance sheet. Choice "1" is incorrect. The pooling-of-interests method is no longer allowed when accounting for business combinations. Choice "2" is incorrect. The consolidated income statement will include 100% of Puny Co.'s revenues and expenses, and will also show a line item deduction for the noncontrolling interest in the earnings of Puny equal to Puny's Year 1 earnings multiplied by the noncontrolling interest percentage of 10%. Choice "3" is incorrect. The noncontrolling interest in Puny reported on the consolidated balance sheet will be equal to Puny's fair value multiplied by the noncontrolling interest percentage of 10%(not 90%).

MCQ-09254 Which of the following statements is correct regarding sale-leaseback arrangements? A. The seller is the lessor who books a profit/loss at lease inception. B. The buyer is the lessee who gives the seller the right to use the asset. C. The buyer is the lessee who will defer profits if substantial rights are retained. D. The seller is the lessee whose profit is the difference between the fair value and book value.

D. The seller is the lessee whose profit is the difference between the fair value and book value. Choice "4" is correct. In a sale-leaseback, the seller is the lessee and the buyer is the lessor. The seller/lessee will book a profit when the sale price exceeds the book value of the asset sold. Choice "1" is incorrect. The seller may book a profit/loss at inception, but will be the lessee rather than the lessor. Choice "2" is incorrect. The buyer is the lessor (not the lessee) who gives the seller the right to use the asset. Choice "3" is incorrect. The buyer is the lessor, not the lessee.

MCQ-09336 Which of the following must be disclosed for most financial instruments? Carrying Value -- Fair Value A.No No B. No Yes C. Yes No D. Yes Yes

D. Yes Yes Disclosure of Financial Instruments *Fair value must be disclosed for all financial instruments* for which it is practicable to estimate that value together with the *related carrying amounts.* Disclosure of concentrations of credit risk is required. Credit risk is the possibility of loss from the failure of another party to perform according to the terms of a contract. Disclosure of market risk is encouraged but not required under U.S. GAAP. Under *IFRS, disclosure of market risk (risk of losing money in the stock market) is required.*

*Variable Interest Entities (VIEs)*

Definition A variable interest entity is a corporation, partnership, trust, LLC or other legal structure used for business purposes that either does not have equity investors with voting rights or lacks the sufficient financial resources to support its activities. *Company and Business Entity Have an Arrangement* 1. You have a variable interest (financial stake in them). 2. In a V.I.E (they lack basic equity items). 3. You will be the primary beneficiary (you will be biggest winner/loser). Primary Beneficiary The primary beneficiary is the entity that has the *power to direct the activities of a variable interest entity* that most significantly impact the entity's economic performance, and: - *absorbs the expected VIE losses*; or - *receives the expected VIE residual returns.* U.S. GAAP Consolidation Rule Under U.S. GAAP, the primary beneficiary of a variable interest entity must consolidate the variable interest entity. Under U.S. GAAP, all consolidation decisions are evaluated first under the VIE model. If consolidation is not required under the VIE model, then the investor (parent) company determines whether consolidation is necessary under the voting interest model (consolidate when ownership is more than 50 percent of the investee's voting stock, as previously covered). IFRS Consolidation Rule IFRS focus on the accounting for special purpose entities. A special purpose entity (SPE) is a specific type of VIE created by a sponsoring company to hold assets or liabilities, often for structured financing purposes (e.g., sales of receivables, synthetic leases, securitization of loans). *Under IFRS, a sponsoring company controls, and must consolidate, an SPE when the company:* - is benefited by the SPE's activities. - has decision-making powers that allow it to benefit from the SPE. - absorbs the risks and rewards of the SPE. - has a residual interest in the SPE.

Fair Value Measurement

Definition Fair value is the price that would be received to sell an asset or paid to transfer a liability in an orderly transaction between market participants in the principal (or most advantageous) market at the measurement date. - Fair value is a market-based measure. - Fair value is an *exit price,* not an entrance price. - Fair value does *not include transaction costs.* Market Determination - Market participants are buyers and sellers who are independent and willing to enter a transaction. - If there is a *principal market,* the price in that market will be used. - If there is no principal market, the price in the most advantageous market will be the fair value measurement. - *Transaction costs are used to determine the most advantageous market,* but are not included in the final fair value measurement. Measuring Fair Value Entities can use the market approach, the income approach, the cost approach, or a combination of these, as appropriate, when measuring the fair value of an asset or a liability. Valuation techniques should maximize the use of observable inputs (Level 1 and Level 2) and minimize the use of unobservable inputs (Level 3). - Level 1 inputs are quoted prices in active markets for *identical assets or liabilities.* - Level 2 inputs are other directly or indirectly *(similar)* observable inputs. - Level 3 inputs are unobservable inputs *(i.e., discounted cash flows, present value)*.

Encumbrance Activity

Encumbrances are *recorded when purchase orders are issued.* The issuance of a purchase order does not represent a liability, rather, the reserve for *encumbrance account acts as a limitation that reduces the available fund balance.* Entries to record the encumbrance of funds and the receipt of goods are as follows: To record the *issuance* of a purchase order: Dr Encumbrances $XXX Cr Budgetary control $XXX *Upon receipt of goods* the encumbrance of funds associated with the issued order is reversed and the related expenditure is recorded. Dr Budgetary control $XXX Cr Encumbrances $XXX *(The encumbrance always reflects the PO amount)* Dr Expenditures $XXX Cr Vouchers payable $XXX Relationship Between Accounts at Year-End Governmental funds record budget, actual, and encumbrance activities separately. *Both budget and actual activities are recorded during the year and then closed at the end of the year (BAE BAE). Encumbrances are recorded and closed throughout the year.* At year-end, if encumbrances are still outstanding, they are reported as a component of *committed or assigned fund balance* and disclosed if appropriations do not lapse. Inventory of Supplies When the government buys supplies, two methods can be used for the transaction: purchase and consumption. At the time of purchase: *The Purchase Method* Dr Expenditures $5,000 Cr Vouchers payable 5,000 At year-end (assumption: 1,000 of supply is still on hand): Dr Inventory of supply $1,000 Cr Nonspendable fund balance - inventory 1,000 *The Consumption Method* Dr Inventory of supply $5,000 Cr Vouchers payable 5,000 At year-end (assumption: 1,000 of supply is still on hand): Dr Expenditures $4,000 Cr Inventory of supply $4,000 Under the purchase method the remaining inventory must be placed in the balance sheet (it was all expensed at the beginning) and because it cannot be spent in that year, the fund balance should be reclassified as being nonspendable fund balance—inventory.

Pass-Through Contributions to Not-for-Profit Beneficiary (Non-For-Profit)

FASB ASC 958-605 defines the manner in which separate organizations that either receive or benefit from contributions account for the donations. Principles are largely driven by application of the concept of variance power. The pronouncement considers the *common situation of a foundation, the recipient, which raises money for another not-for-profit organization, the beneficiary (e.g., a university).* The statement also considers instances where the recipient is a federated or community-wide organization, such as the United Way, and the beneficiary is a smaller not-for-profit organization. Donor -> Recipient (NFP) -> Beneficiary (NFP) General Rule: Recipient (e.g., a Foundation) Without Variance Power If a recipient organization receives donations on behalf of another not- for-profit and does not have any discretion with regard to the use of the contribution, the donation is *recorded as a liability.* (Example: A donor gives a recipient 50k and tells the recipient to pass the 50k only to *one specific* NFP for cancer research.) Dr Asset $XXX Cr Refundable advance liability $XXX With Variance Power If a recipient organization receives donations on behalf of another not-for-profit and has discretion regarding the use of the contribution, the donation is *recorded as revenue.* (Example: A donor gives a recipient 50k and tells the recipient to pass the 50k to a NFP for cancer research of *their choice.*) Dr Asset $XXX Cr Revenue $XXX General Rule: Beneficiary (e.g., a University) Without Variance Power If a recipient organization receives donations on behalf of another not-for-profit and does not have any discretion with regard to the use of the contribution, the *donation is recorded as revenue on the beneficiary's books.* Receivable $XXX Contribution revenue $XXX With Variance Power If a recipient organization receives donations on behalf of another not-for-profit and *has discretion with regard to the use of the contribution,* the donation is *generally not recorded on the beneficiary's books* unless a financial relationship exists. Financially Interrelated Recipients and Beneficiaries Recipients and beneficiaries are deemed to be interrelated if 1. *one organization has the ability to influence the decisions of the other and* 2. *one organization has an ongoing interest in the other. Beneficiaries recognize an interest in the net assets of the recipient when they are financially interrelated with the recipient.* The interest is adjusted for the beneficiary's share of the change. Dr Interest in net assets $XXX Cr Change in interest in recipient net assets $XXX Beneficiaries recognize a beneficial interest *in an unconditional right to receive specified cash flows from pools of assets.* Changes in value are recorded on the beneficiary's books as follows: Dr Beneficial interest $XXX Cr Contribution revenue $XXX

Foreign Currency Translation & Remeasurement

Foreign Currency Translation Foreign currency translation is the conversion of a financial statement ofa foreign subsidiary into financial statements expressed in the reporting currency of the parent company. The method used to convert the financial statements depends on the functional currency of the subsidiary. *Step 1: Adjust to GAAP* *Step 2: Adjust foreign (local) currency to functional currency (Remeasuring) & G/L goes on Income statement* *Step 3: Functional to reporting currency (Translation) & G/L goes to OCI & appears on Balance sheet* Balance sheet: - Remeasure: LC to LF - Monetary-current - Inventory, PPE, Intangibles = Historical exchange rate APIC - Historical exchange rate, End RE (plug figure) Income statement: - Most use weighted average exchange rate COGS, depreciation, amortization = Historical exchange rate - G/L Remeasurement Method Foreign currency remeasurement is the restatement of foreign financial statements from the *foreign currency to the entity's functional currency* in the following situations: - *The reporting currency (parent) is the functional currency.* - The entity's books of record must be restated in the entity's functional currency prior to translating the financial statements from the functional currency to the reporting currency. Translate: T - C to reporting IS - Weighted avg. -> End RE All A + L = CSA or T APIC = Historical cost End RE AOCI = Plug figure Remeasurement starts with the balance sheet and converts *monetary items using current/year-end exchange rates* and *nonmonetary (inventory, PPE, intangibles) items using historical exchange rates.* The income statement is then converted using a weighted average exchange rate for all items except those related to the balance sheet (depreciation, amortization, and cost of goods sold). Balance sheet related items are converted using the appropriate historical rate. A gain or loss is plugged to net income to get the required balance needed to adjusted retained earnings so that the balance sheet balances. Remeasurement gains and losses are included in income. Translation Method - All current Foreign currency translation is the restatement of financial statements denominated in the *functional currency (primary economic environment) to the reporting currency.* Translation starts with the income statement and converts all elements using a weighted average exchange rate. Translated net income is transferred to retained earnings. Assets and liabilities on the balance sheet are then converted using the current/year-end exchange rate, common stock/APIC are converted using historical exchange rates, retained earnings is rolled forward, and then a gain or loss is plugged to OCI to make the balance sheet balance. Translation gains and losses are part of other comprehensive income *(PUFIE).*

Foreign Currency Transactions

Foreign currency transactions are transactions with a foreign entity (e.g., buying from and selling to) denominated in (to be settled in) a foreign currency. Foreign exchange transaction gains and losses must be computed at a given balance sheet date on all recorded transactions denominated in foreign currencies that have not be settled. *A/P denominated in a foreign currency:* *- If foreign currency increases in value, liability increase = loss* *- If foreign currency decreases in value, liability decrease = gain* A/R denominated in a foreign currency: - If foreign currency increases in value, asset increases = gain - If foreign currency decreases in value, asset decrease = loss On 12/1/Yr 1, Green company purchased goods on credit for 100,000 pesos. Green paid for the goods on 3/1/Yr 2. The exchange rates were: Date -- Rate 12/1/Yr 1 $0.10 12/31/Yr 1 $0.08 (Peso buys less EOY 1) 3/1/Yr 2 $0.09 (Peso buys more EOY 2)

*Fund Financial Statements*

Fund financial statements are presented for all funds based on their applicable measurement focus and related basis of accounting. *Only major funds are reported separately*; *nonmajor funds are reported in the aggregate.* Individual nonmajor funds may be reported as optional supplementary information. *Governmental Fund Financial Statements = GRaSPP* - *Balance Sheet:* Included in this statement are *assets plus deferred outflows* of resources, *liabilities plus deferred inflows* of resources, and *fund balance.* - *Statement of Revenues, Expenditures, and Changes in Fund Balance (IS):* Included in this statement are *other financing sources* (proceeds from debt and interfund transfers) and *other financing uses.* *Proprietary Fund Financial Statements = SE* - *Statement of Net Position (BS):* Included in this statement are *assets plus deferred outflows* of resources, *liabilities plus deferred inflows of resources*, and *net position.* - *Statement of Revenues, Expenses, and Changes in Fund Net Position (IS)* - *Statement of Cash Flows* *Fiduciary Fund Financial Statements = CIPPOE* - *Statement of Fiduciary Net Position (BS):* Included in this statement are *assets plus deferred outflows* of resources, *liabilities plus deferred inflows* of resources, and *net position.* - *Statement of Changes in Fiduciary Net Position (IS)* Determination of Major Funds GASB 34 as amended by GASB 63 emphasizes reporting by major fund rather than fund type. To qualify as a major fund the two following criteria must be met: 1. An individual fund's total assets plus deferred outflows of resources, or liabilities plus deferred inflows of resources, or revenues, or expenditures/expenses, are at least *10 percent or more* of the corresponding *total assets plus deferred outflows of resources, or liabilities plus deferred inflows* of resources, or revenues, or expenditures/expenses of all governmental funds or enterprise funds (e.g., a special revenue fund's revenues would need to be 10 percent of the revenues for the governmental fund financial statement category;a water and sewer fund's revenues would need to be 10 percent of all enterprise funds' revenues). All Major fund classification 10% test: fund amount has to be 10 percent higher than combined total governmental fund assets or total enterprise fund assets. 5% test: fund amount has to be 5 percent higher than combined total governmental and enterprise fund assets. -The general fund is ALWAYS a major fund-Internal service fund is not a major fund

Changes in Accounting Principle - Must Justify

General Rule (Retrospective Application) Any change from one generally accepted accounting principle to another generally accepted accounting principle is recognized using the *retrospective approach by adjusting beginning retained earnings for the cumulative effect of the change, net of tax.* Prior period financial statements are restated (IDA). Changes in Accounting Entity (Retrospective Application) Include changes in the companies that make up the *consolidated or combined financial statements from year to year.* Hence, if five-year comparative statements are presented, all these statements would be restated as though all the companies were always combined. The concept of a change in accounting entity is not discussed in IFRS. Error Corrections (Restatement Approach) *Error corrections require retroactive restatement by adjusting the beginning balance of retained earnings, net of tax, in the earliest year presented.* If the error occurred in a year presented, the error is corrected in those prior financial statements. *(i.e., Non-GAAP = Error)* Under IFRS, when it is impracticable to determine the cumulative effect of an error, the entity is required to restate information prospectively from the earliest date that is practicable. U.S. GAAP does not have an impracticality exemption for error corrections. Cumulative Effect The cumulative effect of a change in accounting principle is computed as of the beginning of the earliest year presented, regardless of the actual date of the change, by applying the new principle to the item to be changed since inception. The difference between the two principles is the catch-up amount for all prior affected periods. It includes direct effects and only those indirect effects that are entered into the accounting records. *IFRS - Present 3 BS, 2 other* Under IFRS, when an entity applies an accounting principle retroactively or makes a retrospective restatement of items in the financial statements, the entity must (at a minimum) present *three balance sheets* (end of current period, end of prior period, and beginning of prior period) and two of each of the other financial statements (current period and prior period). The cumulative effect adjustment would be shown as an adjustment of the beginning retained earnings on the balance sheet for the beginning of the prior period. *U.S. GAAP does not have a three balance sheet requirement.*

*Fund Balance Classifications*

Governmental fund (GRaSPP) balances are classified in any one of five ways mnemonic *NUCAR* (different levels of constraint): CA - CL = Fund balance N Nonspendable U Unrestricted C Committed A Assigned R Restricted *Nonspendable* Nonspendable fund balances represent resources that are nonspendable because they are not in spendable form *(e.g., inventories or prepaid expenditures)* or legally or contractually required to be maintained intact (e.g., permanent fund principal). *Restricted* Restricted fund balances represent resources whose *use has been limited by such external sources* as creditors (e.g., debt covenants), contributors, other governments, laws, constitutional provisions, or enabling legislation. Cannot be an internal group such as management. *Committed* Committed fund balances represent resources that can only be used for specific purposes pursuant to constraints imposed by formal action of the *government's highest level of decision-making authority (internal group).* *Assigned* Assigned fund balances are constrained by the government's intent to be used for specific purposes but are neither restricted nor committed. *Unassigned* Unassigned fund balances is the *residual classification for the general fund.* This classification represents fund balance that has not been assigned to other funds and that has not been restricted, committed or assigned to specific purposes within the general fund. *The general fund should be the only fund that shows a positive unassigned fund balance amount.* Over- expenditure of resources in other governmental funds may, however, result in a reported negative unassigned fund balance.

Fund Accounting Mechanic

Govt. fund - GRaSPP Fund accounting mechanics generally focus on the accounting for the governmental funds and require knowledge of the journal entries used to record the *budget, actual activities, and encumbrances* (BAE BAE). Budgetary Activity *To record the budget* into the accounting records, the following entry is used. Any balancing amounts are posted to budgetary control. Dr Estimated revenue control $XXX Cr Appropriations control $XXX Cr Budgetary control (Surplus) XXX Budgetary control can also be a debit. *Budgetary accounts are recorded at the beginning of the year and are closed at the end of the year.* Budgetary accounts are only impacted when establishing, amending or closing the budget. Actual Activity Actual activities are *recorded as they happen throughout the year.* Expenditures are typically recorded as they are incurred. *Capital purchases* are identified as capital outlay because of their long-term nature but they are *accounted for as expenditures* and they are not reported in the balance sheet. In addition, *no depreciation expense* is recorded on governmental fund financial statements. Generally, all spending is recorded currently as "expenditures." Dr Capital outlay expenditures $XXX Cr Cash or vouchers payable $XXX *Principal payment on debt* is displayed as an *expenditure* since there is no non-current debt recorded on the governmental fund financial statements. Dr Debt service—principal expenditure $XXX Cr Cash XXX In addition, new debt proceeds are recorded as *other financing sources,* a resource inflow: Dr Cash $XXX Cr Other financing sources—debt proceeds $XXX *Leases that include contracts that transfer ownership, or leases that are other than short-term leases (more than 12 months) and contracts that transfer ownership, are also recorded as other financing sources.* Dr Expenditure-capital outlay $XXX Cr Other financing sources—lease $XXX Lease payment on a contract that transfers ownership would be recorded in a manner consistent with other debt service payments. Dr Expenditure-principal $XXX Dr Expenditure-interest $XXX Cr Cash $XXX The modified accrual basis of accounting records (accrues) revenue when it is *measurable and available, generally collected within 60 days of year-end.* Only the amount available is recorded as revenue. Note the entry below to record *property tax receivable (imposed non-exchange revenue) and the related allowance for uncollectible taxes.* Tax receivable is recorded when the tax is levied but only available revenue is recognized. *No "bad debt expenditure" is recognized.* Dr Property tax receivable—current $XXX Cr Allowance for uncollectible taxes—current $XXX Cr Property tax revenue XXX

Industry Applications (Not-For-Profit)

Health Care Organization Revenue Recognition *Patient service revenue* should be accounted for on the *accrual basis* at *usual and customary fees,* even if the full amount is not expected to be collected. Although patient service revenue is accounted for on a *gross basis, deductions* are made from gross revenue for reporting purposes to display revenues net. Charity care, the value of services that a health care organization gives away, is not displayed in the financial statements. *Bad debt expense* (Operating expense) is recorded when the allowance is established after an evaluation of a patient's ability to pay. Otherwise, bad debts are recorded as a deduction from revenue. University and Institutions of Higher Learning Revenue Recognition Student tuition and fees should be reported at gross amount (Total income earned before deductions). Scholarships, tuition waivers, and similar reductions are considered either expenses or a separately displayed allowance reducing revenue.

MCQ-09708 A municipality that appropriately classifies a derivative as a hedge will account for an increase in the value of the derivative, which is reported as an asset, as: A. An increase in deferred inflows of resources. B. A decrease in deferred inflows of resources. C. An increase in deferred outflows of resources. D. A decrease in deferred outflows of resources.

Hedge Accounting Derivatives used as hedges to mitigate the risk of fluctuations in fair value or cash flows qualify for accounting as deferred outflows of resources and deferred inflows of resources if they are effective. Effectiveness means that changes in the value of a derivative used as a hedge offsets the fluctuations in the fair value or cash flows of the hedged item. A sample entry for the decline (unrealized loss) in value of a qualifying derivative instrument would be: Dr Deferred outflows of resources (loss) $XXX Cr Derivative instrument$XXX *The entry for the increase (unrealized gain) in value of a qualifying derivative instrument would be:* *Dr Derivative instrument $XXX* *Cr Deferred inflow of resources (gain) $XXX*

Infrastructure

Infrastructure can have an impact on both the statement of net assets and statement of activities. Included among the *capitalized non-current assets* on the government-wide statement of net assets should be eligible *infrastructure, such as roads, drainage systems, and bridges.* Consequently, *depreciation of such assets is required,* and should be reported in the statement of activities. However, if the government is *unable* to arrive at the cost data for its infrastructure, the use of a *modified approach* (no capitalization needed) is acceptable, provided that *supplementary information* describing the infrastructure, its condition, and estimation of expenses needed to maintain condition is included in the RSI. A complete new professional assessment of the infrastructure condition is necessary every three years.

Calculating Leases

Lease Payments In the calculation of lease payments, the lessee will include all of the following. (R) Required contractual fixed payments (which include any variable payments that are "in-substance" fixed payments) less any lease incentives paid or payable to the lessee. (E) Exercise option to buy the asset that lessee is reasonably certain to exercise. (P) Purchase price of the asset at lease end when lessor can require lessee purchase. (O) Only indexed or rate variable payments: No changes to future lease payments should be assumed based on increases or decreases in the index or rate. (R) Residual guarantees which are likely to be owed. (T) Termination penalty due from the lessee upon lease termination. Lessee lease payments may or may not include (at the lessee's option): (N) Nonlease components: Amounts allocated to non-lease components of a contract. Lessee lease payments will specifically *exclude* the following. (G) Guarantees of lessor debt by lessee. (O) Other variable lease payments (other than those noted above). *REPORTN are capitalized for Finance Capital Leases and Operating Capital Leases.* Lease Term Before the lease term begins the lease is recorded as a *footnote* and once the commencement date begins you record a *journal entry.* The lease term begins on the commencement date (when the asset is available for lessee use) and extends to the end of the noncancelable period (the period in which the lessee's right is enforceable). An option to extend the lease is included in the term if the lessee is reasonably certain to exercise the option, and an option to terminate is included in the term if the lessee is reasonably certain not to exercise the option. Both options are included if the lessor controls the exercise. Discount Rate The *lessor* will use the *rate implicit* in the lease. The *lessee* uses either the *rate implicit (1st option)* in the lease (if known) or if this rate is not readily determinable, the incremental borrowing rate (2nd choice) of the lessee. Initial Direct Costs *Initial direct costs incurred* as a result of executing the lease will be *included* in the valuation of the right-of-use (ROU) asset.

*Partnerships*

New Partner Contributions Contributions to a partnership are recorded at *fair value.* Dr Land $5,000 Cr Green, capital $5,000 Three basic methods are available for accounting for a new partner's contributions: "Exact" Method *No goodwill or bonus is recorded.* In the exact method, the exact amount that the new partner contributes is the exact amount credited to his/her capital account. Dr Cash $1,000 Cr Green, capital $1,000 *"B"onus Method - ''B" Balance in capital accounts* The old capital plus the new partner's investment equals the total new capital. However, the new partner's capital account is *credited for an amount different from his/her investment.* Any difference between the new partner's contribution and the amount credited is a *bonus to/from the new partner and is divided based on the old partner's profit/loss ratio.* Dr Cash $1,000 Dr X, capital (60%) 120 Dr Y, capital (40%) 80 Cr Green, capital $1,200 *"G"oodwill Method = "G" Going in is more than the total capital* Goodwill = Total new capital - (Old capital + New partner's investment). Any difference between the total new capital of the partnership and the total of the old capital plus the investment by the new partner is *goodwill for the old partners* and is allocated to their capital accounts in the old partnership profit/loss sharing ratio. Dr Cash $1,000 Dr Goodwill 400 Cr X, capital (60%) $240 Cr Y, capital (40%) 160 Cr Green, capital 1,000 Division of Profits/Losses For partnership operations, partnership income or loss is distributed among the various partners in accordance with their profit/loss sharing ratio. *If the partnership agreement does not give a profit/loss sharing ratio, then the division is equal.*

*U.S. GAAP VS IFRS NCI*

Noncontrolling Interest Noncontrolling interest is recognized in the consolidated financial statements when the parent company owns less than 100 percent of the subsidiary. Noncontrolling interest on the balance sheet is the noncontrolling shareholder's share of the fair value of the subsidiary. *Under U.S. GAAP, the noncontrolling interest included in equity on the balance sheet is calculated as:* *Noncontrolling interest (BS) = 100% Fair value of subsidiary × Noncontrolling interest percentage* Noncontrolling interest must be recognized as a line-item deduction on the income statement for the portion of the subsidiary's net income not allocated to the parent company: *Noncontrolling interest in net income of subsidiary = Subsidiary net income × Noncontrolling interest percentage* Comprehensive income attributable to the noncontrolling interest is presented on the consolidated statement of comprehensive income.A reconciliation at the beginning and end of the period of the carrying amount of the equity attributable to the noncontrolling interest is shown on the consolidated statement of changes in equity. *Under IFRS, noncontrolling interest (and goodwill) can be calculated using either the full goodwill method, which is the method required under U.S. GAAP, or the partial goodwill method. Under the partial goodwill method, noncontrolling interest on the balance sheet is calculated as:* *Noncontrolling interest (BS) = % Fair value of subsidiary's net assets × Noncontrolling interest percentage*

Not-for-Profit Entities

Not-for-profit entities possess the following characteristics: *- Contributions* of significant amounts of resources received from providers who do not expect commensurate or proportionate return. - Operating purposes other than to provide goods or services at a profit. They do not provide exchange transactions. - Absence of ownership interest, no shareholders. The *FASB has the primary responsibility* of providing guidance on generally accepted accounting principles for not-for-profit entities. Very similar to commercial accounting methods except for the three above. Required Financial Statements All not-for-profit entities are required to prepare three basic financial statements on the full accrual basis: √ Statement of Financial Position (Balance sheet) √ Statement of Activities (Income Statement) √ Statement of Cash Flows Disclosure of Functional Expenses (Difference bt NFP & Commercial Accounting) Not-for-profit organizations are required to present disclosure of functional expenses, analyzed by object classification. The objective of the functional expense disclosure is to present the *programmatic and support expenses* displayed horizontally on the statement of activities in separate columns and to analyze the *expenses by object (natural classifications).* Disclosure of expenses by natural classification is done on the statement of activities, or footnotes.

Lessee Accounting

Operating Capital Lease *1 expense = lease expense, no depreciation* For an operating lease, the lessee's balance sheet will reflect a right-of-use (ROU) asset and lease liability and both will be amortized over the life of the lease using the effective interest method. On the income statement, *lease expense* will be recognized each year over the lease term using the straight-line method for expense measurement. Instead of reporting interest expense on the income statement, the lessee will report the interest as part of lease expense. *An operating lease is treated like renting—lease payments are considered as operating expenses. Assets being leased are not recorded on the company's balance sheet; they are expensed on the income statement. So, they affect both operating and net income.* Finance Capital Lease = OWNES *2 expenses = Interest expense, and depreciation (amortization) expense* If the lease is a finance lease, the lessee will recognize both an ROU asset and a corresponding liability on its balance sheet. Both will be amortized over the life of the lease. On the income statement, interest expense and amortization expense will be recognized over the life of the lease. Accounting Policy Election *Short term -> don't have to capitalize* If the lease term is 12 months or less, lessees can make an accounting policy election and choose to not recognize ROU assets and lease liabilities. To do this, the lease cannot include purchase options for the asset that the lessee is reasonably certain to exercise. Lease expense will be recognized on the income statement and the cash payments will have no effect on the balance sheet.

Revenue Sources (SE Funds)

Operating Revenues: Operating revenues, or billings for services provided, are *recognized when earned.* *(I.e., charges for services)* JE to record billings for services rendered to other funds: Dr Cash (or due from other funds) $XXX Cr Billings to other departments (operating revenues) $XXX Non-Operating Revenues: Nonoperating revenues are earnings or *non-exchange transactions* (such as taxes and certain fees and charges) and interest. - *Shared revenues* are revenues (e.g., from gasoline or sales taxes) collected by one government (e.g., state) and shared on a predetermined basis with another (e.g., local) government. They are nonoperating revenues of the enterprise fund.

*Investment in Subsidiary*

Original Carrying Amount The original carrying amount of the investment in subsidiary account on the parent's books is: - Original cost: measured by the fair value (on the date the acquisition is complete) of the considered given (debit: investment in sub). Business combination costs/expenses in an acquisition are treated as follows: - *Direct out-of-pocket costs and indirect costs are expensed (debit: expense).* - *Stock registration and issuance costs such as SEC filing fees are a direct reduction of the value of the stock issued (debit: additional paid-in-capital)*

*Summary of Treasury Stock Journal Entries*

Original Issue 10,000 shares of $10 par value, CS are sold for $15 per share Dr Cash 150,000 Cr Common stock 100,000 Cr APIC—CS 50,000 *Cost method - G/L recorded upon reissue* Buy Back Below 200 shares repurchased for $12 per share Dr Treasury stock (200 X $12 PP) 2,400 Cr Cash 2,400 *No G/L yet* Reissue Above Cost 100 shares repurchased for $12 are resold for $15 $15 SP - $12 Cost = $3 Gain increases Equity Dr Cash (100 x $15 SP) 1,500 Cr Treasury stock (100 x $12 Cost) 1,200 *Dr APIC—TS 300* *The 300 APIC-TS (Gain) is used to absorb future losses.* Reissue Below Cost 100 shares repurchased for $12 are resold for $3 Dr Cash (100 x $3 PP) 300 Dr APIC—TS 300 Dr Retained earnings (plug) 600 Cr Treasury stock (100 x $12 par) 1,200 *The 300 APIC-TS (Gain) is used to absorb part of the 900 losses and the remaining 600 reduces RE.* Original Issue 10,000 shares of $10 par value, CS are sold for $15 per share Dr Cash 150,000 Cr Common stock (10,000 x $10 par) 100,000 Cr APIC—CS (10,000 x $5) 50,000 *Par Value Method - G/L calculated upon repurchase* $15 SP - $12 Cost = $3 Gain increases Equity Dr Treasury stock (200 x $10 par) 2,000 Dr APIC—CS (200 x $5) 1,000 Reverse original entry using $5 gain Cr Cash (200 x $12PP) 2,400 Cr APIC—TS (200 x $3) 600 *The 600 APIC-TS (Gain) increases Equity & is used to absorb future losses.* Reissue Above Cost 100 shares repurchased for $12 are resold for $15 $15 SP - $12 Cost = $3 Gain increases Equity Dr Cash (100 x $15) 1,500 Cr Treasury stock (100 x $10 par) 1,000 Cr APIC—CS (100 x $5) 500 Reissue Below Cost 100 shares repurchased for $12 are resold for $3 Dr Cash (100 x $3SP) 300 Dr APIC—TS 600 Dr Retained earnings (plus) 100 Cr Treasury stock 1,000 *Sold below par thus APIC is debited* *The 600 APIC-TS (Gain) is used to absorb losses and the remainder 300 reduces RE.* *Note: Retained earnings may be debited, but never credited in treasury stock transactions.*

MCQ-09446 The General Fund of Revenue Township benefits from both derived tax revenues and imposed non-exchange revenues. Application of the modified accrual basis of accounting to receivables associated with these revenues includes: Derived tax revenues -- Imposed non-exchange revenues A. Accrual upon receipt -- Accrual when billed B. Accrual when billed -- Accrual upon receipt C. Accrual when billed -- Accrual when billed D. Accrual upon receipt -- Accrual upon receipt

Receivables associated with derived tax revenues are generally recorded upon receipt while receivables associated with imposed non-exchange revenues are recorded when billed. *General Rule:* Derived tax revenues, such as income *taxes* and *sales taxes* are accrued if measurable and available. Typically derived tax revenues are accrued at year-end if received within 60 days of year-end. Imposed non-exchange revenues, such as *property taxes* and *fines*, are recorded as a receivable when billed. Although revenue recognition is still subject to the availability criteria (received within 60 days of year end), the enforceable rights of the government allows for recording the receivable. Choice "2" is incorrect. Receivables associated with derived tax revenues are generally recorded upon receipt while receivables associated with imposed non-exchange revenues are recorded when billed. Choice "3" is incorrect. Although receivables associated with imposed non-exchange revenues are recorded when billed, receivables associated with derived tax revenues are generally recorded upon receipt. Choice "4" is incorrect. Although receivables associated with derived tax revenues are generally recorded upon receipt, receivables associated with imposed non-exchange revenues are recorded when billed.

*Retained Earnings*

Retained earnings (or deficits) are cumulative earnings (or losses) during the life of the corporation that have not been paid as dividends. A portion of retained earnings may be appropriated (restricted) for legal reasons or as a discretionary action of management. The appropriated retained earnings is distinguished from the unappropriated retained earnings account in the balance sheet. Must disclose to stockholders that some of the RE is not available for dividends. Dr RE unappropriated CR RE Appropriated Total RE does not change it only changes form, when the legal or contractual obligation is completed you reverse the original JE above. *Beginning retained earnings* *+ Net income/loss* *− Dividends (cash, FMV property, and stock) declared* *± Prior period adjustments (correct error net of tax)* *± Accounting changes (cumulative effect net of tax)* *− Treasury stock (when necessary)* *+ Adjustment from quasi-reorganization* *= Ending retained earnings* Treasury is a contra equity account and is subtracted. An adjustment from "quasi-reorganization" is done when you remove a deficit from RE by decreasing APIC and increasing RE (Dr APIC Cr RE).

Revenue Recognition

Revenue and Support Recognition Not-for-profit accounting focuses on two terms *(conditional and restricted)* that are often used interchangeably in conversation but have two distinct accounting meanings. - Classification of net assets relates to presence (with) or absence (without) of donor-imposed restrictions on contributions. - Recognition of revenue relates to the treatment of gifts or promises to give. *Conditional promises are not recorded* until the condition is satisfied and *conditional gifts received in advance of satisfying conditions are recorded as liabilities (refundable advance/refundable liability).* - Conditional is not the same as restricted. Resource inflows in not-for-profit organizations are generally displayed in the financial statements as either revenue or other support. Revenues typically represent exchange transactions in which the not-for-profit organization earns resources in exchange for a service performed(e.g., fees). Support often represents unconditional contributions. Cash Contributions and Unconditional Promises Cash contributions and unconditional promises are displayed as *support upon receipt or accrual.* *Conditional Promises and Receipts - No revenue recognition until the condition is satisfied* Cash contributions that can only be used upon meeting a condition and conditional promises to give are not recorded as revenue until the condition is met. A condition is a contingency that must be resolved, rather than a restriction that can be satisfied. *Conditional contributions ("good faith deposits") are displayed as a liability titled "refundable advance."* Conditional promises to give are not recorded. Multiyear Pledges Unconditional promises receivable *over a period of years* are recognized as an increase to *net assets with donor restrictions* since amounts *have an implied time restriction.* Receivables are recorded at their present value with the difference between face and present value recognized as contribution revenue over time, not interest. Other Revenue Transactions and Issues - Agency transactions relate to receipts of resources over which the not-for-profit organization has *no discretion or "variance power."* The absence of variance power over the resources creates a *liability* rather than revenue. - Gifts-in-kind represent *noncash contributions* recorded as *both support and an offsetting expense.* - *Exchange transactions* represent the sale of goods or services in exchange for a fee. Exchange transactions are *classified as revenues without donor restrictions.* - *Donations with donor restrictions* for which restrictions *will be satisfied within the year of receipt* may be classified *as donations without donor restrictions* in the event that the not-for-profit organization consistently applies this policy.

The Five-Step Approach (ISTAR)

Revenue is recognized when the good or service is transferred to the customer and the performance obligation is satisfied. The amount of revenue to be recognized should reflect the expected consideration that the entity is entitled to receive. The five-step approach should be implemented by an entity in order to properly recognize revenue. The steps are as follows: - Step 1: *Identify the contract* with the customer—a contract is an agreement between parties that creates enforceable rights or obligations. - Step 2: Identify the *separate performance obligations* in the contract—a performance obligation is a promise to transfer either a good or a service to a customer. - Step 3: Determine the *transaction price*—the transaction price is the amount of consideration that an entity is entitled to receive in exchange for transferring goods and/or services to a customer. - Step 4: *Allocate* the transaction price to the separate performance obligations—if a contract contains more than one performance obligation, the overall transaction price will need to be allocated to each separate obligation. - Step 5: *Recognize* revenue when or as the entity satisfies each performance obligation—revenue is recognized when the performance obligation is satisfied either at a point in time or over time through transferring the good/service to the customer.

Income (loss) from discontinued operations

Sales and expenses from business segments that are either disposed of or not expected to continue -Classified as held for sale "plan" -"below the line" (not reported with income but after continuing operations) The (normal) loss from discontinued operations can consist of three elements: (1) an impairment loss (2) income/loss from actual operations; and (3) gain loss on disposal. All these losses are included in discontinued operations in the period in which they occur. *Net FV of component - (Disposal cost + Losses pertaining to period) = Discontinued operations*

Single-step Income Statement

Single-step income statement: Step 1: Total revenues and gains - Total expenses and losses other than tax = Earning before tax Step 2: Earning before tax x Tax rate = Tax expense Step 3: Earning before tax - Tax expense = NI

MCQ-09422 On December 31, Stewart and Colbert had capital account balances of $200,000 and $150,000, respectively. Their partnership agreement includes the following provisions: - Profits and losses are to be shared equally. - Stewart is to receive a salary allowance of $65,000. - Colbert is to receive a bonus of 50 percent of profits in excess of $100,000. - Each partner is to receive 10 percent interest on their ending capital accounts. Partnership profit before any allocations to partners was $150,000. What was the total of the distributions to each partner? Stewart -- Colbert A. $85,000 $40,000 B. $85,000 $52,500 C. $97,500 $40,000 D. $97,500 $52,500

Stewart Capital balance: $200,000 Salary allowance: $65,000 10% interst: $200,000 x .10 = $20,000 Equal sharing of remainder profit & loss: $25,000* x .50 =$12,500 Colbert Capital balance: $150,000 Bonus: ($150,000 - $100,000) x .50 = $25,000 10% interst: $150,000 x .10 = $15,000 Equal sharing of remainder profit & loss: $25,000* x .50 =$12,500 Partnership profit before allocations: $150,000 Allocations to partnership: $85,000 + $40,000 = $125,000 $150,000 - $125,000 = $25,000 remaining profit* The original partners had a total capital balance of $130,000 and the new partner was contributing $50,000. Since the partnership capital account will begin with $180,000 including the new partner, there cannot be any goodwill. But there still might be a bonus. Chance gets a one-third interest in the partnership. That means Chance's capital balance will be $60,000 ($180,000 × 1/3) for an investment of $50,000. Unfortunately, three of the four answers have $60,000 in Chance's column. The $10,000 difference comes from the other partners, and it comes from the other partners in their profit and loss sharing ratio, which in this case happens to be 3:2. The $10,000 is perfectly divisible by 5, so it must be the third selection. Of the $10,000, three fifths ($10,000 × 3/5) or $6,000 comes from Coco, and two fifths ($10,000 × 2/5) or $4,000 comes from Chanel. That leaves Coco with $64,000 ($70,000 - $6,000) and Chanel with $56,000 ($60,000 - $4,000), respectively.

MCQ-09314 The county of Deutsch appropriated $45,000 in its General Fund for miscellaneous supplies for its fiscal year ended September 30, Year 1. The county found that it had paid $15,000 for miscellaneous supplies in November Year 0 and issued a $30,000 PO to a sole source vendor for miscellaneous supplies in December Year 0. By August Year 1, the county had received $20,000 related to the order but did not pay the vendor until October pending tax receipts. Appropriations do not lapse. What was the County of Deutsch's budgetary control related to encumbrances at September 30, Year 1? A. $45,000 B. $30,000 C. $15,000 D. $10,000

The County's year-end budgetary control related to encumbrances would be computed as follows: Budgetary control (Purchase order) 30,000 Order received (20,000) = Budgetary control 10,000 To record encumbrances for issued purchase order: Dr Encumbrances 30,000 Cr Budgetary control 30,000 To reverse encumbrances associated with liabilities incurred related to items received: Dr Budgetary control 20,000 Cr Encumbrances 20,000 Dr Expenditures 20,000 Cr Accounts Payable 20,000 Choice "1" is incorrect. The year-end budgetary control related to encumbrances would not be equal to the amount of the total appropriation. Choice "2" is incorrect. The year-end budgetary control related to encumbrances would be reduced by amounts received and accrued as expenditures. Choice "3" is incorrect. The budgetary control related to encumbrances is reduced by the amount of the liability, not the amount previously paid.

Amortization of Premiums and Discounts

The carrying amount of a bond is the bond's face value plus the unamortized premium or minus the unamortized discount (and minus any bond issuance costs, as discussed above). At the maturity of a bond, the carrying amount of the bond is equal to the face. Effective Interest Amortization Method: U.S. GAAP/IFRS. Cash Int (.04) -- Int exp (.05) -- Amort -- Unamort -- CV NA NA NA 50,000 950,000 40,000 47,500 7,500 42,500 957,500 40,000 47,875 7,875 34,625 965,375 *All amortization goes toward the face value.* Balance Sheet: Bond Face × Coupon Rate = Interest Paid Income Statement: Net Carrying Value × Effective Interest Rate = Interest Expense Interest Paid - Amortization = Amortization Dr Interest expense $47,500 Cr Discount on B/P $7,500 Cr Cash 40,000 or Dr Interest expense XXX Dr Premium on B/P XXX Cr Cash XXX Retirement of Bonds Corporations can call or retire bonds prior to maturity. Bonds are retired as a percentage of face value (e.g., 98, 101). Will be gain or loss on the income statement. Issuance of BP with premium JE: Dr Cash $XXX Cr Bonds payable $XXX Cr Premium; on B/P less bond issuance costs $XXX Retirement of BP with premium JE: Dr Bonds payable $XXX Dr Premium on B/P XXX Cr Cash XXX (CR—gain/DR—loss for the difference) or Issuance of BP with discount JE: Dr Cash $XXX Cr Discount on B/P less bond issuance costs $XXX Cr Bonds payable $XXX Retirement of BP with discount JE: Dr Bonds payable XXX Cr Discount on B/P XXX Cr Cash XXX (CR—gain/DR—loss for the difference)

*On January 1, Year 1, Sunshine Corporation acquired an 80% ownership interest in Grey Sky Enterprise by purchasing 400,000 of Grey Sky's 500,000 voting common shares outstanding for $10,000,000. Additional information regarding Grey Sky as of January 1, Year 1, follows:* *Book Value Fair Value* *Net Assets $11,200,000 $11,500,000* *On the acquisition date, what is the goodwill that will be reported on the consolidated balance sheet of Sunshine Corporation under IFRS partial goodwill method?* A. $800,000 B. $1,000,000 C. $1,040,000 D. $1,500,000

The difference between the fair value of the subsidiary and the book value of the subsidiary net assets should be allocated as follows (BIG): B 1. Balance sheet adjustment of the subsidiary's assets and liabilities from book value to fair value. I 2. Identifiable intangible assets recorded at fair value. G 3. Goodwill is excess. Under U.S. GAAP, goodwill is calculated as follows (full goodwill method): Goodwill = 100% Fair value of subsidiary - 100% Fair value of subsidiary's net assets The fair value of the subsidiary is calculated as follows: FV subsidiary x 80% = $10,000,000 10,000,000 / 0.80 = $12,500,000 100% FV subsidiary Therefore, goodwill is: Goodwill = $12,500,000 - $11,500,000 = $1,000,000 *IFRS permits the use of the full goodwill method or the partial goodwill method. Under the partial goodwill method, goodwill is calculated as follows:* *Goodwill = % Acquisition cost - % Fair value of subsidiary's net assets acquired* *Goodwill = $10,000,000 (@ 80%) - ($11,500,000 x 80%) = $10,000,000 - $9,200,000 = $800,000* *Goodwill recognized in a business combination is not amortized.* Instead, it is tested for impairment, and a loss is recognized in income from continuing operations if the goodwill is impaired.

MCQ-09335 Mission Corporation leases equipment to Mars Company for $60,000 per year under a four-year direct financing (finance) lease. The first payment is made at lease inception. The equipment has no residual value at the end of the lease and the lease does not contain a written purchase option. Mission will earn 11% interest on the lease. The present value of an annuity due of $1 at 11% for four years is 3.4437. What is the total amount of interest revenue that Mission will earn over the life of the lease? A. $33,378 B. $60,000 C. $206,622 D. $240,000

The interest revenue on a direct financing (finance) lease is equal to the difference between the total lease payments from the lease and the present value of the lease payments: $60,000 x 4yrs = 240,000 Total payment $60,000 x 3.4437 = 206,622 PV of total principal due 240,000 - 206,622 = 33,378 Interest due Choice "2" is incorrect. This is the annual lease payment, not the interest to be earned over the life of the lease. Choice "3" is incorrect. This is the present value of the annual lease payments, not the interest to be earned over the life of the lease. Choice "4" is incorrect. This is undiscounted total lease payments, not the interest to be earned over the life of the lease.

*Net Asset Classification*

The reporting objectives of not-for-profit organizations include presentation of the *net assets (A-L=NA) at the balance sheet date* and the components of the *change in net assets on the statement of activities (IS)* for the year ending on the balance sheet date. There are two net asset classifications: *Without Donor Restrictions* Net assets free of donor restrictions on usage. *With Donor Restrictions* Only donors may restrict assets. *The management or board can designate or identify assets to be used for a particular purpose.* However, a designation is not a restriction. Net assets contributed with donor-imposed restrictions may be either temporary in nature (due to purpose, time, or acquisition of plant) or in perpetuity. 1. *Purpose (Expires)* The money must be spent *as the donor stipulates* (e.g., cancer research, youth education). Once the net assets are used for the required purpose the restriction is removed. 2. *Time (Expires)* The donated assets may be donor-restricted until a fixed period passes (e.g., a gift of a CD that must be held until maturity and then can be spent as the organization wishes). Time restrictions may also be implied by availability; contributions receivable generally increase net assets with donor restrictions. Once the time restriction is met the restriction is removed. The donated assets are classified as net assets with donor restrictions to purchase or build long-lived assets. Donor Restrictions in Perpetuity (Endowments) Net assets contributed with restrictions, such as an endowment fund where the corpus must be retained in perpetuity and interest income can be used by the not-for-profit organization in accordance with the donor's stipulations. *Acquisition of Plant (Expires)* The donated assets are classified as net assets with donor restrictions *to purchase or build long-lived assets.* Once the plant is purchased the restriction is removed. *Donor Restrictions in Perpetuity (Endowments)* Net assets contributed with restrictions, such as an endowment fund where the corpus must be retained in perpetuity (restricted) and interest income can be used by the not-for-profit organization in accordance with the donor's stipulations.

*Non-For-Profit Statement of Cash Flows*

The statement of cash flows is the same as the statement issued by commercial enterprises with a few unusual features. *Either the direct or indirect method* can be used. There are three categories of cash flow activities: *Operating Activities* - Include applicable agency transactions. - Include receipts of resources *without donor restrictions* designated by the governing body to be used for long-lived assets. *Investing Activities* - Include proceeds from the sale of works of art or purchases of works of art. - Include investment in equipment. - Include *proceeds from the sale of assets that were received in prior periods and whose sale proceeds were donor-restricted to investment in equipment or other LTA.* Disbursements (payment)* of these donor-restricted contributions for either investments or the purpose for which they were intended are classified as investing activities. *Financing Activities* - Proceeds From Donor-Restricted Contributions - Include *cash received with donor-imposed restrictions* limiting its use to purchases of long-term assets or annuity agreements. Other Types of Financing Activities Include receipts and disbursements associated with *borrowing and receipts* of dividends and interest that are donor-restricted to reinvestment. Cash and Cash Equivalents *Donor-restricted securities* that may otherwise meet the cash equivalent criteria in commercial accounting *are excluded.* (I.e., short-term CD's, short-term investments)

Specific Applications Within Revenue Recognition

There are unique aspects to revenue recognition that are applicable in many distinct scenarios, as described below: - *Incremental costs to obtain a contract = Deferred charge (Treat as asset and amortize)* Costs to obtain a contract are treated as assets if the entity expects to recover them. Costs are treated as expenses if they are borne regardless of whether the contract is obtained. *(i.e., sales commission for new customer, outside legal counsel not your company lawyer)* - *Costs to fulfill a contract = RM + DL + FOH* Costs to fulfill a contract are treated as an asset if they relate directly to the contract, they generate/enhance the entity's resources, and they are expected to be recovered. - Principal vs. agent *Principal = Report gross revenue* *Agent = Report net revenue* A principal has control over the good/service prior to transfer and will recognize revenue equal to expected gross consideration. An agent does not have control and will recognize revenue equal to a fee/commission. - *Repurchase agreements = Borrowing (gave collateral), not a sale/revenue* Repurchase agreements are contracts where an entity sells an asset and promises or has the option to repurchase the asset later. An obligation to repurchase is a forward, a right to repurchase at an entity's option is a call option, and a right to repurchase at the customer's option is a put option. - *Bill-and-hold arrangements = General rule = Not a sale until delivery* Bill-and-hold arrangements allow revenue to be recognized prior to the customer receiving the product as long as there is a substantive reason (Exception: buyer request then recognize as sale) for holding the product, the entity cannot use or redirect the product, and the product is separately identified and ready for transfer to the purchasing customer. - *Consignment = Not a sale/revenue* Consignment arrangements exist when an entity provides a product to a dealer to be held until it is ultimately sold to a third-party customer. Revenue is recognized either upon ultimate sale to a customer or after the expiration of a defined period of time. - Warranties (I*S*TAR) Warranties are treated as *separate performance obligations* distinct from the product covered in the contract if the warranty is not required by law, if the coverage period is lengthy, and if there are no specific tasks required regarding compliance assurance. - Right to return When a customer has a right to return, the selling entity should book revenue for the amount of consideration it *expects to receive, a refund liability, and an asset* related to subsequent product recovery.

MCQ-09426 On January 1, Year 1, Black Dog Corp. began operations and issued 30,000 shares of $5 par common stock for $9/share. On June 30, the company bought back 10,000 shares for $8/share. Then, on September 15, the company resold 5,000 shares for $12/share. What amount of total additional paid-in capital should Black Dog report on its December 31, Year 1 balance sheet if Black Dog uses the *cost method* to account for its treasury stock? A. $20,000 B. $120,000 C. $140,000 D. $165,000

This question asks for total APIC which includes APIC-CS and APIC-TS. Issue of CS January, Year 1 JE: Dr Cash $270,000 (30,000 x $9 SP) Cr CS 150,000 (30,000 x $5 par) Cr APIC-CS 120,000 (30,000 x $4) *Cost method - G/L recorded upon reissue* Buyback of TS June 30 JE: Dr TS 80,000 (10,000 x $8 PP) Cr Cash 80,000 *No G/L yet* Resold TS September 15th JE: $12 SP - $8 Cost = $4 Gain increases Equity Dr Cash 60,000 (5,000 x $12) Cr TS 40,000 (5,000 x $8 PP) Cr APIC - TS 20,000 (5,000 x $4 PP) *The 20,000 APIC-TS (Gain) is used to absorb future losses anything in excess reduces RE.*

Treasury Stock

Treasury stock (TS) is stock that has been issued and then repurchased by the issuer. Treasury stock is reported as a contra account to equity (Debit balance). There are two methods used to account for treasury stock under U.S. GAAP: the cost method and the par value method. Financing outflow Difference = timing of G/L *A Gain increases Equity* *A loss reduces Equity, then reduce RE* *Cost Method -G/L recorded upon reissue* Under the cost method, treasury stock is recorded at cost. The total cost of a treasury share is deducted from the total of the stockholders' equity on the balance sheet. When the TS is reissued for less than its acquisition price the loss is recognized by debiting APIC—treasury stock. Retained earnings is debited if there is not a sufficient balance in the APIC—TS account. When TS is reissued for more than its acquisition price the gain is recognized by crediting APIC—treasury stock. *Par Value Method - G/L calculated upon repurchase* Under the par value method, treasury stock is recorded at par value, and APIC is reduced for the amount of additional paid-in capital that was initially recognized on issuance. The total cost of the TS (par value) is deducted from the common stock account on the balance sheet. When treasury stock is reacquired for less than the original issue price (gain), APIC—treasury stock is recognized. When TS is reacquired for more than the original issue price (loss), any APIC—treasury stock is eliminated and then retained earnings is reduced. Gains and Losses *Gains and losses on treasury stock transactions are not reported on the income statement.* A company cannot report income dealing in its own stock. Treasury stock is not an asset. Treasury stock does not vote and does not receive dividends. *When you issue stock, repurchase stock, pay dividend it is not reported on the income statement.*

Accounting for Hedges: Reporting Gains and Losses

Type of Hedge Instrument -- Accounting for Changes in Fair Value No hedge designation -- Income Statement (IS) Fair value hedge -- IS offset by changes in FV of the hedged item Cash flow hedge Ineffective portion -- IS Cash flow hedge Effective portion (PUFI"E"R) -- In OCI then in Accumulated OCI in Equity *No hedge designation, Fair value hedge, Cash flow hedge Ineffective portion -> Income Statement* *Cash flow hedge Effective portion (PUFI"E"R) -- In OCI then in Accumulated OCI in Equity*

Valuation Allowance

U.S. GAAP Deferred tax assets are created by transactions that defer the tax benefits of expenses or transactions that recognize tax income before book income *(TI> BI)*. If it is *more likely than not* that part or all of a deferred tax asset will not be realized, a valuation allowance should be recognized to reduce the amount of the deferred tax asset. IFRS *IFRS prohibits the use of a valuation allowance.* Under IFRS, a deferred tax asset is *recognized when it is probable* that sufficient taxable profit will be available against which the temporary difference can be utilized.

Software Cost

U.S. GAAP Under U.S. GAAP, costs related to computer software developed to be sold, leased, or licensed, are *expensed until technological feasibility has been established and capitalized after that.* Capitalized costs are amortized using the greater of the straight-line method or a percentage of revenue basis. For computer software developed for internal use, costs in the preliminary project stage and costs incurred in training and maintenance are expensed. Costs after the preliminary project stage are capitalized. Capitalized costs are amortized on a straight-line basis. *- Expense as R&D all costs up to and including point of technological feasibility.* *- Technological feasibility is when detailed program is completed or working model.* *- Costs after technological feasibility are capitalized as software until production.* *- Production = inventory* IFRS IFRS does not provide specific guidance for computer software development costs. Under IFRS, *research costs* related to computer software development are *expensed* and *development costs* may be *capitalized* if certain criteria are met.

Balance Sheet Presentation DTA and DTL

U.S. GAAP and IFRS *Under both U.S. GAAP and IFRS, deferred tax assets and deferred tax liabilities are reported as non-current on the balance sheet.* Deferred tax assets and deferred tax liabilities may be netted if the entity has a legally enforceable right to offset current tax assets against current tax liabilities and the deferred tax assets and deferred tax liabilities relate to income taxes levied by the same tax authorities.

*U.S. GAAP vs IFRS Goodwill Consolidations*

Under IFRS, goodwill *(and noncontrolling interest as discussed above)* can be calculated using either the "partial goodwill" method. *Full goodwill method* The full goodwill method is the method used under U.S. GAAP, in which goodwill is calculated as followes: Goodwill = 100% FV of subsidiary - 100% FV of subsidiary's NA Partial Goodwill Method Under the partial goodwill method, goodwill is calculated as follows: Goodwill = % Acquisition cost - % FV of NA Acquired *Note: Partial goodwill and full goodwill methods differ only when the parent owns less than 100 percent of the subsidiary.*

Amortization

Under U.S. GAAP, intangible assets are reported at cost less amortization (finite life intangibles only) and impairment. Finite Life Intangibles: For intangible assets with finite lives, the cost of the asset less its residual value, is amortized over its useful life, generally using the straight-line method. *Succesful defense patent legal fees, legal and other patent registration fees can be capitalized. *Goodwill cannot be amortized, but is subject to the impairment test.* Indefinite Life Intangibles: Intangible assets that have no legal or economic lives are considered to have indefinite useful lives. *These intangible assets are not amortized but are reviewed for impairment periodically.* *If a trademark is expected to be renewed indefinitely, there will be no amortization expense on the books. Amortization is only recorded for intangible assets with a definite life.* IFRS: Under IFRS, intangible assets are reported using the *cost model (same as U.S. GAAP) or the revaluation model.* Revaluation Model Under the revaluation model, revalued intangible assets are reported at *fair value on the revaluation date less subsequent amortization and impairment.* *Revaluation losses are reported on the income statement* and *revaluation gains are generally reported in other comprehensive income.*

*Pension Expense Components (SIRAGE)*

Under U.S. GAAP, the amount of the net periodic pension cost is calculated using the following six components: + *S*ervice cost (current item - goes to compensation) + *I*nterest cost (on the projected benefit obligation) − *R*eturn on plan assets (expected or actual) + *A*mortization of unrecognized prior service cost ± (*G*ains) and losses ± Amortization of *E*xisting net (asset) or obligation = Net periodic pension cost *IR = Current items expensed* *AGE -> AOCI -> Amortized out to I/S* Dr Pension compensation expense $XXX Dr Net periodic pension cost XXX Cr Pension benefit liability XXX Cr Othe comprehensive income XXX *Pension compensation expense = S (Service cost current)* *Net periodic pension = IRAGE* Prior service cost, gains and losses, and existing net obligations or assets are amortized and charged to net periodic pension cost over a specified period of time. *Under U.S. GAAP, service cost is reported as an operating expense* on the income statement in the same line with *other compensation costs.* The other components of net periodic pension cost are presented on the income statement, separately or in total, *below income from operations.* *S = income from operations* *IRAGE = below income from operations* Under IFRS, defined benefit cost includes service cost and net interest on the defined benefit liability (asset). The components of defined benefit cost are generally reported separately on the income statement; there is no requirement that these amounts be aggregated and presented as one amount.

Corridor Approach

Under U.S. GAAP, unrecognized pension gains or losses are amortized over the average remaining service period if, at the beginning of the year, the gain or loss exceeds 10% of the greater of the beginning of the year PBO or the beginning of the year market-related value of plan assets (we will use the fair value of the plan assets in this example as the market-related value is not given and these amounts are approximately equal). At the beginning of the year, Rhino's PBO exceeds the fair value of the plan assets, so the net loss amortization is calculated as: unrecognized gain or loss Less: 10% of Greater of Beg. PBO/Plan Assets = Excess Excess / Average Remaining Service Life = Amortization of unrecognized gain or loss

*Changes in Accounting Estimate (Prospective Approach)*

Under the prospective approach, adjustments for changes in accounting estimate are made in the current and future accounting periods. They do not affect previous periods. Examples include: - Change in useful life - Change in salvage value - Settlement of litigation - *Change in Depreciation method, amortization, depletion* - *Change in inventory cost flow assumption to LIFO* *When a change in accounting principle is inseparable from a change in accounting estimate, it should be reported as a change in accounting estimate.*

MCQ-09394 *On July 1, year 1, after recording interest and amortization, Wake Company's shareholders converted $1,000,000 of its 10% convertible bonds into 50,000 shares of its $1 par value common stock. On the conversion date, the carrying amount of the bonds was $1,500,000, the market value of the bonds was $1,400,000, and Wake's common stock was publicly trading at$40 per share. Using the *market value method*, what amount of *additional paid-in capital* should Wake record as a result of the conversion?* A. $500,000 B. $1,500,000 C. $1,950,000 D. $1,450,000

Using the market value method, the conversion would be recorded as follows: Dr Bonds payable 1,000,000 Dr Premium on bonds 500,000 (1,500,000 Current CV - 1,000,000 BP issue) Dr Loss on conversion 500,000 Cr Common stock 50,000 (50,000 CS shares x $1 par) Cr APIC 1,950,000 (2,000,000 MKT Value BP - 50,000 CS converted) The total market value of the bonds is $2,000,000 (50,000 shares × $40/share). Note that the loss on conversion is equal to the difference between the market value of the stock issued and the book value of the bonds before conversion. Choice "1" is incorrect. This is the common stock at par value to be recorded at the time of conversion. See the journal entry above. Choice "3" is incorrect. This is the additional paid-in capital to be recorded at the time of conversion. See the journal entry above. Choice "4" is incorrect. This is the market value of the stock issued at the time of conversion, not the loss on conversion.

*Gain = Buy subsidiary at bargain/discount*

When a subsidiary is acquired for less than the fair value of 100 percent of the underlying assets acquired, the acquisition cost is first allocated to the fair value of 100 percent of the balance sheet accounts and the fair value of 100 percent of the identifiable intangible assets acquired. This creates a negative balance in the acquisition cost account, which is recognized as a gain in the period of the acquisition. 100% Fair value of subsidiary 100% Book value of subsidiary's net assets Difference − Adjust BS to fair value − Record intangibles at FV = Gain/Goodwill

*Inventory Costing Methods*

*LIFO is not permitted under IFRS.* *FIFO:* inventory consists of the most recent costs and the cost of goods sold consists of the older costs. *End Inventory and COGS "same" for perpetual or periodic.* *Lower COGS, Higher end inv., Higher NI, Higher Equity* *LIFO:* inventory consists of the older costs and the cost of goods sold consists of the most recent costs. U.S. - tax advantage - better matching. *Higher COGS, Lower end inv., Lower NI, Lower Equity* *Dollar-Value LIFO:* Inventory under dollar-value LIFO is measured in dollars and is adjusted for changing price levels. When converting from FIFO to dollar-value LIFO, a price index is used. *Price index = Ending inventory at current year dollar / Ending inventory at base year dollar* *Weighted Average—Used With Periodic Inventory:* Step 1: Weighted average cost per unit = COGAS / Number of units available for sale Step 2: Weighted average cost per unit x End Inv. = Cost of end Inv. *Moving Average Method—Used With Perpetual Inventory:* The moving average method computes the weighted average cost after each purchase by dividing the total cost of inventory available after each purchase (inventory plus current purchase) by the total units available after each purchase. The moving average is more current than the weighted average. A perpetual inventory system is necessary to use the moving average method.

Options and Warrants

A company has 1,000 stock options outstanding (for one share each), which are exercisable at $30 each. If the average market price is $50 per share, then the options are "in the money" and therefore dilutive. When the options are assumed to be exercised for the purposes of computing diluted EPS, it is assumed that the company will receive $30,000, which can be used to purchase 600 shares of stock ($30,000 / $50 share = 600 shares). The company will need to issue 400 new shares (1000 shares - 600 repurchased). The 400 newly issued shares will be added to the weighted average number of common shares outstanding when computing diluted EPS. Step 1: $30 Exercise price < $50 Avg. option price, thus dilutive Step 2: 1,000 x 1 x $30 = $30,000 rec'd Step 3: ($30,000 / $50 Avg. option pric) = 600 repurchaed Step 4: 1,000 issued - 600 repurchased = 400 Net increase to denominator 1,000 shares - (1,000 x $30 option price) / $50 Avg. option price = Shares added to denominator 1,000 - 600 = 400 shares

MCQ-09405 Mission Flowers Company had the following transactions for the year ended December 31: - Sales revenues of $775,000 - Operating expenses of $550,000 - Losses due to employee strike of $200,000. This was the first employee strike in history of the company. - Operating income of $100,000 from a subsidiary sold on November 1. The decision to dispose was made on February 28. The income was earned evenly over the ten months ended October 31. The company's effective tax rate is 35%. What amount should Mission Flowers report as income from continuing operations for the year ended December 31? A. $16,250 B $33,750 C. $81,250 D. $146,250

A. $16,250 Mission Flowers' income from continuing operations is calculated as follows: Sales revenue 775,000 Operating expenses (550,000) Loss from strike (200,000) Income before taxes 25,000 Income tax expense (8,750)* [25,000 x .35] Income from continuing operations 16,250 The loss from the strike is included in income from continuing operations. The operating income from the subsidiary sold during the year is not included in income from continuing operations, but is reported net of tax as income from discontinued operations, along with any gain or loss on the sale of the subsidiary. Choice "2" is incorrect. This answer incorrectly adds income tax expense to determine net income. Choice "3" is incorrect. This answer incorrectly includes the operating income from the subsidiary in income from continuing operations. Choice "4" is incorrect. This answer incorrectly excludes the loss from the strike from income from continuing operations.

MCQ-04489 Nemo Diving Products, Inc. usually stores its product inventory in a special area within its manufacturing facility. Due to a recent fire, all inventory this month was stored off site at a cost of $3,000. Some inventory items are normally purchased from Switzerland. This month's import duty was $2,000. This month's unreimbursable freight charges on product sold were $4,000. Given the above three costs, what amount(s) are chargeable to inventory verses chargeable to expense. Inventory Expense A. $2,000 $7,000 B. $9,000 $0 C. $5,000 $4,000 D. $3,000 $6,000

A. $2,000 $7,000 Import duty should be charged to inventory. The other two items are expensed as incurred. *The $3,000 warehousing cost is not chargeable to inventory since it is not a "usual" cost.* To be included in inventory, a cost must be usual, necessary and make the item ready for sale. The $4,000 freight out charge is a selling expense. Choices "2", "3", and "4" are incorrect, per the above explanation.

MCQ-09353 Money for Nothing Enterprises ("MNE") held the following available- for-sale debt securities during Year 2: Cost - Market Value 12/31/Y1 - Sales Price - Market Value 12/31/Y2 Alpha Corp. $50,000 $53,000 $57,000 NA Beta Corp. 35,000 30,000 NA $38,000 Omega Corp. 21,000 27,000 NA 24,000 There are no expected credit losses. What will MNE report as *unrealized gain* on available-for-sale securities on its Year 2 *statement of comprehensive income* (ignore taxes)? A. $2,000 B. $3,000 C. $6,000 D. $8,000

A. $2,000 *AFS securities are marked to market (Current Yr Mkt value - Cost). The marked to market adjustment runs through OCI bypassing the IS.* Yr 1 OCI: Alpha Corp. Mkt value Yr 1 - Cost = 3,000 unrealized gain Beta Corp. 30k Mkt value Yr 1 - 35k Cost = 5,000 unrealized loss Omega Corp. 27k Mkt value Yr 1 - 21k Cost = 6,000 unrealized gain Net OCI Yr 1: 4,000 unrealized gain Yr 2 OCI: Alpha Corp. Mkt value Yr 2 - Cost = None the asset was sold Beta Corp. 38k Mkt value Yr 2 - 35k Cost = 3,000 unrealized gain Omega Corp. 24k Mkt value Yr 2 - 21k Cost = 3,000 unrealized gain Net OCI Yr 2: 6,000 unrealized gain OCI was already reported as 4k in Yr 1 statement of OCI and recntly increased to 6k in Yr 2. You must make a net adjustment of 6k - 4k = 2k. This unrealized net gain goes into the Yr 2 statement of Comprehensive Income. Choice "2" is incorrect. This is the unrealized gain on the Alpha Corp. debt, which is written off in Year 2 when the security is sold. Choice "3" is incorrect. This is total accumulated other comprehensive income to be reported on the balance sheet related to the available-for-sale debt securities, not the current period unrealized gain on available-for-sale debt securities to be reported on the statement of comprehensive income. Choice "4" is incorrect. This is the unrealized gain to be reported in Year 2 on the Beta Corp. debt, not the total unrealized gain on all available-for-sale debt securities in Year 2.

MCQ-00250 *Grum Corp., a publicly-owned corporation, is subject to the requirements for segment reporting. In its income statement for the year ended December 31, Grum reported revenues of $50,000,000, operating expenses of $47,000,000, and net income of $3,000,000. Operating expenses include payroll costs of $ 15,000,000. Grum's combined identifiable assets of all industry segments at December 31, were $40,000,000.* *In its financial statements, Grum should disclose major customer data if sales to any single customer amount to at least:* A. $5,000,000 B. $4,000,000 C. $1,500,000 D. $300,000

A. $5,000,000 *Materiality test - Must satisfy 1 of 3* An identified operating segment is reportable if it has at least 10 percent of all operating segments: -Total Internal and external sales combines -Total profit or loss -Total assets. Choice "1" is correct. $5,000,000 (10% x $50,000,000 revenue). If revenue from a single external customer is 10% or more of total revenue, then the company should disclose this fact, the total amount of revenue from the customer, and the segment or segments reporting the revenues. The identity of the customer need not be disclosed. Choice "2" is incorrect. A customer is considered to be a major customer if sales to the customer are at least 10% of total revenue, not 10% of combined assets. Choice "3" is incorrect. Payroll costs are not used to determine which customers are major customers. Choice "4" is incorrect. A customer is considered to be a major customer if sales to the customer are at least 10% of total revenue, not 10% of net income.

MCQ-04216 At the end of year 1, a company reduced its inventory cost from $100 to its net realizable value of $80. As of the end of year 2, the inventory was still on hand and its net realizable value increased to $150. Under IFRS, what journal entry should the company record for year 2 to properly report the inventory value? A. Debit inventory for $20 and credit expense for $20. B. Debit inventory for $20, debit expense for $30, and credit retained earnings for $50. C. Debit inventory for $70 and credit expense for $70. D. Debit inventory for $70, credit retained earnings for $50, and credit expense for $20.

A. Debit inventory for $20 and credit expense for $20. Under IFRS, if a lower of cost or market write-down has occurred and subsequently the net realizable value of the inventory item increases, a recovery may be recorded to the extent of the original write-down. In this case, a $20 recovery is allowed to increase the inventory value back up to its original cost of $100 and decrease the expense (original loss recorded), but the total increase of $70 is not allowed as this would cause the inventory value to exceed its original cost. Choice "2" is incorrect. The recovery adjustment and increase to inventory of $20 is allowed, but no entry would be made to retained earnings as the recovery would affect the current year income statement; the debit to expense included in this answer would incorrectly record an additional expense rather than reducing the expense by $20 as should be done. Choice "3" is incorrect. A recovery adjustment of $70 is not allowed as it would exceed the original write-down of $20. Choice "4" is incorrect. A recovery adjustment of $70 is not allowed as it would exceed the original write-down of $20. In addition, no adjustment to retained earnings would occur as a result of this recovery write-up.

MCQ-05913 A manufacturing firm purchased used equipment for $135,000. The original owners estimated that the residual value of the equipment was $10,000. The carrying amount of the equipment was $120,000 when ownership transferred. The new owners estimate that the expected remaining useful life of the equipment was 10 years, with a salvage value of $15,000. What amount represents the depreciable base used by the new owners? A. $125,000 B. $120,000 C. $110,000 D. $105,000

B. $120,000 The depreciable base of an asset is cost minus salvage value. The cost of the equipment to the manufacturing firm was $135,000 and the salvage value estimate by the manufacturing firm was $15,000, so the depreciable base is $120,000 ($135,000 - $15,000). The carrying amount of the equipment was $120,000 when ownership transferred so you make the assumption that $15,000 had already been depreciated.

MCQ-06939 *Campbell Corp. exchanged delivery trucks with Highway Inc. Campbell's truck originally cost $23,000, its accumulated depreciation was $20,000, and its fair value was $5,000. Highway's truck originally cost $23,500, its accumulated depreciation was $19,900, and its fair value was $5,700. Campbell also paid Highway $700 in cash as part of the transaction. The transaction lacks commercial substance. What amount is the new book value for the truck Campbell received?* A. $3,000 B. $3,700 C. $5,000 D. $5,700

B. $3,700 Choice "2" is correct. The gain or loss is calculated by subtracting the net book value of the asset surrendered from its fair value. The fair value of the asset surrendered must always equal the fair value of the asset received, even if only one of these amounts is given. In this fact pattern, the net book value of Campbell's truck is $3,000 ($23,000 − $20,000) and its fair value is $5,000, so there is a gain of $2,000. However, because this exchange lacks commercial substance, Campbell will only recognize a gain if it received boot/cash. If boot/cash is given, or there is no boot/cash in the transaction, then no gain is recognized. Because Campbell pays cash of $700, no gain is recognized. Therefore, the journal entry to record the transaction and "plug" for the book value of the Truck received by Campbell is: Dr New truck (plug) 3,700 Dr Accumulated depreciation 20,000 Cr Old truck 23,000 Cr Cash 700

Janson traded stock in Flax Co. during Year 1 as follows, Number of shares purchased (sold): Price per share: February 3, Year 1 1,100 $11 April 15, Year 1 2,500 9 May 28, Year 1 (750) 13 July 5, Year 1 1,400 12 September 30, Year 1 (4,000) 15 No other transactions took place for Flax during the remainder of the year. At December 31, Year 1, Flax is trading at $10 per share. Janson trades securities on a last in, first out basis. What amount is the net value of the investment in Flax at year-end? A. $2,750 B. ($250) C. $2,500 D. $3,750

C. $2,500 Choice "3" is correct. Equity securities are marked to fair value at the financial statement date. Based on activity during the year, 250 shares of Flax Co. are held by Janson at year-end. *Based on a trading price of $10 on December 31*, the value of the investment is $2,500. Choice "1" is incorrect. $2,750 is the value of the shares based on a purchase price of $11. However, equity securities must be valued based on their market value as of the financial statement date, which in this case is $10 per share. Choice "2" is incorrect. Based on a last in, first out basis, 250 shares at a cost of $11 remain at year-end. The adjustment to mark the shares to fair value is an adjustment of ($250) but the ($250) does not reflect the value of the shares at year-end. Choice "4" is incorrect. $3,750 is the value of the 250 remaining shares based on the September 30 purchase price of $15. However, equity securities must be valued based on their market value as of the financial statement date, which in this case is $10 per share.

MCQ-01241 *Duke Co. reported cost of goods sold of $270,000 for Year 2.* *Additional information is as follows:* *December 31 January 1* *Inventory $ 60,000 $ 45,000* *Accounts payable 26,000 39,000* *If Duke uses the direct method, what amount should Duke report as cash paid to suppliers in its Year 2 statement of cash flows?* A. $272,000 B. $242,000 C. $298,000 D. $268,000

C. $298,000 Choice "3" is correct. $298,000 cash paid to suppliers under the direct method. COGS 270,000 Increase in inventory 15,000 Decrease in accounts payable 13,000 = 298,000 The increase in inventory is added to cost of goods sold because an inventory increase means net purchases of inventory, which results in a cash outflow to suppliers. The decrease in AP is also added because AP decreases when cash payments are made to suppliers. *I'm assuming you assume the payment to suppliers relates to COGS thus increasing it.*

MCQ-09368 Money for Nothing Enterprises ("MNE") held the following available- for-sale debt securities during Year 2: Cost - Market Value 12/31/Y1 - Sales Price - Market Value 12/31/Y2 Alpha Corp. $50,000 $53,000 $57,000 NA Beta Corp. 35,000 30,000 NA $38,000 Omega Corp. 21,000 27,000 NA 24,000 There are no expected credit losses. What will MNE report as *accumulated other comprehensive income* on its 12/31/Y2 *balance sheet* (ignore taxes)? A. $2,000 B. $3,000 C. $6,000 D. $8,000

C. $6,000 Classification: Available-for-sale stocks and bonds Balance Sheet: Current or non-current *Reported: Fair value at balance sheet date* *Unrealized Gain/Loss: Other comprehensive income PUFIER* Realized Gain/Loss: Realized gain/loss in income statement or Unrealized gain/loss is reversed *AFS securities are marked to market (Current Yr Mkt value - Cost). The marked to market adjustment runs through OCI by passing the IS.* Alpha Corp. Mkt value Yr 2 - Cost = None the asset was sold Beta Corp. 38k Mkt value Yr 2 - 35k Cost = 3,000 unrealized gain Omega Corp. 24k Mkt value Yr 2 - 21k Cost = 3,000 unrealized gain Net OCI Yr 2: 6,000 unrealized gain Accumulated other comprehensive income on 12/31/Y2 balance sheet requires the total OCI being 6,000.

MCQ-09280 Alvarado Company had the following common stock balances and transactions during the current year: Jan. 1 Common stock outstanding 60,000 March 1 Issued a 10% common stock dividend 6,000 May 1 Issued common stock 9,000 Nov. 1 Issued common stock for cash 3,000 Dec. 1 Common stock outstanding 78,000 What was Alvarado's current year weighted average shares outstanding for basic EPS? A. 78,000 B. 73,250 C. 72,500 D. 71,500

C. 72,500 Jan. 1 60,000 x 12/12 =60,000 March 1 6,000 x 12/12 = 6,000 Stock dividend or a stock split is treated as if it had happened at the beginning of the year May 1 9,000 x 8/12 = 6,000 Nov. 1 3,000 x 2/12 = 500 Total WACSO = 72,500 60,000 shares were outstanding at the beginning of the year. On March 31, they issued a 10% stock dividend; *a stock dividend or a stock split is treated as if it had happened at the beginning of the year.* So now there are 66,000 (60,000 + 6,000) weighted average shares outstanding. Another 9,000 shares were issued at May 1. That issuance is weighted for the 8 months that they were outstanding; 8/12 x 9,000 shares is 6,000 more shares, for a total of 72,000 (66,000 + 6,000) weighted average shares outstanding. Another 3,000 shares were issued on November 1. They were outstanding for 2 months; 2/12 x 3,000 shares is another 500 shares, for a total of 72,500 (72,000 + 500) weighted average shares outstanding.

MCQ-09322 *In its year-end income statement, Black Knights Company reported cost of goods sold of $450,000. Changes occurred in several balance sheet accounts during the year as follows:* Inventory $160,000 decrease Accounts payable-suppliers 40,000 decrease *What amount should the Black Knights Company report as cash paid to suppliers in its cash flow statement, prepared under the direct method?* A. $250,000 B. $330,000 C. $570,000 D. $650,000

COGS $450,000 Outflow Inventory $160,000 Inflow Accounts payable-suppliers 40,000 Outflow COGS is assumed to be an outflow because it must be paid to vendors still. When inventory decreases by $160,000 its assumed inventory has been sold and cash has been received thus is an inflow. When accounts payable decreased by $40,000 it is assumed cash went out to lower the liability thus and outflow.

Component Depreciation (IFRS Only)

Component depreciation is required under IFRS, not by GAAP. Separate significant components of a fixed asset with different lives should be recorded and depreciated separately. The carrying amount of parts or components that are replaced should be derecognized.

MCQ-08718 Charm Co. owns a delivery truck with an original cost of $10,000 and accumulated depreciation of $7,000. Charm acquired a new truck by exchanging the old truck and paying $2,000 in cash. The new truck has a fair value of $5,000 at the time of the exchange. What amount of gain or loss should Charm recognize? A. $3,000 loss. B. $2,000 gain. C. $2,000 loss. D. $0

D. $0 Gains and losses are recognized in exchanges having commercial substance and are computed as the difference between the fair value of the acquired asset and the book value of the asset given up. At the time of exchange, the truck has a depreciated cost of $3,000 (historical cost of $10,000 less accumulated depreciation of $7,000). The exchange of the truck, plus $2,000 cash, results in a total decrease of $5,000 in net assets. The fair value of the acquired truck determines the cost to record at acquisition, but as the acquired cost of $5,000 is equal to the book value of what is exchanged, there is no gain or loss on the transaction. Journal entry to record the exchange: Dr Asset (new truck @ FV ) 5,000 Dr Accumulated depreciation 7,000 Cr Asset (old truck @ cost) 10,000 Cr Cash 2,000 Choice "1" is incorrect. The $3,000 loss agrees to the book value of the exchanged truck, but does not consider the fair value of the new truck acquired or the book value of the old truck exchanged. Choice "2" is incorrect. The $2,000 gain ignores the cash that is provided in addition to the truck that is exchanged. The cash provided is consideration for the equipment and must be accounted for in the journal entry. Choice "3" is incorrect. The $2,000 loss ignores the impact of the cash provided in exchange for the new truck. However, the loss is calculated incorrectly and would be a gain if the cash were ignored in the exchange transaction.

MCQ-01233 *Lino Co.'s worksheet for the preparation of its Year 2 statement of cash flows included the following:* *December 31 -- January 1* *Accounts receivable $29,000 $23,000* *Allowance for uncollectible accounts 1,000 800* *Prepaid rent expense 8,200 12,400* *Accounts payable 22,400 19,400* *Lino's Year 2 net income is $150,000. What amount should Lino include as net cash provided by operating activities in the statement of cash flows?* A. $151,000 B. $148,600 C. $145,400 D. $151,400

D. $151,400 *Indirect Method CF*: Step 1: NI (starting point) Step 2: Analyze CA ↑CA = ↓CF ↓CA = ↑CF Analyze CL ↑CL = ↑CF ↓CL = ↓CF Step 3: Add non-cash expenses such as dereciation, amortization and bad debt Step 4: Add losses Deduct Gains Deduct equity earnings of affiliate Choice "4" is correct. $151,400 net cash provided by operating activities, as follows: Net income 150,000 Increase in A/R (6,000) [↑CA = ↓CF] Increase in allowance 200 [↑CL = ↑CF] Decrease in prepaid 4,200 [↓CA = ↑CF] Increase in A/P 3,000 [↑CL = ↑CF] = 151,400 Choice "1" is incorrect. This answer is computed by incorrectly subtracting the increase in the allowance for uncollectible accounts. The allowance is a contra-asset and, like a liability, an increase in contra-asset should be added to net income to compute operating cash flow. Choice "2" is incorrect. This answer is computed by incorrectly adding the increase in AR and incorrectly subtracting the increase in the allowance, the decrease in the prepaid and the increase in AP. Choice "3" is incorrect. This answer is computed by incorrectly subtracting the increase in AP. The increase in AP should be added because an increase in AP is a source of cash and increases operating cash flow.

MCQ-00298 Anders Co. uses the moving-average method to determine the cost of its inventory. During January, Anders recorded the following information pertaining to its inventory: Unit -- Units Cost -- Total Cost Balance on 1/1 40,000 $5 $200,000 Sold on 1/17 35,000 NA NA Purchased on 1/28 20,000 8 160,00 160,000What amount of inventory should Anders report in its January 31 balance sheet? A. $150,000 B. $200,000 C. $162,500 D. $185,000

D. $185,000 $185,000 inventory cost at 1/31. Rule: The moving-average method assumes the company has perpetual records. A new weighted-average cost is computed after each purchase and issues are priced at the latest weighted average cost. UnitsUnit CostTotal Cost Balance 1/1 40,000 $5.00 $200,000 Sold 1/17 (35,000) 5.00 (175,000) Balance 1/17 5,000 5.00 25,000 Purchased 1/28 20,000 8.00 160,000 Balance 1/31 25,000 7.40 $185,000 In this example, the next units sold would be priced at $7.40, the new weighted average cost.

MCQ-00196 Brock Corp. reports operating expenses in two categories: (1) selling and (2) general and administrative. The adjusted trial balance at December 31 included the following expense and loss accounts: Accounting and legal fees $120,000 Advertising 150,000 Freight out 80,000 Interest 70,000 Loss on sale of long-term investment 30,000 Officers' salaries 225,000 Rent for office space 220,000 Sales salaries and commissions 140,000 One-half of the rented premises is occupied by the sales department. Brock's total selling expenses are: A. $370,000 B. $400,000 C. $360,000 D. $480,000

D. $480,000 Accounting and legal fees $120,000 *(general and administrative)* Advertising 150,000 *(selling)* Freight out 80,000 *(selling)* Interest 70,000 *(non operating)* Loss on sale of long-term investment 30,000 *(non operating)* Officers' salaries 225,000 *(general and administrative)* Rent for office space 220,000 *(1/2 selling, 1/2 general and administrative)* Sales salaries and commissions 140,000 *(selling)* Advertising 150,000 Freight out 80,000 Office space 110,000 (1/2 x $220,000) Sales salaries and commissions 140,000 Total selling expenses $480,000 Note: Only one-half of rent for office space was used for sales office.

MCQ-09391 Selected financial results from Water Works Inc.'s five operating segments were as follows: Segment -- Total revenues -- Total profit -- Total assets Rain $100,000 $70,000 $300,00 Snow 130,000 30,000 350,000 Ice 600,000 300,000 700,000 Hail 95,000 15,000 190,000 Steam 400,000 220,000 550,000 The company had no intersegment sales. Which operating segment(s) is (are) deemed to be reportable segments? A. Ice only. B. Ice and Steam. C. Ice, Steam, and Snow. D. Ice, Steam, and Snow and Rain.

D. Ice, Steam, and Snow and Rain. An operating segment is considered to be a reportable segment if it accounts for at least 10 percent of total revenues (external and intercompany) or total profits/losses or total assets. In this problem, 10 percent of total revenues is $132,500, so Ice and Steam qualify as reportable segments based on total revenues; 10 percent of total profits is $63,500, so Rain, Ice, and Steam qualify based on total profits; and 10 percent of total assets is $209,000, so Rain, Snow, Ice, and Steam qualify based on total assets. Hail is the only operating segment that does not qualify as a reportable segment based on the 10 percent size tests. Note that Rain, Snow, Ice, and Steam together account for $1,230,000 or 93 percent of total outside sales, which means that Water Works is reporting sufficient segment information based on the rule that reportable combined outside sales must be at least 75 percent of total outside sales. Choices "1", "2", and "3" are incorrect, based on the above explanation.

Sale of Available for Sale Debt Securities

Impairment on available-for-sale securities is accounted for differently from impairment on held-to-maturity securities because the investor has the option to sell an available-for-sale security if the loss on the sale will be less than the expected credit loss. As a result, the credit loss reported in net income on an available-for-sale security is limited to the amount by which fair value is below amortized cost. Any additional loss is reported as an unrealized loss in other comprehensive income. Facts: Cost $100 FV 1/01/Year 1 120 *(120 - 100 = 20 originally in OCI)* Sold 9/15/Year 1 150 *(150 - 100 = 50 IS)* Dr Cash 150 Dr Unrealized gain ("U" in PUFIER) 20 *(reverse OCI)* (120 FV - 100 Cost) Cr Available for sale debt securities 120 Cr Realized gain 50 *(150 Sales price - 100 Cost) (goes on IS)*

*Comprehensive Income*

NI + OCI = Comprehensive Income Step 1: OCI Includes (PUFIER): *P*ension adjustments *U*nrealized gains and losses on AFS debt securities *F*oreign currency items (Foreign currency translation gains/losses) *I*nstrument specific credit risk *E*ffective portion of cash flow hedge gain or loss *R*evaluation surplus/gain (IFRS Only) = OCI before tax *Step 2: OCI before tax x Tax expense = Net OCI* *Comprehensive income includes all changes in equity during a period, except those resulting from investments by owners and distributions to owners.*

*Direct vs Indirect Statement of Cash Flows*

The direct method is the preferred method of presentation by US GAAP and IFRS. *Ignore NI, depreciation, and gain or loss.* *A reconciliation is required as supplemental disclosure for direct method SCF.* Direct: Cash received from customers (starting point) Add Sales of trading securities Add Interest revenue & dividend revenue Subtract Interest paid to vendors Subtract cash paid to employees Subtract interest and taxes paid Subtract SGA expenses Indirect: *Ignore cash received and cash paid* Step 1: NI (starting point) Step 2: *Operating Assets = All CA except cash & cash equivalnets include trading sec., exclude AFS & HTM.* ↑CA = ↓CF ↓CA = ↑CF *Operating Liabilities = All non-interest bearing debt include Deferred Tax Liability.* ↑CL = ↑CF ↓CL = ↓CF Step 3: Add non-cash expenses such as dereciation, amortization and bad debt Step 4: Add losses Deduct Gains Deduct equity earnings of affiliate (own 20% -50%) *Under the indirect method SCF supplemental schedule must show income taxes paid, interest paid. *

*Fundamental Qualitative Characteristics*

The fundamental qualitative characteristics of useful financial information are *relevance and faithful representation.* Both characteristics must be present for financial information to be useful. *Relevance:* Financial information is relevant if it is capable of making a difference in the decisions made by the users. To be relevant, financial information must have predictive value and/or confirming value, and must be material *(PCM)*. 1. *P*redictive value 2. *C*onfirmatory value 3. *M*aterial *Faithful Representation:* To be useful, financial information must faithfully represent the reported economic phenomena. Faithful representation requires financial information to be *complete, neutral, and free from error.* Although perfect faithful representation is generally not achievable, these characteristics must be maximized. 1. Complete 2. Neutral 3. Free from error

Double Declining Balance Depreciation

The salvage value is not considered upfront; it is considered at the end. The asset should never be depreciated below the estimated salvage value. Cost: $50,000 Salvage value: $4,000 Useful life: 5yrs Step 1: (Cost / Estimated useful life) = Straight line depreciation exp x 2 ($50,000 / 5) = $10,000 SL depreciation per yr Step 2: $10,000 / $50,000 = 20% x 2 = *40% DDB deprication rate* Beg.BV × Rate = DDB Depreciation for the period Beg BV - % DDB - Depri exp - Accum depri - End BV 50,000 .40 20,000 20,000 30,000 30,000 .40 12,000 32,000 18,000 Yr 1: $50,000 Beg. BV x .40 = 20,000 depri exp. $50,000 - 20,000 depri exp. =$30,000 End BV Accumulated depri Yr 1 = 20,000 Yr 2: $30,000 BV x .40 = 12,000 depri exp. $30,000 - 12,000 depri exp. =$18,000 End BV Accumulated depri Yr 2 = 20,000 + 12,000 = 32,000

Reporting Fixed Assets

Under U.S. GAAP, the carrying value of a fixed asset is calculated as follows: Carrying value = Historical cost − Accumulated depreciation − Impairment Under IFRS, fixed asset carrying value can be calculated using the cost model or the revaluation model as follows: CV (revaluation model) = FV on revaluation date - Subsequent accumulated depreciation - Subsequent impairment When fixed assets are revalued under IFRS, *revaluation losses are reported on the income statement* and *revaluation gains are reported in other comprehensive income* as revaluation surplus. When the fair value of a revalued asset differs materially from its carrying amount, a further revaluation is required. Initial gain = OCI PUFIE *R* *(Revaluation gain IFRS only)* *Initial loss Income statement*

MCQ-09419 On March 1, Year 1, LuxWear Inc. had beginning inventory and purchases, at cost of $50,000 and $20,0000, respectively. The beginning inventory and purchases has a retail value of $75,000 and $30,000, respectively. The company has sales of $60,000, as well as markups of $6,000 and markdowns of $10,000. What would LuxWear report as the cost of its ending inventory March 31, Year 1 using the conventional retail method? A. $25,830 B. $27,470 C. $28,290 D. $41,000

Under the conventional retail method, the cost of ending inventory is calculated as follows: At Cost -- At Retail Beg. Inv 50,000 75,000 + Purchases 20,000 30,000 + Markups NA 6,000 Available for sale 70,000 111,000 = 63% Cost/Retail ratio - Sales NA (60,000) - Markdowns NA (10,000) Ending inventory at retail NA 41,000 Ending inventory at cost 25,830 NA ($41,000 x 63%)

MCQ-09267 *Giddens Company adopted the dollar-value LIFO inventory method on December 31, Year 1. On December 31, Year 1, Giddens' inventory was in a single inventory pool and was valued at $400,000 under the dollar-value LIFO method. Inventory data for Year 2 are as follows: 12/31 Year 2 inventory at year-end prices $550,000 Price index at 12/31 Year 2 (base year Year 1) 110 Giddens' inventory at dollar-value LIFO at December 31, Year 2 is:* A. $440,000 B. $510,000 C. $500,000 D. $550,000

b. $510,000 The first step would be to deflate the inventory at Year 2 year-end prices using the index to $500,000 ($550,000 / 1.1). The inventory at the previous year-end (Year 1) was $400,000. The increase for the year is thus $100,000 ($500,000 − $400,000). The second step would be to inflate the increase for the year by the same index to $110,000 ($100,000 × 1.1) and then to add the inflated increase for the year to the beginning of year inventory at dollar-value LIFO for a total of $510,000 ($400,000 + $110,000).

Simple Bank Reconciliation (Cash & cash equivalents)

*Banks adjustments* Company's records (starting point) Deposits in transit: Add to bank Outstanding checks: Deduct from bank *Bank add it, LOC (less outstanding checks)* *Book adjustments* Bank stmt. cash bal (starting point) Service charge: Deduct from books Bank collections: Add to books Non-sufficient funds (NSF): Deduct from books Interest Income: Add to books *Bank or both adjustments* Errors Errors made by either *bank or the depositors* are another cause for a difference.

*Investments in Debt Securities*

*Trading Securities:* Trading debt securities are intended for active trading. The portfolio itself is carried at cost and is *reported at fair value in the financial statements* through the use of a valuation account. *Unrealized gains or losses are reported on the income statement.* Dr Unrealized loss $XXX (goes on IS) Cr Trading debt securities $XXX (FV) or Dr Trading debt securities $XXX (goes on BS) Cr Unrealized gain $XXX (FV) *Available-for-Sale (AFS) Securities:* AFS debt securities are carried at cost and are *reported at fair value in the financial statements* through the use of a valuation account. *Unrealized gains or losses are reported in other comprehensive income* (the "U" in PUFIER). Dr Unrealized loss $XXX (goes on OCI/Equity) Cr Available-for-sale debt securitie $XXX (FV) or Available-for-sale debt securities $XXX (goes on OCI/Equity) Unrealized gain $XXX (FV) *Held-to-Maturity (HTM) Securities:* The HTM classification is appropriate when the investor has the ability and intent to hold the debt securities to maturity. The portfolio itself is *reported in the financial statements at amortized cost.*

Accounts Receivable Without Recourse Vs Account Receivable With Recourse

A company may sell its receivables to a factor either with or without recourse. "With recourse" means the seller retains the risk of any losses on collection. "Without recourse" means that the buyer assumes the risk of any losses on collection. Without Recourse - true sale - remove AR from BS Without Recourse Journal entry: Dr Cash 94 Dr Due from $1 (factors margin) Dr Loss (plug) 5 Cr AR 100 With recourse - true sale or loan - *true sale must meet all qualifications* 1. Seller's obligation reasonably estimated "due from" 2. Transferor surrenders control 3. Transferor cannot be required to repurchase AR *If not not treated as a true sale you must treat as loan as if you're pledging receivables as collateral and disclose in the footnotes.*

MCQ-09258 Diamond Inc. purchased the following available-for-sale debt securities at par during Year 1: Values as of 12/31/ Year 1 Purchase price -- Fair value ABC Corp. $50,000 $55,000 XYZ Corp. $35,000 $30,000 On December 31, Year 1, Diamond determined that the present value of the principal and interest expected to be received on the investment in XYZ Corp. is $33,000. What will Diamond report as unrealized gain or loss on available-for-sale securities on its Year 1 statement of comprehensive income? A. $2,000 gain B. $5,000 gain C. $0 D. $3,000 loss

A. $2,000 gain ABC: Unrealized gain: $5,000 gain (Fair value $55,000 − Purchase price $50,000). XYZ: The total loss on XYZ is the $5,000 difference between the purchase price and the fair value at the end of the year. $2,000 of this loss will be recognized as a credit loss on the income statement because the present value of the investment is less than its amortized cost (i.e., its purchase price): $2000 loss = $33,000 present value − $35,000 amortized cost. The remaining $3,000 of the loss is recognized as an unrealized loss in OCI. Total impact to OCI: $2,000 gain ($5,000 gain from ABC − $3,000 loss from XYZ).

MCQ-08498 A company reported the following information for Year 1: Net income $34,000 Owner contribution 9,000 Deferred gain on an effective cash-flow hedge 8,000 Foreign currency translation gain 2,000 Prior service cost not recognized in net periodic pension cost 5,000 What is the amount of other comprehensive income for Year 1? A. $5,000 B. $43,000 C. $14,000 D. $15,000

A. $5,000 Choice "1" is correct. Other comprehensive income for Year 1 is $5,000. ($8,000 + $2,000 - $5,000). This includes the $8,000 of deferred gain on an effective cash-flow hedge, plus $2,000 of foreign currency translation gain, less the $5,000 of prior service cost not recognized in net periodic pension cost. The $5,000 of prior service cost would be a positive addition to comprehensive income in the year that it was amortized to net periodic pension cost. In this problem, it is not being recognized in net periodic pension cost. Choice "2" is incorrect. This answer adds net income of $34,000 to $9,000 of owner contribution. This is not the correct answer because other comprehensive income does not include net income or transactions from owners. Comprehensive income does include net income. The formula is: Net income + Other comprehensive income = Comprehensive income. Choice "3" is incorrect. The $14,000 includes $9,000 of owner contribution and $5,000 of prior service cost not recognized in net periodic net pension cost. The $9,000 of owner contribution is not included in other comprehensive income because other comprehensive income represents the change in equity from transactions related to nonowner sources. The $5,000 of prior service cost not recognized in net periodic pension cost would be a subtraction from other comprehensive income in Year 1 rather than an addition. Choice "4" is incorrect. The answer of $15,000 would include $8,000 of deferred gain on an effective cash-flow hedge plus $2,000 of foreign currency translation plus the $5,000 of prior service cost not recognized in net periodic pension cost. The $5,000 of prior service cost would be a positive addition to other comprehensive income in the year that it was amortized to net periodic pension cost, but not in this situation, where the prior service cost is not recognized in net periodic pension cost.

MCQ-05695 Tam Co. reported the following items in its year-end financial statements: Capital expenditures $1,000,000 Capital lease payments 125,000 Income taxes paid 325,000 Dividends paid 200,000 Net interest payments 220,000 term-24 What amount should Tam report as supplemental disclosures in its statement of cash flows prepared using the indirect method? A. $545,000 B. $745,000 C. $1,125,000 D. $1,870,000

A. $545,000 *Under the indirect method SCF supplemental schedule must show income taxes paid, interest paid. A reconciliation is required as supplemental disclosure for direct method SCF.* Choice "1" is correct. When the indirect method is used, a supplemental disclosure of cash paid for interest and income taxes is required. Tam will report total cash paid for interest and income taxes of $545,000 ($325,000 income taxes paid + $220,000 net interest payments). Choice "2" is incorrect. Under the indirect method, supplemental disclosures will include the cash paid for interest and income taxes, but will not include the dividends paid. Supplemental disclosure is not required for dividend payments because dividends paid is a line-item disclosure in the financing section. Choice "3" is incorrect. Under the indirect method, a supplemental disclosure of cash paid for interest and income taxes is required. However, a supplemental disclosure of capital expenditures and capital lease payments is not required because capital expenditures is a line-item disclosure in the investing section and the principal portion of capital lease payments is a line-item disclosure in the financing section. Choice "4" is incorrect. When the indirect method is used, a supplemental disclosure of cash paid for interest and income taxes is required. Tam will report total cash paid for interest and income taxes of $545,000 ($325,000 income taxes paid + $220,000 net interest payments). However, supplemental disclosure of dividends paid, capital expenditures, and capital lease payments is not required.

MCQ-09259 On May 15, Year 1, Moran Inc. approved a plan to dispose of a component of its business. It is expected the sale will occur on February 1, Year 2, at a selling price of $500,000, which was the current fair value of the component. During Year 1, disposal costs incurred by Moran totaled $15,000. The component had actual or estimated operating losses as follows: January 1 - May 14, Year 1 $130,000 May 15 - December 31, Year 1 50,000 January 1 - January 31, Year 2 15,000 The carrying amount of the components on May 15, Year 1 was $850,000. Before income taxes, what amount should Moran report for discontinued operations in its Year 1 Income Statement? A. $545,000 B. $365,000 C. $15,000 D. $380,000

A. $545,000 There would be an impairment loss on the component during Year 1. The net fair value of the component would be its fair value of $500,000 less the $15,000 disposal costs, for a total of $485,000. The impairment loss would be the net fair value of $485,000 minus the $850,000 carrying value of the component, for a total of $365,000. The results of operations would be reported for Year 1 as a loss of $180,000 ($130,000 + $50,000) and for Year 2 as $15,000. The amount that Moran should report for discontinued operations in its Year 1 income statement would be the loss from operations for Year 1 of $180,000 and the impairment loss for Year 1 of $365,000, for a total of $545,000.

*Bad Debt Expense*

Accounts receivable = Gross AR - AFUA = NRV Percentage of AR Method: Bad debts are estimated as a percentage of ending accounts receivable emphasis is on the valuation of the receivables. This results in the ending balance for allowances for doubtful accounts and the bad debt expense is the "plug." Aging of AR Method: A schedule is prepared categorizing accounts by the number of days or months outstanding. Each category's total dollar amount is then multiplied by a percentage representing uncollectibility based on past experience. The sum of the product for each aging category will be the desired ending balance in the allowance account. *The allowance for doubtful accounts account is a contra asset account, the allowance for doubtful accounts normal balance is a credit balance. So for an allowance for doubtful accounts journal entry, credit entries increase the amount in this account and debits decrease the amount in this account.* AFUA add Beginning balance add Recoveries add *BDE (plug)* less write offs = Ending balance AR (based on AR not expected to be collected) Entry to record BDE: Dr BDE Cr AFUA Write off JE: Dr AFUA Cr AR Recovery of AR JE: DR AR CR AFUA Collection on the account JE: Dr Cash Cr AR

Exchanges Having Commercial Substance (US GAAP)

An exchange has *commercial substance if the future risk, timing or amount of cash flows change as a result of the transaction.* A fair value approach is used. - *Gains/Losses are always recognized on exchanges having commercial substance.* - *Gains/Losses are the difference between the FV and BV of the old asset.* - The fair value of assets given up is assumed to be equal to the fair value of assets received, including any cash given or received in the transaction. Dr New asset (FV of old asset + cash given, if any) $XXX Dr Accumulated depreciation of asset given up XXX Dr Cash received (if any) XXX Dr Loss (if any) XXX Cr Old asset at historical cost XXX Cr Cash given (if any) XXX Cr Gain (if any) XXX

MCQ-12641 Light Co. had the following bank reconciliation at March 31: Balance per bank statement, March 31 $23,250 Add: Deposit in transit 5,150 Total balance 28,400 Less: Outstanding checks 6,300 Balance per books, March 31 $22,100 Additional information from Light's bank statement for the month of April is as follows: Deposits $29,200 Disbursements 24,800 All reconciling items at March 31 cleared through the bank in April. Outstanding checks at April 30 totaled $3,200. What is the amount of cash disbursements per books in April? A. $24,800 B. $21,700 C. $28,000 D. $27,900

April disbursements per bank statement$24,800 March checks cleared (paid) in April statement(6,300) Checks outstanding end of April 3,200 [have not cleared yet] Total cash disbursements in April $21,700 According to the bank statement, disbursements in April totaled $24,800. However, the question says that all outstanding checks from the March reconciliation *cleared (were paid) the bank in April.* This means that $6,300 of the $24,800 disbursement amount was recorded in the company's books during the month of March but paid in April. As a result, $18,500 ($24,800 - $6,300) of the April disbursements per the bank were actually recorded in the company's books during April and cleared the bank during April. The last component to consider are the outstanding checks in the April reconciliation. *These are checks written and recorded in the books that did not clear (have not been paid and are outstanding still) the bank.* Because they are not included in the adjusted disbursement amount of $18,500, the $3,200 of outstanding checks are added to determine total April disbursements per the books of $21,700. Choice "1" is incorrect. The $24,800 considers only the disbursements recorded by the bank during the month of April. This choice does not account for disbursements recorded in the company's books that did not clear the bank ($3,200), nor does it exclude the disbursements that cleared the bank in April that were recorded in the company's books in March ($6,300). Choice "3" is incorrect. The $28,000 includes the $24,800 disbursement total from the bank and adds in the $3,200 of outstanding checks in the April reconciliation. However, the choice does not reduce the bank disbursement total for the checks that were already recorded in the books during March ($6,300) and cleared the bank in April. Choice "4" is incorrect. The $27,900 accounts for the disbursements recorded in the company's books that did not clear the bank ($3,200), and accounts for disbursements that cleared the bank in April that were recorded in the company's books in March ($6,300). The error in the calculation is that the adjustments are the opposite of what they should be. The $6,300 is added to the bank disbursement total and the $3,200 is subtracted, resulting in an erroneous answer.

*Anti-Dilutive Securities*

Are stock options, warrants, convertible debt or convertible preferred stock that would increase EPS if exercised or converted to common stock Example: Converting preferred stock into common stock will create 3,250 new CS shares (6,500 PS × 0.5 CS conversion). The company will save $13,000 in preferred dividends that will no longer have to be paid. This ratio ($13,000 / 3,250 = $4.00/share) exceeds the Basic EPS of $2.74 calculated earlier, which implies that these convertible preferred shares are anti-dilutive. *Thus you do not include in the denominator when calculating dilutive EPS.*

MCQ-07223 On January 1 of the current year, Barton Co. paid $900,000 to purchase two-year, 8%, $1,000,000 face value bonds that were issued by another publicly-traded corporation. Barton plans to sell the bonds in the first quarter of the following year. The fair value of the bonds at the end of the current year was $1,020,000. At what amount should Barton report the bonds in its balance sheet at the end of the current year? A. $1,000,000 B. $1,020,000 C. $900,000 D. $950,000

B. $1,020,000 Choice "2" is correct. The bond investments are classified as trading securities because the bonds are held for the purpose of selling them in the near term. Trading securities are reported at fair value on the balance sheet. Choice "1" is incorrect. Trading securities are not reported at the face value of the bonds. Choice "3" is incorrect. Trading securities are not reported at the original cost of the bonds. Choice "4" is incorrect. Trading securities are not reported at the amortized cost of the bonds.

Regulation S-X

BS2 - Other 3 Regulation S-X outlines the form and content of financial statements to be included in SEC filings. Under Regulation S-X, I) annual financial statements filed with the SEC must be audited and must include *balance sheets for the two most recent fiscal years* II) and statements of income, changes in owners' equity, and cash flows for each of the *three fiscal years* preceding the date of the most recent audited balance sheet.

Deutsch Imports has three securities in its available for sale investment portfolio. Information about these securities as follows: Market value Security -- Cost -- 12/31/Yr 1 -- 12/31/Yr 2 NCB $78,000 $93,600 $100,000 TRR $117,000 $120,000 $0 Enson $58,500 $53,300 $50,700 TRR was sold in Year 2 for $127,400. Which of the following is correct? I. On its 12/31/Year 2 balance sheet, Deutsch should report the NCB stock at its fair value of $100,000. II. On its 12/31/Year 2 balance sheet, Deutsch should report an unrealized gain on the NCB stock of $22,000 in stockholders' equity. III. On its income statement for the year ending December 31, Year 2, Deutsch should report an unrealized gain on the NCB stock of $22,000. A. I only is correct. B. I and II only are correct. C. I and III only are correct. D. None of the listed are correct.

B. I and II only are correct. In this question, they are asking certain questions about the securities in an available-for-sale portfolio. There are three securities, one of which was sold during Year 2, but all of the statements are about one of the securities. Securities in an available-for-sale portfolio should be marked-to-market and any unrealized "holding" gain or loss should be included in other comprehensive income. Other comprehensive income is reported on the balance sheet as a part of accumulated other comprehensive income in the stockholders' equity section. Statement I says that NCB should be reported at its market value of $100,000. That statement is correct since the $100,000 is the market value at 12/31/Year 2 and the security would have been marked to market. Statement II says that an *unrealized holding gain of $22,000* would be reported in the *stockholders' equity section of the balance sheet*. The unrealized gain would be $22,000 ($100,000 cost − $78,000 at 12/31/Year 2), and it would be included in other comprehensive income as a part of accumulated other comprehensive income on the balance sheet. So, this statement is correct also, which means that choices "1" and "4" are both incorrect. Statement III says that an unrealized holding gain of $22,000 would be reported on the income statement. If the gain had been realized or if the security had been in the trading portfolio, it would have been reported on the income statement, but the gain was unrealized and the security is in the available-for-sale portfolio. So, this statement is incorrect, and choice "3" is incorrect.

MCQ-08610 Isle Co. owned a copy machine that cost $5,000 and had accumulated depreciation of $2,000. Isle exchanged the copy machine for a computer that cost $4,000. Isle's future cash flows are not expected to change significantly as a result of the exchange. What amount of gain or loss should Isle report and at what amount should it record the asset? A. $1,000 gain in the income statement; $4,000 asset in the balance sheet. B. No gain or loss in the income statement; $3,000 asset in the balance sheet. C. $1,000 gain in the income statement; $3,000 asset in the balance sheet. D. No gain or loss in the income statement; $4,000 asset in the balance sheet.

B. No gain or loss in the income statement; $3,000 asset in the balance sheet. Choice "2" is correct. This is an exchange that lacks commercial substance because a copy machine will have very similar cash flows to a computer. The copy machine is presumed to have the same fair value as the computer, which is $4,000. The book value of the copy machine at the time of the exchange was $3,000. The difference represents a gain of $1,000, but the gain is not recognized because no boot is received. The basis of the asset acquired will be equal to the basis of the old asset, which is $3,000. Choice "1" is incorrect. No gain is recognized because this is an exchange that lacks commercial substance and no boot is received. Choice "3" is incorrect. No gain is recognized because this is an exchange that lacks commercial substance and no boot is received. Choice "4" is incorrect. Although the new asset has a cost of $4,000, Isle must book the basis of the new asset at an amount equal to the basis of the old asset ($3,000).

MCQ-09393 Chili Co. had the following balances at December 31: Foreign currency translation gain $150,000 Unrealized loss on trading securities (35,000) Net income 650,000 Loss on discontinued operations (75,000) The company's effective tax rate is 40 percent. What amount should Chili Co. report as comprehensive income for the year ended December 31. A. $674,000 B. $719,000 C. $740,000 D. $800,000

C. $740,000 NI + OCI = Comprehensive Income Step 1: OCI Includes (PUFIER): *P*ension adjustments *U*nrealized gains and losses on AFS debt securities *F*oreign currency items *I*nstrument specific credit risk *E*ffective portion of cash flow hedge *R*evaluation surplus (IFRS Only) = OCI before tax *Step 2: OCI before tax x Tax expense = Net OCI* *Comprehensive income includes all changes in equity during a period, except those resulting from investments by owners and distributions to owners.* Comprehensive income includes all items included in net income, plus all other comprehensive income items (pension funded status changes, unrealized gains and losses on available-for-sale debt securities, foreign currency translation gains/losses, instrument-specific credit risk, and unrealized gains/losses on the effective portion of a cash flow hedge). The unrealized loss on the trading security and the loss on discontinued operations are already included in net income, so comprehensive income is: Net income 650,000 Foreign currency translation gain (net of tax)* 90,000 Comprehensive Income740,000 * Translation gain (net of tax) = $150,000 × (1 − 0.40) = $90,000 Choice "1" is incorrect. This answer incorrectly includes both the unrealized loss on the trading security and the loss on discontinued operations, which are already included in net income. Choice "2" is incorrect. This answer incorrectly includes the unrealized loss on the trading security, which is already included in net income. Choice "4" is incorrect. This answer fails to reduce the foreign currency translation gain by the effect of income taxes.

MCQ-04469 Zulu Transportation Inc.'s pension trustee provided the company with the following information for its defined benefit pension plan at December 31: Past service cost $240,000 Remeasurement of the net defined benefit liability (net gain) 75,000 Current service cost 385,000 Under IFRS, what amount would Zulu report in accumulated other comprehensive income related to its pension plan on its December 31 balance sheet? A. $165,000 B. $0 C. $75,000 D. $385,000

C. $75,000 Step 1: OCI Includes (PUFIER): *P*ension adjustments *U*nrealized gains and losses on AFS debt securities *F*oreign currency items (Foreign currency translation gains/losses) *I*nstrument specific credit risk *E*ffective portion of cash flow hedge *R*evaluation surplus (IFRS Only) = OCI before tax Choice "3" is correct. Under IFRS, gains and losses that are remeasurements of the net defined benefit liability are reported in other comprehensive income. Therefore, Zulu would report $75,000 in accumulated other comprehensive income. The past service cost and current service cost are both reported on the income statement under IFRS. Choice "1" is incorrect. Under IFRS, the past service cost of $240,000 is reported on the income statement when the plan is amended. Choice "2" is incorrect. Under IFRS, gains and losses that are remeasurements of the net defined benefit liability are reported in other comprehensive income. Choice "4" is incorrect. Under IFRS (and U.S. GAAP), current service cost is reported on the income statement in the period incurred.

MCQ-06049 *On December 31, Year 1, an entity revalued its buildings to their fair value of $2,700,000 and recorded a revaluation gain of $200,000. On December 31, Year 4, the buildings had a carrying value of $2,295,000 and a recoverable amount of $2,000,000. What amount of impairment loss will the entity report on its December 31, Year 4 income statement?* A. $0 B. $200,000 C. $95,000 D. $295,000

C. $95,000 Choice "3" is correct. When the buildings were revalued in Year 1, the $200,000 revaluation gain was booked to other comprehensive income as a revaluation surplus. Under IFRS, if a revalued asset becomes impaired, the impairment is recorded by first reducing any revaluation surplus to zero, with further impairment losses reported on the income statement. In this problem, the buildings were impaired on December 31, Year 4 because the $2,295,000 carrying value of the buildings exceeded the $2,000,000 recoverable amount. The $295,000 ($2,000,000 - $2,295,000) impairment loss is recorded by first reducing to zero the $200,000 revaluation surplus from the Year 1 revaluation, and then recorded the $95,000 remaining impairment loss on the income statement. Choice "1" is incorrect. An impairment loss must be recorded in Year 4 because the carrying value of the buildings exceeds the recoverable amount. Choice "2" is incorrect. The $295,000 ($2,000,000 recoverable amount - $2,295,000 carrying value) impairment is recorded by first reducing the $200,000 revaluation surplus in equity to zero and then recording the remaining $95,000 impairment loss on the income statement. Choice "4" is incorrect. When the $295,000 ($2,000,000 recoverable amount - $2,295,000 carrying value) impairment is recognized, $200,000 of the impairment will be recorded by reducing the $200,000 revaluation surplus in equity to zero and the remaining $95,000 impairment will be recorded on the income statement.

MCQ-09383 Which of the following statements regarding the accounting for held-to-maturity securities is incorrect? I. To classify an investment as held-to-maturity, the company must have either the intent or the ability to hold the security until maturity. II. If a debt security is purchased at par value, it will be valued and reported at the purchase price until it matures. III. A held to maturity security can be reported as either a current or non-current liability. A. I only. B. III only. C. I and III. D. I,II, and III.

C. I and III. Statement I is incorrect because the company must have both the intent AND the ability to hold the security until maturity. Statement III is incorrect as a held-to-maturity can be reported as either a current or a non-current asset. Statement II is a correct statement. Choices "1", "2", and "4" are incorrect, based on the above explanation.

MCQ-08501 An entity purchased new machinery from a supplier before the entity's year-end. The entity paid freight charges for the purchased machinery. The entity took out a loan from a bank to finance the purchase. Under IFRS, what is the proper accounting treatment for the freight and interest costs related to the machinery purchase? A. The freight and interest costs should be capitalized as part of property, plant, and equipment. B. The freight and interest costs should be immediately expensed. C. The freight cost should be capitalized as part of property, plant, and equipment, and the interest cost should be immediately expensed. D. The interest cost should be capitalized as part of property, plant, and equipment, and the freight cost should be immediately expensed.

C. The freight cost should be capitalized as part of property, plant, and equipment, and the interest cost should be immediately expensed. Under IFRS, the freight cost should be capitalized as part of the machinery's historical cost. Just like GAAP, the interest incurred to finance the purchase is expensed and not capitalized as part of the historical cost. Choice "1" is incorrect. The freight and interest costs are not both capitalized. Only the freight costs are capitalized as part of the cost of the machinery under GAAP and IFRS. Choice "2" is incorrect. The freight and interest costs are not both immediately expensed. The freight is capitalized, and the interest is expensed over the life of the loan. Choice "4" is incorrect. Under IFRS, the interest costs associated with the loan to purchase the machinery would not be capitalized as part of its historical cost. Interest is capitalized when the machinery is self-constructed and not purchased. This applies to U.S. GAAP as well.

MCQ-08471 Materiality and relevance are both defined by: A. The consistency in the application of methods over time. B. The perceived benefits to be denied that exceed the perceived costs associated with it. C. What influences or makes the difference to a decision maker. D. Quantitative criteria set by the Financial Accounting Standards Board.

C. What influences or makes the difference to a decision maker. The accountant's determination of materiality and relevance is based on professional judgment and is affected by the needs of those who will be using the financial statements to make decisions. Choice "1" is incorrect. Materiality and relevance are not defined by consistency in the application of methods over time. Consistency in the application of methods over time is a quality needed for the overall accounting process. Choice "2" is incorrect. The perceived benefits to be denied that exceed the perceived costs associated with it does not define materiality and relevance. The perceived benefits achieved that exceed the costs associated with them better describes the cost constraint, which holds that the benefits of financial reporting must be greater than the costs of obtaining and presenting the information. Choice "4" is incorrect. The Financial Accounting Standards Board does not establish quantitative criteria that define materiality and relevance.

MCQ-00365 The following trial balance of Trey Co. at December 31 income tax expense. Dr. Cr. Cash $ 550,000 Dr. Accounts receivable, net 1,650,000 Dr. Prepaid taxes 300,000 Dr. Accounts Payable $120,000 Cr. Common stock 500,000 Cr. Additional paid-in capital 680,000 Cr. Retained earnings 630,000 Cr. Foreign currency translation adjustment 430,000 Dr. Revenues 3,600,000 Cr. Expenses 2,600,000 Dr. Totals $ 5,530,000 $5,530,000 Additional information: - During the year, estimated tax payments of $300,000 were charged to prepaid taxes. Trey has not yet recorded income tax expense. There were no differences between financial statement and income tax income, and Trey's tax rate is 30%. - Included in accounts receivable is $500,000 due from a customer. Special terms granted to this customer require payment in equal semiannual installments of $125,000 every April 1 and October 1. In Trey's December 31 balance sheet, what amount should be reported as total current assets? A. $2,250,000 B. $2,200,000 C. $2,500,000 D. $1,950,000

Choice "4" is correct. $1,950,000 total current assets. IncomeStatement Revenues 3,600 Expenses ( 2,600) Pretax income 1,000 Tax at 30% ( 300) Net income 700 AJE Current Assets -- Prelim -- Dr (Cr) --Final Cash 550 NA 550 A/R 1,650 (250) 1,400 Prepaid tax 300 (300) 0 Total curr 2,500 (550) 1,950

MCQ-00521 During Year 1, Wall Co. purchased 2,000 shares of Hemp Corp. common stock for $31,500. The market value of this investment was $29,500 at December 31, Year 1. Wall sold all of the Hemp common stock for $14 per share on December 15, Year 2, incurring $1,400 in brokerage commissions and taxes. On the sale, Wall should report a realized loss in its income statement of: A. $1,500 B. $4,900 C. $3,500 D. $2,900

Choice "4" is correct. $2,900 realized loss in Year 2 income statement on sale of stock. Rule: Equity securities are reported at fair value with unrealized gains and losses included in earnings. Fair value becomes the new basis (revalued cost) for computing realized gains or losses upon sale. *Trading Investment* Original cost in Year 1 31,500 Unrealized loss charged to income in Year 1 (2,000) = Fair value at 12-31-Year 1 29,500 Sales proceeds ($14 x 2,000 SH) 28,000 Less brokerage commissions & taxes (1,400) 26,600 Fair value at 12-31-Year 1 29,500 Less Net sales proceeds (28,000 - 1,400) 26,600 = Realized loss in Year 2 income statement 2,900

*Summary of Investments in Debt Securities*

Classification: Trading stocks and bonds (CA & CFO) Balance Sheet: Current or non-current asset *Reported: Fair value at balance sheet date* *Unrealized Gain/Loss: Income statement Year 1: (Cost - Current Yr FV = Unrealized Gain/Loss)* Year 2: (Yr 2 FV - Yr 1 FV = Unrealized Gain/Loss)* Realized Gain/Loss: Income statement (Selling price -CV on balance sheet on time of sale) Classification: Available-for-sale stocks and bonds (CFI) Balance Sheet: Current or non-current asset *Reported: Fair value at balance sheet date* *Unrealized Gain/Loss: Other comprehensive income PUFIER* *Unrealized Gain/Loss are reported on the balance sheet as (Current Yr FV - Amortized cost = Unrealized Gain/Loss).* *Unrealized Gain/Loss are reported net of change for current year on the statement of Comprehensive Income (Current Yr OCI net change - Prior Yr OCI Net change)* *This is mind confusing topic, make sure you read question carefully.* Realized Gain/Loss: Realized gain/loss in income statement or Unrealized gain/loss is reversed Classification: Held-to-maturity bonds (CFI) Balance Sheet: Current or non-current asset *Reported: Amortized cost at balance sheet date* *Unrealized Gain/Loss: None on balance Sheet* Realized Gain/Loss: Not applicable

MCQ-09304 Ray Corporation had the following transactions during the year: - A $100,000 gain on reacquisition and retirement of long-term bonds. Ray frequently acquires and retires its debt. - A $500,000 loss on the disposal of its entire retail store business. Ray never abandoned any of its various businesses previously. It plans to operate only as a wholesaler in the future. - A $100,000 loss on the abandonment of assets that are no longer being used. In its current year income statement, what would be the total amount to be included in income from continuing operations under U.S. GAAP? A. $100,000 gain B. $600,000 loss C. $400,000 loss D. $0

D. $0 The $100,000 gain on reacquisition and retirement of long-term bonds is considered a normal business activity and is reported in income from continuing operations. The $100,000 loss on the abandonment of assets no longer being used is also a normal activity and is included in income from continuing operations. The combination of the gain and the loss results in $0. Choice "1" is incorrect. The $100,000 gain on the acquisition and retirement of long-term bonds is reported as an ordinary operating income item, but so is the loss on abandonment of assets. This answer ignores the loss on abandonment. Choice "2" is incorrect. The loss of $500,000 on disposal of a component of a business would normally be reported in discontinued operations (assuming that the portion of the business was actually a component, which this one seems to be). The $100,000 loss on abandonment of assets that are no longer being used would be reported in income from continuing operations. Choice "3" is incorrect. As explained above, the $500,000 loss on the disposal of the retail store business is reported in discontinued operations and the $100,000 gain on the acquisition and retirement of the bonds is reported in income from continuing operations.

MCQ-05105 On January 1, Year 3, Starlight Construction Co. began a construction project qualifying for capitalization of interest. The total amount spent on this project during Year 3 was $250,000, spent uniformly during the year. To help pay for construction, $200,000 was borrowed at 10% on January 1, Year 3, and funds not needed for construction were temporarily invested in short-term securities, yielding $3,000 in interest revenue. Other than the construction funds borrowed, the only other debt outstanding during the year was a $150,000, 10-year, 7% note payable dated January 1, Year 1. How much interest should be capitalized by Starlight during Year 3? A. $9,500 B. $22,000 C. $25,000 D. $12,500

D. $12,500 Compare avoidable interest to actual interest and capitalize the lower amount. Avoidable interest Total expenditures of $250,000/2 = $125,000 average total accumulated expenditures 125,000 * 0.10 rate on specific borrowing = $12,500 of avoidable interest *Since expenditures were incurred evenly throughout the year, the total has to be divided by 2 to get a weighted average.* Actual interest $200,000 * 0.10 = $20,000 $150,000 * 0.07 = $10,500 = $30,500 of actual interest Total actual interest cost = $30,500 > $12,500 avoidable interest Capitalize the lower amount. Interest earned on money invested is interest revenue. It does not affect the amount of interest expensed or capitalized.

MCQ-11133 *Mentor Co., a U.S. corporation, owned 100 percent of a Swiss corporation. The Swiss franc is the functional currency. The remeasurement of Mentor's financial statements resulted in a $25,000 gain at year-end. The translation of the financial statements resulted in a $40,000 gain at year-end. What amount should Mentor recognize as foreign currency gain in its income statement?* A. $40,000 B. $0 C. $65,000 D. $25,000

D. $25,000 Choice "4" is correct. The remeasurement method is used to convert the financial statements of a subsidiary from the foreign (local) currency to the functional currency. The starting point is the balance sheet and any gain/loss is recorded on the income statement. The $25,000 gain from remeasurement will therefore be reflected on the income statement. The translation method is used to convert the financial statements of a subsidiary from the functional currency to the reporting currency. The starting point is the income statement and any gain/loss is recorded in other comprehensive income (OCI). The $40,000 gain from translation will therefore be reflected in OCI. Choice "1" is incorrect. The $40,000 gain from translation should be reflected in OCI. Choice "2" is incorrect. The $25,000 gain from remeasurement should be reflected on the income statement. Choice "3" is incorrect. Only the $25,000 gain from remeasurement should be recognized on the income statement.

MCQ-09319 Allison Corporation's current year income from continuing operations before taxes was $1,000,000 before taking the following items into consideration: - Depreciation was understated by $100,000. - A strike by the employees of a supplier resulted in a loss of $200,000. This strike was the first such strike that Allison had encountered. - The inventory at December 31 of the prior year was overstated by $300,000. The inventory at December 31 of the current year was correct. - A flood in Allison's Houston facility destroyed equipment worth $500,000. The facility had just been rebuilt from damages that occurred in a flood in the prior year. What was Allison's adjusted income from continuing operations before taxes? A. $1,000,000 B. $900,000 C. $700,000 D. $500,000

D. $500,000 Income from continuing operations should be adjusted for all of the items. The $100,000 in additional depreciation needs to be deducted from income from continuing operations. The $200,000 loss from the strike by the supplier's employees and the $500,000 flood loss should be included in income from continuing operations. The inventory overstatement in the prior year results in a cost of goods sold overstatement in the current year, so $300,000 should be added back to income from continuing operations. Allison Corporations adjusted income from continuing operations is calculated as follows: Unadjusted income from continuing operations 1,000,000 Less: Additional depreciation (100,000) Less: Loss from strike (200,000) Plus: Adjustment to COGS for inventory misstatement 300,000* Less: Loss from flood (500,000) Adjusted income from continuing operations 500,000 Beg. inventory + purchases = COGAS - End inventory 300 = COGS -300 Sales - 300 COGS = - 300 Gross profit This causes an excess 300k decrease in NI and must be added back.

MCQ-09257 Tyler Co. paid $150,000 on January 1, Year 1, to purchase a $150,000, five-year bond at par value. Tyler classified the investment as held-to-maturity. On December 31, Year 1, Tyler determined that the present value of the interest and principal to be paid by the issuer over the life of the bond is $142,000. The fair value of the bond on December 31, Year 1, was $144,000. What amount should Tyler report as a credit loss in its Year 2 income statement? A. $0 B. $2,000 C. $6,000 D. $8,000

D. $8,000 For HTM security, the credit loss is equal to the difference between amortized cost and present value when the present value is less than the amortized cost. Amortized cost $150,000 - Present value 142,000 Difference 8,000 Choice "1" is incorrect. There is a credit loss because the present value is less than the amortized cost. Choice "2" is incorrect. The credit loss is not equal to the difference between the present value and the fair value of the bond. Choice "3" is incorrect. The $144,000 fair value is not used in the credit loss calculation because this is a held-to-maturity security. If the investment was classified as available-for-sale, the credit loss would have been limited to the difference between the amortized cost and the fair value of the bond.

MCQ-05217 *Which of the following factors determines whether an identified segment of an enterprise should be reported in the enterprise's financial statements?* *I. The segment's assets constitute more than 10% of the combined assets of all operating.* *II. The segment's liabilities constitute more than 10% of the combined liabilities of all operating segments.* A. II only. B. Neither I nor II. C. Both I and II. D. I only.

D. I only. *Materiality test - Must satisfy 1 of 3* An identified operating segment is reportable if it has at least 10 percent of all operating segments: -Total Internal and external sales combines -Total profit or loss -Total assets. For segment reporting, if an identified segment's assets constitute more than 10% of the combined assets of all operating segments, the segment should be reported. The same rule does not apply for the segment's liabilities. The candidate does have to remember the 10% and also the 10% of "what." Choice "1" is incorrect. For segment reporting, if an identified segment's assets constitute more than 10% of the combined assets of all operating segments, the segment should be reported. The same rule does not apply for the segment's liabilities. Choice "2" is incorrect. For segment reporting, if an identified segment's assets constitute more than 10% of the combined assets of all operating segments, the segment should be reported. The correct answer cannot be "Neither." Choice "3" is incorrect. For segment reporting, if an identified segment's assets constitute more than 10% of the combined assets of all operating segments, the segment should be reported. The same rule does not apply for the segment's liabilities, so the correct answer cannot be "Both."

MCQ-00274 *When the allowance method of recognizing uncollectible accounts is used, the entries at the time of collection of a small account previously written off would:* A. Have no effect on the allowance for uncollectible accounts. B. Decrease the allowance for uncollectible accounts. C. Increase net income. D. Increase the allowance for uncollectible accounts.

D. Increase the allowance for uncollectible accounts. A collection of a previously written-off account receivable would increase the "allowance" account, which is a credit balance account. *The allowance for doubtful accounts account is a contra asset account, the allowance for doubtful accounts normal balance is a credit balance. So for an allowance for doubtful accounts journal entry, credit entries increase the amount in this account and debits decrease the amount in this account.* Entry to record BDE: Dr BDE Cr AFUA Write off JE: Dr AFUA Cr AR *This would be considered the two JE at the time of collection of a small account previously written off.* Recovery of AR JE: DR AR CR AFUA Collection on the account JE: Dr Cash Cr AR

MCQ-09274 Lyon Company has the following transactions in the current year. Assuming that all the transactions are material, which of them will most likely have no effect on current year net income? A. The sale of a factory building that was contributed by a shareholder in the prior year. B. The settlement of litigation over an accident in the prior year but for which a loss had not previously been considered to be probable and reasonably estimable. C. The determination that certain junk bonds purchased on a speculative basis several years previous were worthless. D. The collection of a receivable from a customer whose account was written off in the prior year by a charge to the allowance for bad debts account.

D. The collection of a receivable from a customer whose account was written off in the prior year by a charge to the allowance for bad debts account. This is the collection of an account receivable previously written off. The original write off would have been Dr Allowance and Cr Accounts Receivable. The collection would have been recorded in two transactions. The first would be to reestablish the account as Dr Accounts Receivable and Cr Allowance. The second would be to treat the collection as if the account had never been written off as Dr Cash and Cr Accounts Receivable. The net of the two Journal Entries is Dr Cash and Cr Allowance. There is nothing here that could potentially affect income. Choice "1" is incorrect. This is the sale of a factory building that was contributed by a stockholder. The original contribution would have been Dr Factory building and Cr Additional Paid-in Capital. Once the factory building was recorded, it would be accounted for regardless of the way it was acquired in the first place. Sale of the building would normally have involved a gain or a loss which would have had an effect on net income. Choice "2" is incorrect. This is a settlement of litigation. Previously, there was no liability recognized since the liability was not considered probable and reasonably estimable. Once the loss was certain (the litigation was settled), a loss would have been recognized. Choice "3" is incorrect. This is a write-off of junk bonds purchased previously. Regardless of how "junky" the bonds were, they cost something. Once the bonds were determined to be worthless, that cost would have been written off as a loss.

Earning per share (EPS)

Must present EPS on face of income statement. Simple capital structure: Have only CS and no securities that can become CS. Complex Capital Structure: If securities that can be converted into common stock, inclusive of convertible preferred stock, convertible bonds, options, and warrants. Must present basic and diluted per share amounts (assuming that there is dilution). Avg. market price > exercise price, thus dilutive Basic EPS = (NI - PS dividends) / WACSO *Diluted EPS = (NI - PS dividends) + Saved PS dividends + Saved Interest expense net of tax / Conversion of all potential WACSO* *Noncumulative PS dividends: use dividends declared that Yr* *Cumulative PS dividends: use amount that accumulates that Yr regardless of amount declared or paid.* *Stock dividends and stock splits are treated retroactively and treated to earliest Yr presented.*

Sale of Trading Debt Securities

Realized gains/losses are recognized in net income when a debt security is sold or impaired. Dr Cash $XXX Cr Trading debt securities $XXX Cr Realized gain $XXX (goes on income statement) or Dr Cash $XXX Dr Realized loss $XXX (goes on income statement) Cr Trading debt securities $XXX

Exchanges of Similar Assets and Dissimilar Assets (IFRS)

Under IFRS, nonmonetary exchanges are characterized as exchanges of similar assets and exchanges of dissimilar assets. *Exchanges of dissimilar assets are regarded as exchanges that generate revenue* and are accounted as *having commercial substance* under U.S. GAAP. Exchanges of similar assets are not regarded as exchanges that generate revenue and *no gains are recognized.*

Going Concern Assumption

Under U.S. GAAP, preparation of financial statements presumes that the reporting entity will continue as a going concern (to stay in business). Under this presumption, financial statements are prepared under the going concern basis of accounting. *If an entity's liquidation is imminent (and the entity is therefore no longer considered to be a going concern), financial statements are prepared under the liquidation basis of accounting.* Management is required to evaluate whether there is substantial doubt about an entity's ability to continue as a going concern within one year after the date that the financial statements are issued.


Set pelajaran terkait

Copyright Law: 12 Do's and Dont's

View Set

CompTIA Security Plus Architecture and Design Part 7

View Set

Exam 3 Study Guide: 14, 24, 25, 26, 27, 28, 29

View Set

Entrepreneurship Final Exam #1 (Ch. 1,3,4,7, 8, 17)

View Set

MNGT 301 || Chapter 3: The Manager's Changing Work Environment and Ethical Responsibilities: Doing the Right Thing

View Set

Patterns for a Purpose Chapters 1-3

View Set